You are on page 1of 155

KENDRIYA VIDYALAYA SANGATHAN

KOLKATA REGION

STUDENT SUPPORT MATERIAL


SESSION – 2023-24
CLASS XI
GEOGRAPHY

Coordinator:
SHRI MON BAHADUR CHETTRI
PRINCIPAL, KV SUKNA (KHAPRAIL)

Content Review and Moderator:

Mr. Kapil Deo Sah, (PGT Geo) KV Dum Dum OF


Dr. Himansu Sekhar Rana, (PGT Geo) Barrackpore-Army
Dr. T.P Singh, (PGT Geo) KV Farakka-NTPC
STUDENT SUPPORT MATERIAL
SESSION -2023-24
OUR PATRONS

SHRI Y. ARUN KUMAR

DEPUTY COMMISSIONER, KVS RO KOLKATA

SHRI SANJIV SINHA SHRI CH. VIJAYARATNAM SHRI DIWAKAR BHOI SHRI AMIT BAIDYA

ASSISTANT COMMISSIONERS, KVS RO KOLKAT

1
CONTENT

2
Book: Fundamentals of Physical Geography
Chapter No. Name of the Chapter Content Developed by Name of the Page
KV No.
01. Geography as a Discipline Mr. Bhupinder Singh Adra 7
02. Origin and Evolution of the Earth PGT (Geography) 13
03. Interior of the Earth Dr. Vandana Yadav Chittaranjan 18
PGT (Geography)
04. Distribution of Oceans and Mr. Jawed Alam Asansol 27
Continents PGT (Geography)
05. Geomorphic Processes Mr. Arvind Kumar Singh Ballygunge 41
PGT (Geography)
06. Landforms and their Evolution Dr. H. S. Rana Barrackpore 46
PGT (Geography) (Army)
07. Composition and Structure of Mr. Arvind Kumar Singh Ballygunge 53
Atmosphere PGT (Geography)
08. Solar Radiation, Heat Balance Mr. Jawed Alam Asansol 57
and Temperature PGT (Geography)
09. Atmospheric Circulation and Mr. Mahendra Kumar Bamangachi 66
Weather Systems PGT (Geography)
10. Water in the Atmosphere Mr. Premchand Kumar Gupta 75
11. World Climate and Climate PGT (Geography) Barrackpore 80
Change (AFS)
12. Water (Oceans) Mr. Biswajit Pal Cossipore 83
PGT (Geography)
13. Movement of Ocean Water Mr. Krishnendu Dutta Cooch Behar 87
PGT (Geography)
14. Biodiversity and Conservation Md. Zaffar Alam Command 94
PGT (Geography)
Book: India: Physical Environment
15. India: Location Mrs. Moonmoon Mitra CRPF, 103
PGT (Geography) Durgapur
16. Structure and Physiography Mr. Krishnendu Dutta Cooch Behar 111
PGT (Geography)
17. Drainage System Mr. Biswajit Pal Cossipore 122
PGT (Geography)
18. Climate Mr. R. B. Mishra Salt Lake No- 129
PGT (Geography) 2
19. Natural Vegetation Dr. H. S. Rana Barrackpore 137
PGT (Geography) (Army)
20. Natural Hazards & Disasters Mr. Amit Kumar Das Asansol 148
(now Vice-Principal)

3
CLASS XI
COURSE STRUCTURE

Book: Fundamentals of Physical Geography


Chapter Chapter Name Weightage
No.
Unit- I Geography as a Discipline
1 Geography as a Discipline 3
Unit II The Earth
2 The Origin and Evolution of the Earth
3 Interior of the earth 9
4 Distribution of Oceans and Continents
Unit III Landforms
5 Geomorphic Processes
6 Landforms and their Evolutions 6
Unit IV Climate
7 Composition & Structure of the Atmosphere
8 Solar Radiation, Heat balance and Temperature
9 Atmospheric Circulations and Weather Systems 8
10 Water in the Atmosphere
11 World Climate and Climate Change
(To be tested through internal assessments in the form of project
and presentation)
Unit V Water (Oceans)
12 Water (Oceans) 4
13 Movements of Ocean Water
Unit VI Life on the Earth
14 Biodiversity and Conservation
(To be tested through internal assessments in the form of project ---
and presentation)

Map Work 5
TOTAL 35

Book: India – Physical Environment


Chapter Chapter Name Weightage
No.
Unit- I Introduction
1 India : Location 5
Unit II Physiography
2 Structure & Physiography
3 Drainage System 13
Unit III Climate, Vegetation & Soil
4 Climate
5 Natural vegetation 12
Unit-IV Natural Hazards and Disasters: Causes Consequences and Management

6 Natural Hazards and Disasters (To be tested through internal ---


assessment in the form of Projects and presentation)
MAP WORK 5
TOTAL 35

4
MAP WORKS
Chapter Map work
Map Items for locating and labelling on outline political World Map Fundamentals of Physical Geography
Distribution of oceans ▪ Political Map of all Continents of the world.
and continents ▪ Major Oceans of the world: Indian Ocean, Pacific
Ocean, Atlantic Ocean, Arctic Ocean, Southern Ocean
▪ Major lithospheric plates and Minor lithospheric plates,
Ring of fire (Pacific Ocean), Mid-Atlantic Ridge.
Atmospheric Major Hot Deserts of the world:
Circulations and ▪ Mojave Desert- Nevada, US
Weather Systems ▪ Patagonian Desert- Argentina
▪ Sahara- Africa
▪ Gobi Desert- Mongolia, Asia
▪ Thar desert- India
▪ Great Victoria Desert- Australia

Water(Oceans) OCEAN CURRENTS-


Cold currents
▪ Humboldt c.
▪ California c.
▪ Falkland c.
▪ Canaries c.
▪ West Australian c.
▪ Oyashio c.
▪ Labrador c.
Warm currents
▪ Alaska c.
▪ Brazilian c.
▪ Aughlas c.
▪ Kuroshio c.
▪ Gulf stream c.
Biodiversity and Ecological hotspots
Conservation ▪ Eastern Himalaya, India
▪ Western Ghats, India
▪ Indonesia, Asia
▪ Eastern Madagascar, Africa
▪ Upper Guinean forests, Africa
▪ Atlantic forest, Brazil
▪ Tropical Andes
Map Items for locating and labelling on outline political map of India India Physical Environment
India- Location ▪ Latitudinal extent of India
▪ Longitudinal extent of India
▪ Standard Meridian of India
▪ Important latitude passing through India (Tropic of
Cancer)
▪ Southern Most Point of main land of India
(Kanniakumari)
Drainage System ▪ Rivers: Brahmaputra, Indus, Satluj, Ganga, Yamuna,
Chambal, Damodar Mahanadi, Krishna, Kaveri,
Godavari, Narmada, Tapti and Luni
▪ Lakes: (Identification)Wular, Sambhar, Chilika, Kolleru,
Pulicat & Vembanad

5
▪ Straits, Bays , Gulfs: Palk Strait, Rann of Kachch, Gulf
of Kachch, Gulf of Mannar & Gulf of Khambat
Climate ▪ Area with highest temperature in India
▪ Area with lowest temperature in India
▪ Area with highest rainfall in India
▪ Area with lowest rainfall in India
Natural Vegetation (Identification on an outline map of India) Tropical evergreen,
Tropical deciduous, Tropical thorn, Montane and Littoral/
Swamp forests. Wildlife reserves: (locating and labeling)
▪ National Parks: Corbett, Kaziranga, Ranthambore.
Shivpuri, Simlipal
▪ Bird Sanctuaries: Keoladev Ghana and Ranganathitto
▪ Wild life Sanctuaries: Periyar, Rajaji, Mudumalai,
Dachigam,

Guidelines for Internal Assessment/ Geography Practical


1. A practical file must be prepared by students covering all the topics prescribed in the practical
syllabus.
2. The file should be completely handwritten with a cover page, index page and
acknowledgment.
3. All practical works should be drawn neatly with appropriate headings, scale, index etc. Data
can be taken from the NCERT text book.
4. The practical file will be assessed at the time of term end practical examinations.
5. A written exam of 25 marks will be conducted based on prescribed practical syllabus.
6. Viva will be conducted based on practical syllabus only.
7. Written Exam -25 Marks
8. Practical file- 03 Marks
9. Viva- 02 Marks

6
Chapter-1
Geography as a Discipline
Multiple Choice Questions:

Q1. We study under physical geography:


(a) Weather, soil, atmosphere, etc.
(b) Agriculture
(c) Population Industry
(d) Urban and rural settlement.
Ans. (a) Weather, soil, atmosphere, etc.

Q2. Which of the following is not studied under economic geography?


(a) Agriculture
(b) Industry
(c) Transport
(d) Population.
Ans. (d) Population

Q3. Which of the following is not a sub-branch of Biogeography?


(a) Zoo Geography
(b) Plant Geography
(c) Human Geography
(d) Climate Geography
Ans. (d) Climate Geography

Q4. Geography is concerned with the description and explanation of the areal
differentiation of the earth’s surface. Who said it?
(a) Herodotus
(b) Erathosthenese
(c) Richarde Hartshorne
(d) Galileo.
Ans. (c) Richard Hartshome
Q5. Geography studies the differences of phenomena usually related in different
parts of the earth’s surface. Who gave this definition?
(a) Hambolt
(b) Ratzel
(c) K. Sample
(d) Alfred Hartner
Ans. (d) Alfred Hertner

Q6. Which of the following is not studied under population geography?


(a) Sex Ratio
(b) Migration and Occupational Structure
(c) Pollution

7
(d) Population Density.
Ans. (c) Pollution

Q7. Who developed systematic geography?


(a) Hambolt
(b) Karl Ritter
(c) K. Sample
(d) Alfred Hartner
Ans. (a) Hambolt

Q8. Who developed regional geography?


(a) Hambolt
(b) Karl Ritter
(c) K. Sample
(d) Alfred Hartner
Ans. (b) Karl Ritter

Q9. Under which approach is the world divided into regions at different hierarchical
levels and then all the geographical phenomena in a particular region are studied.
(a) Systematic Approach
(b) Physical Approach
(c) Dualism Approach
(d) Regional Approach
Ans. (d) Regional Approach

Q10. Under which approach, a phenomenon is studied world over as a whole, and
then the identification of typologies or spatial patterns is done?
(a) Systematic Approach
(b) Physical Approach
(c) Dualism Approach
(d)Regional Approach
Ans. (a) Systematic Approach

Short answer type Questions (3 marks):

Q11. What is studied under Environment Geography?


Ans. It is concerned with environmental problems such as land gradation, pollution and
environment conservation.

Q12. What are main branches of Biogeography?


Ans. It has three branches: Plant Geography, Zoo Geography and Ecology.

8
Q13. Differentiate between Physical geography and Biogeography.
Ans. The main differences between Physical geography and Biogeography are given below:
Basis Physical Geography Bio geography
Meaning It has developed as a It has emerged as a result
subject concerned with of the interface between
study of evaluation and physical geography and
management of natural human geography
resources
Branches It has three sub branches: It has three branches: Plant
Geomorphology, Geography, Zoo
climatology and hydrology. Geography and Ecology.
Subject matter It studies abiotic elements It studies biotic elements of
of the earth. the earth.

Q14. Define the term ‘Geography’.


Ans. The term geography was first coined by Eratosthenese, a Greek scholar (276¬194
BC.). The word has been derived from two Greek words geo (earth) and graphos
(description). Put together, they mean description of the earth. The earth has always been
seen as the abode of human beings and thus, scholars defined geography as, “the
description of the earth as the abode of human beings”.

Q15. What is studied under Environment Geography?


Ans. It is concerned with environmental problems such as land gradation, pollution and
environment conservation.

Q16. What are the recent techniques that helped the geographer to understand the
earth’s surface better?
Ans. GIS and GPS, Computer cartography

Q17. Explain the changes occurred in the civilization of man in course of time?
Ans. Many changes have occurred in the civilization of man in course of time.
Man moved from stage of necessity to stage of freedom.
Created new possibilities from the nature.
We find now humanized nature and naturalized man.
Space got organized with the help of transport and communication.

9
Long answer questions (5 marks):

Q18. What is the importance of Physical Geography?


Ans. Physical geography includes study of Lithosphere, Atmosphere, Hydrosphere and
Biosphere. Each element is very important for human beings. Landforms provide base for
agriculture, industries, transport and communication, and settlements. Mountains provide
water to rivers, forests-center for tourist spots. Climate influences on the cropping pattern,
livestock, food and clothes of the people. Climate and precipitation influence the type of
forests. Oceans provide food, water transport, and influence the climate; they are the source
of hydrological cycle.
Each element of physical environment is important for human beings. Landforms provide
the base on which human activities are located. The plains are utilised for agriculture.
Plateaus provide forests and minerals. Mountains provide pastures, forests, tourist spots
and are sources of rivers providing water to lowlands. Climate influences our house types,
clothing and food habits.
The study of physical geography is emerging as a discipline of evaluating and managing
natural resources. In order to achieve this objective, it is essential to understand the intricate
relationship between physical environment and human beings. Physical environment
provides resources, and human beings utilise these resources and ensure their economic
and cultural development. Accelerated pace of resource utilisation with the help of modern
technology has created ecological imbalance in the world. Therefore, a better understanding
of physical environment is essential in study of Geography.

Q19. Physical and human factors both are dynamic not static. Explain.
Ans. The geographical phenomena, both the physical and human, are not static but highly
dynamic. They change over times as a result of the interactive processes between ever
changing earth and untiring and ever-active human beings.
1. Primitive human societies were directly dependent on their immediate environment.
Human beings have come to terms with nature through adaptation and modification.
2. The present society has passed the stage of primitive .societies, which were directly
dependent on their immediate physical environment for sustenance. Present societies have
modified their natural environment by inventing and using technology and thus, have
expanded the horizon of their operation by appropriate utilisation of the resources provided
by nature.
3. With the gradual development of technology, human beings were able to loosen the
shackles of their physical environment. Technology helped in reducing the harshness of
labour, increased labour efficiency and provided leisure to human beings to attend to the
higher needs of life. It also increased the scale of production and the mobility of labour. The
interaction between the physical environment and human beings has put their imprints
everywhere and created new possibilities in collaboration with nature. Thus, we find
humanised nature and naturalised human beings and geography studies this interactive
relationship.
4. The space got organised with the help of the means of transportation and communication
network. The links (routes) and nodes (settlements of all types and hierarchies) integrated
the space and gradually, it got organised.

10
5. It takes note of the associations and interrelationships between the phenomena over
space and interprets them providing explanations for these patterns. It also takes note of the
associations and inter-relationships between the phenomena resulting from the dynamic
interaction between human beings and their physical environment.

Q20. Explain different branches of Physical geography.

Ans. It has four sub-branches which are as follows:


Geomorphology: It is concerned with the study of landforms, their evolution and related
processes.
Climatology: It is concerned with the study of structure of atmosphere and elements of
weather and climates and climatic types and regions.
Hydrology: It studies the realm of water over the surface of the earth including oceans,
lakes, rivers and other water bodies and its effect on different life forms including human life
and their activities.
Soil Geography: It is concerned with the study of the processes of soil formation, soil types,
their fertility status, distribution and use.

Q21. What matters are studied under Human Geography?


Ans. Following facts are studied under Human Geography:
Social/Cultural Geography: It is concerned with the study of society and its spatial
dynamics as well as the cultural elements contributed by the society.
Population Geography: It studies population growth, distribution, density, sex ratio,
migration and occupational structure, etc.
Settlement Geography: It studies the characteristics of rural and urban settlements.
Economic Geography: It studies economic activities of the’people including agriculture,
industry, tourism, trade, and transport, infrastructure and services, etc.
Historical Geography: It studies the historical processes through which the space gets
organised. In other words, it studies how history has influenced the geography of a region.
Political Geography: It studies the impact of political events and studies boundaries, space
relations between neighbouring political units, delimitation of constituencies, election
scenario and develops theoretical framework to understand the political behaviour of the
population.

Q22. Geographers play an important role for a country. How?


Ans. Geographers are important for any country because:
They study the variations and association of the features on the earth surface e.g. cropping
pattern differs from place to place and it is due to difference in the climate, soil, demand,
transport facility and capacity of the farmer.
A geographer also studies the cause and effect relationships.
The interaction between man and nature is highly dynamic and not static; so it is also called
as the study of the relation between unstable earth and untrusting man.
He helps to predict climate.
He helps in handling natural calamities.

11
Source Based Questions:
1.1 Geography is a discipline of synthesis. It attempts spatial synthesis & history attempts
temporal synthesis. Its approach is holistic in nature. It recognizes the fact that the world is
a system of interdependencies. It recognises the fact that the world is a system of
interdependencies. The present world is being perceived as a global village. The distances
have been reduced by better means of transportation increasing accessibility. The audio-
visual media and information technology have enriched the data base.

Read the above paragraph & answer the following questions in your own words:

a. How would you distinguish between Geography & History?


b. What is the approach of Geography & how it recognizes the world?

1.2 Dualism is one of the main characteristics of Geography which got introduced from the
very beginning. This dualism depended on the aspect emphasized in the study. Earlier
scholars laid emphasis on Physical Geography. But human beings are an integral part of
the earth’s surface. They are part & parcel of nature. They also have contributed through
their cultural development. Thus, developed human geography with emphasis on human
activities.

Read the above paragraph & answer the following questions in your own words:

a. How did dualism in physical & human geography develop?


b. What is the main emphasis of human geography?

12
Chapter- 2
Origin and Evolution of Earth
Multiple Choice Questions:
Q.1 Which one is the universally accepted theory of Earth’s origin.
a. Binary star b. Big Bang Theory
c. Nebular Hypothesis d. Tidal
Ans: b. Big Bang Theory

Q.2 Which hypothesis was propounded by James and Jeffery?


a. Nebular b. Binary Theory
c. Nuclear d. Supernova hypothesis

Ans: b. Binary Theory

Q.3 What is the distance in one light year.


a. 9.46*1012 KM b. 9.46*1010 KM
11
c. 9.46*10 KM d. 9.64*1012 KM

Ans: a. 9.46*1012 KM
Q.4 Three gases of solar nebula are:
a. Hydrogen, Methane, CO2 b. Hydrogen, Methane, Dust particle
c. Hydrogen, Helium, Dust particle d. Hydrogen, Oxygen, Dust Particle

Ans: c. Hydrogen, Helium, Dust particle

Q.5 How many inner planets are in our solar system?


a. 5 b. 6
c. 3 d. 4
And: d. 4

Q.6 Which of the following has the longest duration?


a. Eons b. Era
c. Period d. Epoch

Ans: a. Eons

Q.7 Which one of the following represent the inner planets?


a. Planets between Sun and Earthb.Planets between sun and belt of asteroids
c. Planets with satellite d. Planets with out satellite
Ans: a. Planets between Sun and Earth

Q.8 Life on the earth surface appear around how many years before the present time?

13
a. 13.7 billion years ago b. 3.8 millions years ago
c. 4.6 billions years ago d. 3.8 billions years ago
Ans: d. 3.8 billions years ago

Q.9 Approx how many years ago Moon came into existence?
a. 2.44 billion years ago b. 4.44 billions years ago
c. 3.44 billions years ago d. 5.44 billions years ago
Ans: b. 4.44 billions years ago

Q.10 ‘Belt of Asteroid’ found between which two planets.


a. Mercury and Venus b. Mars and Jupitar
c. Earth and Mars d. Jupitar andSaturn

Ans: b. Mars and Jupitar

Q.11. Which one of the following is not related to the formation and modification of
present in atmosphere?

a. Solar Wind b. Degassing


c. Differentiation d. Photosynthesis
Ans: c. Differentiation

Q.12 Who suggested that the moon and earth formed from the single rotating body?
a. Sir George Darwin b. James
c. Emanual Kant d. Hubble
Ans: a. Sir George Darwin

Ans: a. Sir George Darwin

Q.13 Light year is unit of measure which of the following?


a. Light b. Time
c. Distance d. All of the Above
Ans: c. Distance

14
Section: B (3 Marks Question
Q.1 What is meant by process of differentiation?
Ans: Starting from the surface to the central parts, we have layers like the crust, mantle,
outer core and inner core. From the crust to the core, the density of the material increases.
This process of the earth forming material got separated into different layers is called
differentiation.
Q.2 What was the Nature of earth initially?
Ans: The planet earth initially was a barren, rocky and hot object with a thin atmosphere of
hydrogen and helium. This is far from the present day picture of the earth. It is said that in
initial stage the earth was in liquid form. Certainly, there must have been some events-
processes, which may have caused this change from rocky, barren and hot earth to a
beautiful planet with ample amount of water and conducive atmosphere favouring the
existence of life.
Q.3 Differentiate between the concept of earth origin given by:
a. Kant and Laplash b. Chamberlin and Molten
Ans:
1. Kant and Laplace’s Principle: The hypothesis considered that the planets were formed
out of a cloud of material associated with a youthful sun, which was slowly rotating.
According to this principle which emerged in 1796, the interior of the earth must be gaseous
because the earth has originated from gas form.

2. Chamberlain and Moulton: In 1900, Chamberlain and Moulton considered that a


wandering star approached the sun. As a result, a cigar-shaped extension of material was
separated from the solar surface. As the passing star moved away, the material separated
from the solar surface continued to revolve around the sun and it slowly condensed into
planets. Later on, the arguments considered of a companion to the sun to have been
coexisting. These arguments are called binary theories.

Q.4 Why inner planets are rocky in nature?


Ans:

• The terrestrial planets were formed in the close vicinity of the parent star where it was
too warm for gases to condense to solid particles.
• The solar wind was most intense nearer the sun; so, it blew off lots of gas and dust
from the terrestrial planets.
• The terrestrial planets are smaller and their lower gravity could not hold the escaping
gases

Q.5 What do you mean by Binary star hypothesis?


Ans:This concept of earth origin consider that solar system originateform two stars.
According to this due to massive gravitation farce of ‘intruding star’ huge amount of material
ejected form the primitive sun, which later on become the building material for future
planets.
15
Q.6 Write a short note on big splat theory of moon’s origin.
Ans: A body of size one to three times that of mars collided into earth sometime after
formation of earth it blasted large part of earth into the space. This material started to rotate
around the earth.
Q.7 How does life originate on the earth surface, briefly explain?
Ans: Initially the earth or even the atmosphere of the earth was not conducive for the
development of life. Modern scientist believes that origin of life is one kind of chemical
reaction, took place in the oceans. Due to lightning, the complex organic molecules were
combined into a certain form which can duplicate themselves. They are called first single
cell animals. They are able to convert inanimate things into animate things.

Section: C (5 Marks Question)

Q.1 Write a note on Bing Bang theory?


Ans:
i. In the beginning, all matter was in the form of tiny ball (singular atom) with
unimaginable small volume, infinite temperature and infinite density.
ii. About 13.7 billion years ago the tiny ball exploded violently. The expansion
continues even today.
iii. As a result, some energy was converted into matter.
iv. Within fraction of second there was rapid expansion.
v. The expansion slowdown after three minutes and first atom formed.
vi. After 300000 years the temperature dropped down to 4,500 K and gave rise to
atomic matter.
vii. The universe becomes transparent.

Q.2 What were the stages of evolution of lithosphere?


Ans:
i. There was volatile state during its primordial stage
ii. Due to high-density temperature increased
iii. The material started separating depending on their density Light material came
outside and heavy material went inside the earth
iv. It cooled and condensed into solid which is called lithosphere
v. At the time of formation of the moon, the earth again became hot
vi. Due to differentiation different layers formed

16
Q.3 How does atmosphere and hydrosphere originate on the earth surface. Explain.
Ans:
There are three stages of atmosphere and hydrosphere formation.
Stage I - The early atmosphere consists of hydrogen and helium. This primordial
atmosphere lost due to solar winds.
Stage- II – i. Gases were released from the earth’s interior such as Water vapor and other
gases.
ii. There were water vapor, nitrogen, carbon dioxide methane, ammonia and
little free oxygen.
iii. The process of outpouring the gases from the interior of the earth is called
degassing.
iv. Volcanic eruptions contributed the water vapor and CO 2. After cooling of
earth condensation stated.
v. Rainwater collected into the depressions called oceans.
Stage III- Living organisms changed the composition of the atmosphere due to
photosynthesis.

Source based Question


2. Out of the eight planets, Mercury, Venus, Earth & Mars are called inner planets as they lie
between the sun & the belt of asteroids, the other four planets are called outer planets.
Alternatively, the first four are called Terrestrial, meaning earth-like as they are made up of
rock & metals, & have relatively high densities. The rest four are called Jovian or Gas Giant
Planets. Jovian means Jupiter-like.
Read the above paragraph & answer the following questions in your own words:

a. Mention two different types of planets of our solar system.


b. How are Terrestrial planets different from Jovian planets?

3. The moon is the only natural satellite of the earth. Like the origin of the earth, there have
been attempts to explain how the moon was formed. In 1838, Sir George Darwin suggested
that initially, the earth & the moon formed a single rapidly rotating body. The whole mass
became a dumb-bell-shaped body & eventually it broke. It was also suggested that the
material forming the moon was separated from what we have at present the depression
occupied by Pacific Ocean.
Read the above paragraph & answer the following questions in your own words:

a. How was the moon originated?


b. What is the proof that the moon got separated from the earth?

17
Chapter - 3

INTERIOR OF THE EARTH

Multiple-Choice Questions

Q.1. Which region of the earth have the greatest density?

(a) The crust

(b) The mantle

(c) Outer core

(d) Inner core

Ans. (d) Inner core

Q.2. Core is made of which metals?

(a) Iron and Magnesium

(b) Iron and Silicon

(c) Nickel and Iron

(d) Nickel and Silicon

Ans. (d) Nickel and Iron

Q.3. What is the approximate depth of mantle?

(a) 2400 km

(b) 2900 km

(c) 3200 km

(d) 3500 km.

Ans. (b) 2900 km

Q.4. Density of the core is the highest because-

(a) Due to heavy pressure of overlying rocks

(b) Due to heavy temperature of overlying rocks

(c) Due to heavy density of overlying rocks

(d) None of the above.

Ans. (a) Due to heavy pressure of overlying rocks

18
Q.5. Magma refers to.............

(a) Rocks

(b) Material in upper mantle

(c) Material in upper core

(d) Material in the crust

Ans. (b) Material in upper mantle

Q.6. Fast moving waves which reach the earth earliest are called:

(a) Alpha Waves

(b) S-Waves

(c) P-Waves

(d) Beta Waves.

Ans. (c) P-Waves

Q.7. Which waves cannot pass through liquid materials?

(a) Primary Waves

(b) Secondary waves

(c) Surface waves

(d) All of the above

Ans. (b) Secondary waves

Q8. Match the following and choose the correct option:

Intrusive structure Definition

(a) Lapolith A. Horizontal Intrusions


(b) Dykes B. Saucer shaped, concave structures
(c) Phacofith C. Vertical intrusions
(d) Sill D. Wavy mass found in folded structures

(a) 1-D,2-A, 3-B, 4-C

(b) 1-A, 2-B, 3-C, 4-D

(c) 1-D, 2-C, 3-A, 4-B

(d) 1-B, 2-C, 3-D, 4-A

Ans. (d)

19
Q.9. Which one of the following describes the lithosphere?

(a) Upper and middle mantle

(b) Crust and Upper mantle

(c) Crust and core

(d) Mantle and core

Ans. (b) Crust and Upper mantle

Q.10. The upper portion of mantle is called:

(a) Asthenosphere

(b) Crust

(c) Lithosphere

(d) Fossil Sphere.

Ans. (a) Asthenosphere

LONG ANSWER TYPE QUESTIONS

Q.1. What are plutonic rocks? Describe major intrusive structures .

Ans. The cooling of magma may take place either on reaching the surface or also while the
lava is still in the crustal portion. When cooling of magma takes place below or within crust
they forms plutonic rocks. On the basis of shapes intrusive structures can be classified as:

1. Lapolith: As and when the lava moves upwards, a portion of the same may tend to move
in a horizontal direction wherever it finds a weak plane It may get rested in different forms.
In case it develops into a saucer shape, concave to the sky body, it is called lapolith.

2. Phacolith: A wavy mass of intrusive rocks, at times, is found at the base of synclines or at
the top of anticline in folded igneous country. Such wavy materials have a definite conduitto
source beneath in the form of magma chambers (subsequently developed as batholiths).
These are called the phacoliths.

3. Lacoliths: These are large dome-shaped intrusive bodies with a level base and connected
by a pipe-like conduit from below. It resembles the surface volcanic domes of composite
volcano.

4. Sills: The near horizontal bodies of the intrusive igneous rocks are called sill or sheet,
depending on the thickness of the material. The thinner ones are called sheets while the
thick horizontal deposits are called sills.

20
5. Dykes: When the lava makes its way through cracks and the fissures developed in the
land, it solidifies almost perpendicular to the ground. These vertical structures are called
dyke.

6. Batholiths: A large body of magmatic material that cools in the deeper depth of the crust
develops in the form of large domes. These are granitic bodies. Batholiths are the cooled
portion of magma chambers.

Q.2. Explain different types of earthquakes.

Ans. The various types of earthquakes are: -

• Collapse earthquake: In the areas of intense mining activity, sometimes the roofs of
underground mines collapse causing minor tremors. These are called collapse earthquakes.

• Tectonic earthquake: The most common ones are the tectonic earthquakes. These are
generated due to sliding of rocks along a fault plane.

• Volcanic earthquake: A special class of tectonic earthquake is sometimes recognized as


volcanic earthquake. However, these are confined to areas of active volcanoes.

• Reservoir Induced earthquake: The earthquakes that occur in the areas of large reservoirs
are referred to as reservoir induced earthquakes. Sometimes earthquakes also occur in
mines due to mining processes. Sometimes earthquakes also occur below the oceans on
surface of the ocean causing tsunamis.

• Explosion earthquake: Ground shaking may also occur due to the explosion of chemical or
nuclear devices. Such tremors are called explosion earthquakes.

Q.3. What are different sources of information about the interior of the earth?

Explain with examples.

Ans. Some of the direct sources are:

• Magnetic surveys: Magnetic surveys also provide information about the distribution of
magnetic materials in the crustal portion, and thus, provide information about the distribution
of materials in this part.

• Earthquake: It is simple shaking of the earth, through Earthquake waves we get better
Insight about the earth Interior.

• Mining: It is a process by which commercially valuable mineral resources are extracted


from Earth's surface which includes precious stones, rocks and solid fuels.

• Volcanic Eruptions: When molten material is thrown onto the surface of the earth during
volcanic eruption it becomes available for analysis.

Some of the indirect sources of information:

21
• Meteors: The material and the structure observed in the meteors are similar to that of the
earth. They are solid bodies developed out of materials same as, or similar to, our planet.
Hence, this becomes yet another source of information about the interior of the earth.

• Gravitation: The gravitation force (g) is not the same at different latitudes on the surface. It
is greater near the poles and less at the equator. This is because of the distance from the
centre at the equator being greater than that at the poles. The gravity values also differ
according to the mass of material.

• Drilling: Scientists world over are working on two major projects such as "Deep Ocean
Drilling projects" and "integrated ocean drilling project". The deepest drill at kola, in Arctic
Ocean, has so far reached a depth of 12 km.

Q.4. Describe the major type of volcanoes found in the world.

Ans. "Volcanoes are classified on the basis of nature of eruption and the

form developed at the surface. Major types of volcanoes are as

follows:

1. Shield Volcanoes: the shield volcanoes are the largest of all the volcanoes on the earth.
These volcanoes are mostly made up of basalt, a type of lava that is very fluid when
erupted. For this reason, these volcanoes are not steep.

2. Composite Volcanoes: These volcanoes are characterized by eruptions of cooler and


more viscous lavas than basalt. These volcanoes often resulting explosive eruptions. Along
with lava, large quantities of pyroclastic material and ashes find their way to the ground.
This material accumulates in the vicinity of the vent openings leading to formation of layers,
and this makes the mounts appear as composite volcanoes.

3. Caldera: These are the most explosive of the earth's volcanoes. They are usually so
explosive that when they erupt they tend to collapse on themselves rather than building any
tall structure. The collapsed depressions are called calderas.

4. Flood Basalt Provinces: These volcanoes outpour highly fluid lava that flows for long
distances. Some parts of the world are covered by thousands of sq. km of thick basalt lava
flows. There can be a series of flows with some flows attaining thickness of more than50 m.
The Deccan Traps from India are a much larger flood basaltprovince.

5. Mid-Ocean Ridge Volcanoes: These volcanoes occur in the oceanic areas. There is a
system of mid-ocean ridges more than70,000 km long that stretches through all the ocean
basins. The central portion of this ridge experiences frequent eruptions.

Q.5. Describe the interior structure of the earth.

Ans. On the basis of direct and indirect evidences earth's interior can be divided into three
layers:

22
1. The Crust: It is the outermost solid part of the earth, The thickness of the crust varies
under the oceanic and continental areas. The mean thickness of oceanic crust is 5 km
whereas that of the continental is around 30 km. It has very low density and majorly made
up of silica and aluminium.

2. The Mantle: The portion of the interior beyond the crust is called the mantle. The mantle
extends from Moho's discontinuity to a depth of 2,900 km. The upper portion of the mantle
is called asthenosphere. The word astheno means weak. It is considered to be extending
upto 400 km. It is the main source of magma that finds its way to the surface during volcanic
eruptions. The crust and the uppermost part of the mantle are called lithosphere. Its
thickness ranges from 10-200 km. The lower mantle extends beyond the asthenosphere. It
is in solid state.

Q.6. Explain different types of earthquake waves.

Ans.

Types of earthquake waves

Body waves Surface waves Primary waves or P-waves

Secondary waves or S-waves

Earthquake waves are basically of two types body waves and surface waves.

• Body waves: These are generated due to the release of energy at the focus. They move in
all directions travelling through the body of the earth. These are less destructive than the
surface waves.

• S-Waves: S-waves arrive at the surface with some time lag. These are called secondary
waves. An important fact about S-waves is that they can travel only through solid materials.
This characteristic of the S-waves is quite important. It has helped scientists to understand
the structure of the interior of the earth. Reflection causes waves to rebound whereas
refraction makes waves move in different directions. The variations in the direction of waves
are inferred with the help of their record on seismograph.

• Surface waves: The body waves interact with the surface rocks and generate new set of
waves called surface waves. These waves move along the surface. These waves are more
destructive. The surface waves are the last to report on seismograph. These waves are
more destructive. They cause displacement of rocks, and hence, the collapse of structures
occurs.

There are two types of body waves. They are called P- and S-waves,

• P-waves: They move faster and are the first to arrive at the surface. These are also called
primary waves'. The P-waves are similar to sound waves. They travel through gaseous,
liquid and solid materials.

23
Source Based Question

All natural earthquakes take place in the lithosphere. It is sufficient to note here that the
lithosphere refers to the portion of depth up to 200 km from the surface of the earth. An
instrument called ‘seismograph’ records the waves reaching the surface. A curve of
earthquake waves recorded on the seismograph. Note that the curve shows three distinct
sections each representing different types of wave patterns. Earthquake waves are basically
of two types — body waves and surface waves. Body waves are generated due to the
release of energy at the focus and move in all directions travelling through the body of the
earth. Hence, the name body waves. The body waves interact with the surface rocks and
generate new set of waves called surface waves. These waves move along the surface.
The velocity of waves changes as they travel through materials with different densities. The
denser the material, the higher is the velocity. Their direction also changes as they reflect or
refract when coming across materials with different densities. There are two types of body
waves. They are called P and S-waves. P-waves move faster and are the first to arrive at
the surface. These are also called ‘primary waves’. The P-waves are similar to sound
waves. They travel through gaseous, liquid and solid materials. S-waves arrive at the
surface with some time lag. These are called secondary waves. An important fact about S-
waves is that they can travel only through solid materials. This characteristic of the S-waves
is quite important. It has helped scientists to understand the structure of the interior of the
earth. Reflection causes waves to rebound whereas refraction makes waves move in
different directions. The variations in the direction of waves are inferred with the help of their
record on seismograph. The surface waves are the last to report on seismograph. These
waves are more destructive. They cause displacement of rocks, and hence, the collapse of
structures occurs.

(i) Which one of the following earthquake waves is more destructive?

(a) P-waves

(b) Surface waves

(c) S-waves

(d) None of the above

Ans- (a) P-waves

(ii) Which of the following sequences correctly lists the different arrivals from first to
last?

(a) P waves ... S waves .... Surface waves


(b) Surface waves ... P waves .... S waves
(c) P waves ... Surface waves ... S waves
(d) S waves ... P waves .... Surface waves

Ans- (a) P waves ... S waves .... Surface waves

24
(iii) Body waves consist of the

(a) P waves only


(b) S waves only
(c) P and S waves
(d) Surface waves
Ans- (c) P and S waves

(iv) Fast moving waves which reach the earth earliest are called:
(a) Alpha Waves
(b) S-Waves
(c) P-Waves
(d) Beta Waves.
Answer:(c) P-Waves
Earthquake waves get recorded in seismographs located at far off location. The earthquake
events are scaled either according to the magnitude or intensity of the shock. The
magnitude scale is known as the Richter scale. The magnitude relates to the energy
released during the quake. The magnitude is expressed in numbers, 1-10. The intensity
scale is named after Mercalli, an Italian seismologist. The intensity scale takes into account
the visible damage caused by the event. The range of intensity scale is from 1-12.

Earthquake waves get recorded in seismographs located at far off location

(i) How do we measure magnitude of earthquake?

(a) Richter Scale

(b) Mercalli Scale

(c) Measuring Scale

(d) Seismograph.

Answer: (a) Richter Scale

(ii) How do we measure intensity of earthquake?

(a) Richter Scale

(b) Mercalli Scale

25
(c) Measuring Scale

(d) Seismograph.

Ans- (b) Mercalli Scale

(iii) Who invented the Richter scale?

(a) Mercalli
(b) Charles Richter
(c) John Milne
(d) Giuseppe Mercalli

Ans- (b) Charles Richter

(iv)What is the range Richter scale?

(a) Value of 0-10


(b) Value of 1-9
(c) Value of 1-10
(d) Value of 2-9

Ans- (c) Value of 1-10

26
Chapter – 4
Distribution of Oceans and Continent
CONTINENTAL DRIFT: Abraham Ortelius a Dutch map maker 1596 first proposed the
possibility of joining the continents such as America with Europe and Africa Antonio
Pellegrini drew the map showing the three continents together.
Alfred Wegener a German meteorologist put forth The Continental Drift Theory. According
to him:
All continents formed a single continental mass called Pangaea.
All oceans formed a single universal ocean called Panthalassa.
Around 200 million year ago the Pangaea began to split into two large masses
called Laurasia and Gondwanaland
By further splitting Laurasia formed northern continents and Gondwana land formed
southern continents.

EVIDENCES IN THE FAVOUR OF CONTINENTAL DRIFT

1. THE MATCHING OF CONTINENTS (JIG-SAW FIT)


A. the shorelines of S. America and Africa have remarkable match
B. a map was produced by Bullard in 1964 to show the jigsaw fit of these two continents.
C. it was fit around 1000 fathom line of the shoreline

2. ROCKS OF SAME AGE ACROSS THE OCEANS


A. the belt of ancient rocks of 2000 my from Brazil coast matches with those of Western
Africa
B. Marine deposits of South America and Africa belong to Jurassic age.

3. TILLITE
A. sedimentary rock formed out of glacial deposits
B. sediments from India have similar counter parts at different continents of south.
C. tillile indicates prolonged glaciations
D. The same glaciations is found in Africa, Falklands, Madagascar, Antarctica and Australia
E. the glacial tillite indicates that unambiguous evidence of palaeo climates and drifting of
continents.

4. PLACER DEPOSITS
a. Formation of placer deposits of gold in Ghana coast has no source rock.
a. The gold bearing veins of rocks are found in Brazil

5. DISTRIBUTION OF FOSSILS
• Identical species of animals and plants are found along the coastal regions of the different
continents.
• lemurs occurs in India, Madagascar and Africa.
• The contiguous Land mass was called LEMURIA
• the fossils of Mesosaurus were found in only South Africa and Brazil.

27
FORCES FOR DRIFTING THE CONTINENTS
1. Wegner suggested that the movement responsible for the drifting of the continents was
caused by:
A. Polar Fleeing Force
B. Tidal Force

Possible driving forces for plate tectonics:


2. bottom lithosphere tractions by convection currents.
3. trench pull (covered earlier).
4. ridge push (sliding off a high, crust in compression).
5. trench suck (rollback).
6. global expanding or contracting forces.
7. membrane forces on spinning ellipsoid (e.g. variants of polar fleeing forces).

TIDAL FORCE
Wegener suggested that these two forces are responsible for the movement of continents.
Most of the scholars consider that these forces are not sufficient to move the plates.

POST DRIFT STUDIES


Information collected from the ocean mapping is more useful to study the continental drift

Convectional current theory


It was proposed by ARTHUR HOLMES IN 1930 Due to difference in the temperature
currents are formed due to disintegration of radioactive materials inside the earth.

These currents are found entire mantle

28
OCEAN FLOOR MAPPING
1. Existence of ridges and deep trenches nearby continental margins
2. Mid oceanic ridge is the most active for volcanic eruptions
3. The ocean floor is much younger than the continents
4. Rocks of equal distance of the ridge have similar chemical composition and age

OCEAN FLOOR CONFIGURATION


The ocean floor is segmented into three major divisions Based on depth and
configuration
1. Continental margins
a. Form transitional zone between continental shore and deep-sea basins
b. They include continental slope, shelf, continental rise and deep oceanic trenches
ABYSSAL PLAINS

1. EXTENSIVE PLAINS
2. FOUND BETWEEN CONTINENTAL MARIGN AND MID OCEANIC RIDGE
3. CONTINENTAL SEDIMENTS GET DEPOSITED

DISTRIBUTION OF VOLCANOES AND EARTHQUAKES


1. all volcanoes and earthquakes are parallel to the coast
29
2. this line also coin sides with mid- Atlantic ridge
3. alpine Himalayan system
4. around the Pacific Ocean it is called ring of fire Mid oceanic ridges

CONCEPT OF SEA FLOOR SPREADING


1 .it was proposed by Hess in 1961
2. he believed that new lava pushes out the plates from the mid oceanic ridge
3. palaeo magnetic studies of the ocean floor reveals that
A. along the mid oceanic ridge there is intense volcanic eruption
B. huge amount of lava comes out along the mid-Atlantic ridges
C. the equidistant rock formations have similar age and chemical compositions & magnetic
properties
7. rocks closer to the mid oceanic ridges are young and normal polarity
8. The age of rocks increases as the distance increases from the mid oceanic ridge
9. Oceanic crust is much younger(200my) than continental crust (3200my)
10. The sediments of ocean floor is very thin
11. earth quakes are common along the deep-sea trenches Positions of continents through
geological past

PLATE TECTONICS
1. The theory of plate tectonics was introduced by McKenzie, parker and Morgan in 1967
2. A tectonic plate is also called as lithosphere plate
3. It is a massive irregularly shaped slab of solid rock
4. Consists of oceanic and continental sphere
5. Plates move horizontally over the Asthenosphere
6. Average thickness is 100 km of oceanic part and 200 km continental part
7. It may be oceanic or continental
8. Pacific plate is largest oceanic plate
whereas Eurasian plate is the largest continental plate

MAJOR PLATES OF THE EARTH CRUST


MAJORPLATES

1. Antarctica And Surrounding OceanicPlate


2. North American Plate
3. South American Plate
4.Pacific Plate
5.India-Australia-New Zealand PLATE
6.African Plate Eurasian Plate
MINOR PLATES

1. Cocos Plate
2. Nazca Plate
3. Arabian Plate
4. Philippine Plate

30
5. Caroline Plate
6. Fuji Plate
These plates are moving constantly throughout geological time not the continent believed by
Wegener Pangaea was the convergent of all the plates

TYPES OF PLATE BOUNDARIES

I. DIVERGENT BOUNDARIES
1. New crust is generated 2. plates move away from each other
2. These are called spreading sites 3. Ex. Mid Atlantic ridge

II. CONVERGENT BOUNDARY


1. Crust is destroyed
2. sinking of plate is called ‘subduction zone’
3. There are three ways in which subduction occurs

III TRANSFORM BOUNDARIES


1. Crust is neither produced nor destroyed
2. Plates slide horizontally
3. Perpendicular to the mid oceanic ridges
4. Differential movement of a plate at the same time
5. Rotation of the earth has its effect on this movement

TYPES OF PLATES
i. Ocean and continent ii. Ocean and ocean iii. continent and continent plates

RATES OF PLATE MOVEMENT


1. The strips of normal and reverse magnetic field helped the scientists to study the rate of
plate movement
2. Arctic ridge has the slowest rate less than 2.5 cm /year east pacific rise has more than
3. 15 cm/year

FORCES OF THE PLATE MOVEMENT


31
1. Surface of the earth is dynamic
2. Interior is always mobile
3. Beneath the lithosphere there is always movement of magma horizontally
4. Heated material rises to the top and cooled material sinks down
5. This cycle is repeated over the time and form convection cells
6. It was first considered by Arthur Holmes in 1930
7. Later it also influenced Harry Hess

MOVEMENT OF INDIAN PLATE


1. Indian plate includes India and Australia
2. Northern boundary is along the Himalayas
3. It is the place of continental convergence
4. In the east it extends up to Rakinyoma mountains of Myanmar
5. Eastern margin is spreading site
6. Western margin extends along Kirthar mountains, Makran coast red sea rift.
7. The boundary between India and Antarctica is called divergent boundary
8. Till 225 m y a India was separated by Tethys sea
9. About 200 m y a India started its journey towards north
10. India collided with Asia about 40-50 m y a and caused the upliftment of Himalayas
11. About 140 MYAthe position of Indian plate is at 50°s latitude
12. During the movement of Indian plate two events occurred in India
13. A. out pouring of lava and formation of Deccan plateau
B. Subsidence of west coast
14. The Himalayas started rising about 40 million year ago

Multiple Choice Questions:

1.) What are the organisms that live in salt water?


A.) Mesosaurus B.) Lemuria C.) HerbivoresD.) Ocean lives
2.) Cause of continental drift according to Wagner
A.) Magnetic force and polar fleeing force B.) Polar fleeing force and tidal force
C.) Mesosaurus and tidal force D.) None of the above
3.) Who first proved the effect of convection currents in mantle region?
A.) Pindarian B.) Abraham C.) Arthur Holmes D.) Julfred
4.) Continental subduction, continental shield, continental rise, and deep ocean are all found
in what?
A.) Continental margins B.) Shield sea C.) Continent D.) The ocean
5.) What is an active volcanic field on the Pacific Ocean called?
A.) Ring of the sea B.) Ring of earth C.) Ring of fire D.) Ring of oceans
6.) Who explained sea plains spreading?
A.) Hess B.) Abraham C.) Henry D.) Thomas

32
7.) In 1967, who presented the concept of
plate tectonics?
A.) Newton B.) John d C) Mackenzie, Parker and Morgan D.) August
8.) What areLaurasia and Gondwanaland?
A.) Continental masses B.) Water C.) Soil D.) The ocean
9.) Gold found on which coast?
A.) Ghana coast B.) Himani C.) Continent D.) Tillite
10.) Where are gold bearing veins found?
A.) Iraq B.) Africa C.) America D.) Brazil

11. Assertion(A): Movement in tectonic plates are a natural phenomenon.


Reason (R): due to Polar Fleeing Force & Tidal Force by Alfred Wegner

A. Both are correct and reason explain the


assertion
B. Both are correct but reason not explain the assertion.
C. Only assertion is correct
D. Only reason is correct
12. Assertion (A): Volcanic activities took place more at the margin of Pacific plate.
Reason (R): It is due to presence of convergent boundary.
A. Both are correct and reason explain the assertion
B. Both are correct but reason not explain the assertion.
C. Only assertion is correct
D. Only reason is correct

ANSWERS
1. A.) Mesosaurus
2. B.) Polar fleeing force and tidal force
3. C.) Arthur Holmes
4. A.) Continental margins
5.c.) Ring of fire
6. A.) Hess
7. C) Mackenzie, Parker and Morgan
8. A.) Continental masses

33
9. A.) Ghana coast
10. D.) Brazil
11. B. Both are correct but reason not explain the assertion
12. A. Both are correct and reason explain the assertion

SOURCE BASED QUESTION

13) Observe the shape of the coastline of the Atlantic Ocean. You will be surprised by the
symmetry of the coastlines on either side of the ocean. No wonder, many scientists thought
of this similarity and considered the possibility of the two Americas, Europe and Africa, to be
once joined together. From the known records of the history of science, it was Abraham
Ortelius, a Dutch map maker, who first proposed such a possibility as early as 1596.
Antonio Pellegrini drew a map showing the three continents together. However, it was Alfred
Wegener—a German meteorologist who put forth a comprehensive argument in the form of
“the continental drift DISTRIBUTION OF OCEANS AND CONTINENTS CHAPTER theory”
in 1912. This was regarding the distribution of the oceans and the continents. According to
Wegener, all the continents formed a single continental mass and mega ocean surrounded
the same. The super continent was named PANGAEA, which meant all earth. The mega-
ocean was called PANTHALASSA, meaning all water. He argued that, around 200 million
years ago, the super continent, Pangaea, began to split. Pangaea first broke into two large
continental masses as Laurasia and Gondwanaland forming the northern and southern
components respectively. Subsequently, Laurasia and Gondwanaland continued to break
into various smaller continents that exist today. A variety of evidence was offered in support
of the continental drift. Some of these are given below.

Q13.1. What is Pangaea?


ANS. The super continent of the world was known as PANGAEA, which meant all earth.
Q13.2. Who first propounded the theory of continental drift and when?
ANS. Alfred Wegener—a German meteorologist who put forth a comprehensive argument
in the form of “the continental drift DISTRIBUTION OF OCEANS AND CONTINENTS” in
1912.
Q13.3. Around how many year ago Pangea, began to split and what was the name of two
mega splited part of Pangea?
ANS. around 200 million years ago, the super continent, Pangaea, began to split. Pangaea
first broke into two-part Laurasia and Gondwanaland.

34
14) Wegener suggested that the movement responsible for the drifting of the continents was
caused by pole-fleeing force and tidal force. The polar-fleeing force relates to the rotation of
the earth. You are aware of the fact that the earth is not a perfect sphere; it has a bulge at
the equator. This bulge is due to the rotation of the earth. The second force that was
suggested by Wegener — the tidal force — is due to the attraction of the moon and the sun
that develops tides in oceanic waters. Wegener believed that these forces would become
effective when applied over many million years. However, most of scholars considered
these forces to be totally inadequate.
Q14.1 Which forces are responsible for the drifting of the continent by Wegner?
Ans. Pole-fleeing force and Tidal force.
Q14.2 Why the bulge emerged at equator?
Ans. This bulge is due to the rotation of the earth.
Q14.3 Who considered these forces are totally inadequate for drifting of the continent?
Ans. Most of scholars considered.

SHORT ANSWER QUESTION

Q1. Explain the hypothesis, known as the “sea floor spreading” given by Hess.

Ans. Hess argued that constant eruptions at the crest of oceanic ridges cause the rupture of
the oceanic crust forces and the new lava wedges into it, pushing the oceanic crust on
either side. The ocean floor, thus spreads. Two facts made Hess think about the
consumption of the oceanic crust. The younger age of the oceanic crust.The spreading of
one ocean does not cause the shrinking of the other.He further maintained that the ocean
floor that gets pushed due to volcanic eruptions at the crest, sinks down at the oceanic
trenches and gets consumed

Q2. Explain different types of boundaries that form as a result of movement of tectonic
plates.

Ans. Convergent Boundaries: Where the crust is destroyed as one plate dived under
another, it is called convergent boundaries.

Divergent Boundaries: Where new crust is generated as the plates pull away from each
other, these are called divergent boundaries.

Transform Boundaries: Where the


crust is neither produced nor destroyed as the plates slide horizontally past each other.

Q3. The rate of plate movement varies considerably. Justify.

35
Ans. It is very rightly said. The strips of normal and reverse magnetic field that parallel the
mid-oceanic ridges help scientists determine the rates of plate movement. These rates vary
considerably.

Slowest Rate: The Arctic Ridge has the slowest rate. It is less than 2.5 cm/yr.

Fastest Rate: The East Pacific rise near Easter Island, in the South Pacific about 3,400 km
west of Chile, has the fastest rate. It is more than 15 cm/yr.

Q4. What is the basic difference between the drift theory and plate tectonics.

Ans: The basic difference between the drift theory and plate tectonics are given below:

(a) The continental drift theory was postulated by Alfred Wegener in 1912.

(b) Continental drift theory believed that all the present continents were united one. He
named the supercontinent Pangaea.

(c) Drift theory considers the horizontal movement only.

(d) Continental drift theory explained only the origin and distribution of ocean and
continents.

Plate tectonics:

(a) In 1967, Makenzie, Parker and Morgan forwarded the plate tectonics ideas.

(b) According to the plate tectonics theory, the upper part of the earth (lithosphere) is made
of some blocks untitled as plates.

(c)It considers the subduction of plates besides horizontal movement.

(d)Plate tectonics theory is a theory which is able to explain the origin and distribution of
Oceans and continents, earthquakes, volcanic activity, sea-floor spreading and fold
mountains on earth’s surface.

LONG ANSWER TYPE QUESTION


Q1. According to tectonic plates theory in how many plates has the earth been divided?
Explain.

Ans. The theory of plate tectonics proposes that the earth’s lithosphere is divided into seven
major and some minor plates. The major plates are as follows:

i. Antarctica and the surrounding oceanic plate

ii. North American plate

iii, South American plate

iv. Pacific plate

v. India-Australia-New Zealand plate

36
vi. Africa with the eastern Atlantic floor plate

vii. Eurasia and the adjacent oceanic plate.

Some important minor plates are:

i. Cocos plate: It is between Central America and Pacific plate

ii. Nazca plate: It is between South America and Pacific plate

iii. Arabian plate: It includes mostly the Saudi Arabian landmass

iv. Philippine plate: It is between the Asiatic and Pacific plate

v. Caroline plate: It is between the Philippine and Indian plate (North of New Guinea)

vi. Fuji plate: It includes North-east of Australia.

Pacific plate is largely an oceanic plate whereas the Eurasian plate may be called a
continental plate. Plates are not static. Plates may converge or diverge. Plates may break
as well.

Q2. – Explain tectonic plate theory and its working.

Ans. These plates have been constantly moving over the globe throughout the history of the
earth.The theory of plate tectonics was introduced by McKenzie, Parker and , Morgan in
1967.

A tectonic plate is also called as lithosphere plate. It is a massive irregularly shaped slab of
solid rock. Consists of oceanic and continental sphere. Plates move horizontally over the
asthenosphere. Average thickness is 100 km of oceanic part and 200 km of continental
part.It may be oceanic or continental.

Pacific plate is largest oceanic plate whereas Eurasian plate is the largest continental plate.
These plates are moving constantly throughout geological time not the continent, believed
by Wegener. It creates three types of boundaries.

1. Divergent boundaries:

i. New crust is generated

ii. Plates move away from each other

iii. These are called spreading sites

iv. For example, the Mid Atlantic Ridge

Convergent boundaries :

Crust is destroyed. Sinking of plate is called “subduction zone”. There are three ways in
which subduction occurs

(i) between an oceanic and continental plates;

37
(ii) between two oceanic plates; and

(iii) between two continental plates.

Transform boundaries: Where the crust is neither produced nor destroyed as the plates
slide horizontally past each other.

Q3.What is the evidence in support of the continental drift theory?

Ans: To support the continental drift theory, Wegener collected a lot of evidence, from
various sources Among them the following are worth mentioning.

(a) The matching of continents: The shorelines of Africa and South America facing each
other have a remarkable and unmistakable match. The matching situation or the jig-saw-fit
prove that once all the landmasses were united one.

(b) Rocks of the same age across the oceans:From the radiometric dating methods it is
clear that the rock formation of both coasts are the same age.

(c)Tillite: Tillite is the sedimentary rock formed out of deposits of glaciers. It is found in six
different landmasses of the southern hemisphere (Africa, Falkland Islands, Madagascar,
Antarctica, Australia and India). The glacial Tillite provides unambiguous evidence of
paleoclimates and also of drifting of continents.

(d)Placer Deposits: The occurrence of rich placer deposits of gold in the Ghana coast and
the Brazil plateau prove the continental drift theory.

(e) Distribution of fossils: The presence of Lemurs in three countries respectively India,
Madagascar and Africa, prove that once all the landmasses were united one.

(f) The Plate tectonics also support the continental drift theory of Alfred Wegener.

Q4. What were the major post- drift discoveries that rejuvenated the interest of scientists in
the study of distribution of Oceans and continents?

Ans: For Continental drift, most of the evidence was collected from the continental areas in
the form of distribution of flora and fauna or from other deposits. A number of discoveries
during the post- drift period added new Ocean and continents. Particularly the information
collected from the ocean floor mapping provided new dimensions for the study of
distribution of Oceans and continents.Some facts are given below:-

(a) Most of the volcanic eruptions are scattered all along the mid- oceanic ridges, which
bring out the lava to 6 surfaces in such areas.

(b) The presence of conventional current in the mantle, which helps to move the land blocks
from one place to another.

(c) Information about the sea floor spreading.

(d) The rocks of the oceanic crust are much younger than the continental one which is
another discovery of the post drift period.

38
MAP WORK

Maps items for locating and labelling on outline political World map.

A) Major oceans and continents of the world: -

OCEAN’S NAME CONTINENT’S NAME


a) Indian ocean
1) Asia
b) Pacific Ocean
2) Europe
c) Atlantic Ocean
3) North America
d) Arctic ocean
4) South America
e) Southern Ocean
5) Africa
6) Australia

7) Antarctica

39
B) Major lithospheric plates and Minor lithospheric plates, Ring of fire (Pacific Ocean), Mid-
Atlantic Ridge

C) MAJOR HOT SPOT OF THE WORLD.


a) Hawaii b) Galapagos c) Yellowstone

40
Chapter: 5
Geomorphic Processes

Section-A [MCQ Type]

Q1) The science of soil is known as…..


A) Geomorphology
B) Pedalogy
C) Ecology
D) Hydrology

Q2) Mountain building through folding is called as ……..


A) Denudational Process
B) Weathering
C) Orogenic Process
D) Exogenic Process

Q3) Which one of the following is not included in denudation?


A) Weathering
B) Mass Movement
C) Erosion
D) Deposition

Q4) Which one of the following is not a type of chemical weathering?


A) Oxidation
B) Carbonation
C) Hydration
D) Siltation

Q5) Which of the following is type of weathering?


A) Physical
B) Chemical
C) Biological
D) All of the above

Q6) Which of the following is the continental-building process?


A) Epeirogenic processes
B) Volcanism
C) Earthquakes
D) Orogenic processes

Q7)All the exogenic geomorphic processes are covered under a general term, what is it
called?
A) Denudation
41
B) Mass wasting
C) Erosion and deposition
D) Diastrophism

Q8) Flaking off of more or less curvedsheets of shells from over rocks or bedrockresults in
smooth and rounded surfaces. This process is called…..
A) Surface runoff
B) Exfoliation
C) Chemical Weathering
D) None of these

Q9) Which of the following geomorphic agents participate in mass movement?


A) Running water
B) Wind
C) Glacier
D) None of these

Q10) Free fall of earth debris from a vertical oroverhanging face is called …..
A) Land slide
B) Rock Fall
C) Debris Fall
D) Mud Flow

[Source Based Questions]

Q1) Read the given paragraph carefully and answer the questions that follow.

Weathering processes are responsible for breaking down the rocks into smaller fragments
and preparing the way for formation of not only regolith and soils, but also erosion and mass
movements. Biomes and bio-diversity is basically a result of forests(vegetation) and forests
depend upon the depth of weathering mantles. Erosion cannot be significant if the rocks are
not weathered. That means, weathering aids mass wasting, erosion and reduction of relief
and changes in landforms are a consequence of erosion. Weathering of rocks and deposits
helps in the enrichment and concentrations of certain valuable ores of iron, manganese,
aluminium, copper etc., which are of great importance for the national economy. Weathering
is an important process in the formation of soils.
a) Which processes are responsible for breaking down of rocks into smaller fragments?
b) Name any two valuable ores which are enriched by weathering processes?
c) What is not significant if rocks are not weathered?

Q2) In our country, debris avalanches and landslides occur very frequently in the
Himalayas. There are many reasons for this. One, the Himalayas are tectonically active.
They are mostly made up of sedimentary rocks and unconsolidated and semi-consolidated

42
deposits. The slopes are very steep. Compared to the Himalayas, the Nilgiris bordering
Tamil Nadu, Karnataka, Kerala and the Western Ghats along the west coast are relatively
tectonically stable and are mostly made up of very hard rocks; but, still, debris avalanches
and landslides occur though not as frequently as in the Himalayas, in these hills. Why?
Many slopes are steeper with almost vertical cliffs and escarpments in the Western Ghats
and Nilgiris. Mechanical weathering due to temperature changes and ranges is pronounced.
They receive heavy amounts of rainfall over short periods. So, there is almost direct rock fall
quite frequently in these places along with landslides and debris avalanches.

a) In which part of the country debris avalanches and landslides are very frequent?
b) Which mountains receive heavy amounts of rainfall over short periods?
c) Which mountains are relatively tectonically stable and are mostly made up of very hard
rocks?

[Long Answer Questions]

Q1) “Our earth is a playfield for two opposing groups of geomorphic processes.”Discuss.

Q2) Exogenic geomorphic processes derive their ultimate energy from the sun’s heat.
Explain.

Q3) Are physical and chemical weathering processes are independent of each other. If not,
why?

Q4) What is soil? Explain the factors of soil formation.

ANSWERS

Section-A [MCQ Type]

Q1) B) Pedalogy
Q2) C) Orogenic Process
Q3) D) Deposition
Q4) D) Siltation
Q5) D) All of the above
Q6) A) Epeirogenic processes
Q7) A) Denudation
Q8) B) Exfoliation
Q9) D) None of these

43
Q10) C) Debris Fall

[Source Based Questions]

Q1- a) Weathering processes


b) Iron, manganese, aluminium, copper [any two]
c) Erosion

Q2- a) Himalayas
b) Western Ghats and Nilgiris
c) Nilgiris

[Long Answer Questions]

Q1) -a) Yes. It is a correct statement that our earth is playfield for two opposing groups of
forces. These forces are exogenic and endogenic forces.
b) The external forces are known as exogenic forces and they include sunlight, rainfall,
snowfall, movement of wind. Whereas internal forces are called as endogenic forces. Heat
inside the earth, magma, earthquake and volcanic activities are part of endogenic forces.
c) Endogenic forces are continuously engaged in creation of new landform by making
change on the earth surface. Folding, faulting, uplift, subsidence differences on the earth
surface.
d) On the other hand, exogenic forces are always engaged in obliterating the differences by
the process of weathering and erosion. Higher land are wear down (degradation) and lower
land are filled up (aggradation).
e) At any point of time, whatever landforms are seen in a particular area, they are result of
both exogenic and endogenic forces.

Q2) –a) Exogenic forces are also called as external forces as they work on the earth from
outside. Sunlight, rainfall, snowfall, sea waves, movement of wind are continuously engaged
in denudational work like weathering and erosion.
b) The sun is the ultimate source of all energy on the earth. Every process and system get
energy from the sun to function. When sunlight fall on rocks thermal expansion occurs,
which lead to weathering of the rocks.
c) As far as erosion by river is concerned, they also get energy from the sun. Sunlight
evaporate sea water which fall as rain on the land. That rain in the form river create various
landform like V shaped valley, water fall, canyon, potholes in the mountain areas.
d) Higher temperature create low air pressure. Difference in pressure force wind to move.
Moving wind also create variety of landform like sand dunes, mushroom rock, deflation
basin, loess plain in the desert area.

44
Q3) a) No, physical and chemical weathering are not independent of each other. They are
different but still interdependent.
b) Physical or mechanical weathering processes depend on gravitational forces such as
overburden pressure, load and shearing stress; expansion forces due to temperature
changes, crystal growth or animal activity; water pressures controlled by wetting and drying
cycles.
c) Chemical weathering depends on a group of weathering processes viz; solution,
carbonation, hydration, oxidation and reduction act on the rocks to decompose, dissolve or
reduce them to a fine clastic state
d) Water and air along with heat must be present to speed up all chemical reactions. Over
and above the carbon dioxide present in the air, decomposition of plants and animals
increases the quantity of carbon dioxide underground. These chemical reactions on various
minerals are very much similar to the chemical reactions in a laboratory.
e) Therefore, it is clear that both these weathering go hand with hand and both of them
complementary to each other.

Q4) Top thin layer of the earth surface having loose, unconsolidated materials is called soil.
It has air, water, minerals to support of growth of plant. Soil is formed by weathering and
erosion. It is slow process. Formation of soil depends of many factors.
a) Parent Materials: It controls the type and nature of weathering and erosion.
Characteristics of parent materials is transferred to soil. For example, a rock rich in iron will
produce iron-rich soil. Hard rocks will take long time to become soil. Whereas soft rock can
be eroded very quickly.
b) Climate: Temperature and rainfall also affect soil formation. In humid region rocks are
eroded by running water, whereas in dry region rocks are eroded by wind. Thermal
expansion and cooling of rock leads to physical weathering. It is purely dependent on
temperature.
c) Topography: Soil layer will be thin on steep slopes as most of the materials will be
washed away by running water. Lower valley will have thick layer of soil due to deposition of
sediments. Sun facing slope will have different soil than opposite slope.
d) Biological Activities: They add organic matter, water and nitrogen. Various living creature
also engaged in biological weathering. Dead remains of plant become humus and soil
become more fertile.
e) Time: Soil formation is a slow process. To make a mature soil profile, it may take
hundreds of years. Younger soil do have soil horizon but older soils have clear soil horizons.

45
Chapter: 6
Landforms & their Evolution
• Several related landforms together make up landscapes. Landforms can be defined
as features on the Earth's surface that make up the terrain, such as mountains,
valleys, plains or plateau.
• Landforms once formed may change in their shape, size and nature slowly or fast
due to continued action of geomorphic processes and agents.
• Different agents of geomorphic processes are running water, wind, glacier, ground
water, sea waves.
• All agents of geomorphic processes are engaged in three activities:
a. Erosion b. Transportation c. Deposition
• Due to these activities they form erosional landforms and depositional landforms.
Agent of Geomorphic Erosional Landforms Depositional Landforms
Processes
Running Water V-Shaped Valley, Gorge, Alluvial fans, Flood Plains,
Canyon, Waterfall Natural Levees, Delta
Ground Water Pools, Sinkholes, Lapies, Stalactite, Stalagmite, Pillars
Limestone Pavements
Glacier Cirque. Horn, U-Shaped Valley, Glacial till, Moraines,
outwash plains
Wind Pediments and Pediplains, Sand dunes, Loess
Playas, Mushroom Rock

MCQs:
(i) In which of the following stages of landform development, downward cutting is
dominated?
(a) Youth stage (c) Early mature stage
(b) Late mature stage (d) Old stage
(ii) A deep valley characterised by steep step-like side slopes is known as
(a) U-shaped valley (c) Blind valley
(b) Gorge (d) Canyon

46
(iii) A deep, long and wide trough or basin with very steep concave high walls at its head as
well as in sides is known as:
(a) Cirque (c) Lateral Moraine
(b) Glacial valley (d) Esker

(iv) A land form passes through how many stages of development?

(a) youth (b) mature

(c) old age (d) all of them

(v) Running water / River causes

(a) erosion (b) deposition

(c) both of them (d) none of them

(vi) Overland flow causes__________

(a) sheet erosion (b) hill formation

(c) sheet deposition (d) none of these

(vii) Name the forces which are responsible for the evolution of landforms.

(a) external (b) Internal

(c) both (d) none of these

(viii) Which type of valley is formed by the vertical erosion of the river?

(a) V-shaped (b) canyon

(c) U-shaped (d) sink valley

(ix) What do we call the steep slope facing the sea formed due to erosion ?

(a) canyon (b) coast

(c) mountain (d) cliff

(x) Which one of the following is an erosional landform?


(a) valleys (b) floodplains
(c) deltas (d) alluvial fans

47
(Xi) Which one of the following features is a type of channel pattern?
(a) Floodplains (b) Deltas
(c) Meander (d) Alluvial Fans

Question & Answer


Q. How are the river valleys formed?
Ans: Small and narrow rills gradually develop into long and wide gullies. The gullies further
deepen, widen and lengthen to give rise to valleys.
Depending upon dimensions and shape, many types of valleys are formed.
(a) V-shaped valley,
(b) gorge,
(c) canyon,
In the upper course a river vertical erosion is dominant than lateral erosion. Hence, valley
deepens and form V-shaped valley.
A gorge is a deep valley with very steep to straight sides. A gorge is almost equal in width at
its top as well as its bottom.
A canyon is characterised by steep step-like side slopes and may be as deep as a gorge.
Q. Explain any three depositional landforms formed by river.
Ans: River forms various landforms due to its depositional action.
Alluvial Fans:
Alluvial fans are formed when streams flowing from higher levels break into foot slope plains
of low gradient. Normally very coarse load is carried by streams flowing over mountain
slopes. This load becomes too heavy for the streams to be carried over gentler gradients
and gets dumped and spread as a broad low to high cone shaped deposit called alluvial fan.
Floodplains: Floodplain is a major landform of river deposition. A flood plain is an area of
flat land alongside a river. This area gets covered in water when the river floods. Flood
plains are naturally very fertile due to the river sediment which is deposited there. Flood
plains are intensively cultivated because of its fertility.
Fine sized materials like sand, silt and clay are carried by relatively slow moving waters in
gentler channels usually found in the plains and deposited over the bed and when the
waters spill over the banks during flooding above the bed.
Delta: It is a triangular alluvial tract formed by the river at its end point. River deposits the
fine materials like silt and clay. Here river divides into numerous channels called as
distributaries before flowing into sea..
Q. Describe any three erosional landforms formed by a glacier.

48
Ans: Glacier is a moving mass of ice. It forms various landforms. Some erosional landforms

are:

Cirque:
The cirques quite often are found at the heads of glacial valleys. It is an amphitheatre-like
valley formed by glacial erosion. The accumulated ice cuts these cirques while moving
down the mountain tops.
A lake of water can be seen quite often within the cirques after the glacier disappears. Such
lakes are called cirque or tarn lakes.
Horn: If three or more radiating glaciers cut headward until their cirques meet, high, sharp
pointed and steep sided peaks called horns form.
Q. What are moraines? What are the types of moraines?
Ans: Moraines are sediments deposited by glacier. These are mostly soil and rocks.
Moraines are of different types:
a. Terminal moraines or end moraines are long ridges of debris deposited at the end
(toe) of the glaciers.
b. Lateral moraines form along the sides parallel to the glacial valleys. The lateral
moraines may join a terminal moraine forming a horse-shoe shaped ridge.
c. A medial moraine is found on top of and inside an existing glacier. Medial moraines
are formed when two glaciers meet and two lateral moraines join.
d. Many valley glaciers retreating rapidly leave an irregular sheet of till over their valley
floors. Such deposits varying greatly in thickness and in surface topography are
called ground moraines.
Q. What are sand dunes? What are its types?
Ans: Hills of sand is called sand dunes. When obstructions come across the winds in the
desert, winds get deposited and form sand dunes. On the basis of formation and shapes,
sand dunes can be of different types:

49
a. Barchans:

Crescent shaped dunes called barchans with the points or wings directed away from
wind direction.
b. Parabolic dunes:
Form when sandy surfaces are partially covered with vegetation. That means
parabolic dunes are reversed barchans with wind direction being the same.
c. Seif:
This is similar to barchan with a small difference. Seif has only one wing or point.
This happens when there is shift in wind conditions.
d. Longitudinal dunes :
These dunes are formed when supply of sand is poor and wind direction is constant.
They appear as long ridges of considerable length but low in height.
e. Transverse dunes :
These are aligned perpendicular to wind direction. These dunes form when the wind
direction is constant and the source of sand is an elongated feature at right angles to the
wind direction.

DIAGRAM BASED QUESTIONS


Q. Study the diagram carefully and answer the questions that follow:

50
1. Identify the above course of a river.
2. Identify and name the feature marked A?
3. How do the features A formed?
Q. Study the diagram showing features formed by ground water and answer the questions
that follow:

a. Identify and name the feature marked as A . How is it formed?


b. Identify and name the feature marked as C . How is it formed?
c. Identify and name the feature marked as B . How is it formed?

Source Based Questions


Q. Read the paragraph carefully and answer the questions that follow:
They are long ridges of deposits of glacial till. Terminal moraines are long ridges of
debris deposited at the end (toe) of the glaciers. Lateral moraines form along the sides
parallel to the glacial valleys. The lateral moraines may join a terminal moraine forming a
horse-shoe shaped ridge. There can be many lateral moraines on either side in a glacial
valley. These moraines partly or fully owe their origin to glacio-fluvial waters pushing up
materials to the sides of glaciers. Many valley glaciers retreating rapidly leave an irregular
sheet of till over their valley floors. Such deposits varying greatly in thickness and in surface
topography are called ground moraines. The moraine in the centre of the glacial valley
flanked by lateral moraines is called medial moraine. They are imperfectly formed as
compared to lateral moraines. Sometimes medial moraines are indistinguishable from
ground moraines.
a. What are moraines?
51
1. Sediments deposited by glaciers 2. Sediments eroded by the glaciers
3. river of water 4. Sediments transported by glaciers
b. Lateral moraines are formed due to:
1. sediments deposited at the centre 2. Deposited at the centre
3. glacio-fluvial waters pushing up materials to the sides of glaciers
4. deposited of moraines at the centre
c. How are ground moraines formed?
1. Moraines deposited in the centre
2. an irregular sheet of till deposited over their valley floors.
3. deposited on either side of the valley
4. deposited in horse shoe shape

52
Chapter:7
Composition and Structure of Atmosphere

QUESTIONS

Section-A [MCQ Type]

Q1) Highest concentration of ozone gas is found in which layer of the atmosphere?
A) Troposphere
B) Stratosphere
C) Ionosphere
D) Mesosphere

Q2) 99% mass of the atmosphere is confined up to the height of …………


A) 12 km
B) 32 km
C) 50 km
D) 80 km

Q3) Which layer of atmosphere is found up between height of 50-80 km?


A) Troposphere
B) Ozone Layer
C) Mesosphere
D) Ionosphere

Q4) Which gas absorb harmful ultra-violet radiation coming from the sun and protect us on
the earth?
A) Hydrogen Gas
B) Ozone Gas
C) Oxygen Gas
D) Nitrogen Gas

Q5) Temperature decreases with height at the rate of 10 C per 165 m. In which layer this
phenomena occurs?
A) Troposphere
B) Stratosphere
C) Mesosphere
D) Ionosphere

Q6) Sea salt, pollen, ash, smoke soot, fine soil — these are associated with:
A) Gases
B) Water Vapour
C) Dust particles
D) Meteors

53
Q7) Ozone gas protect us from one harmful radiation of the sun. Name that radiation.
A) Infra-red
B) Ultra Violet
C) Gamma Ray
D) X-Ray

Section-B [Source Based Questions]

Q1) Read the given paragraph carefully and answer the questions that follow.

The troposphere is the lowermost layer of the atmosphere. Its average height is 13 kmand
extends roughly to a height of 8 km near the poles and about 18 km at the
equator.Thickness of the troposphere is greatest at the equator because heat is transported
togreat heights by strong convectional currents. This layer contains dust particles and
watervapour. All changes in climate and weather take place in this layer. The temperature
inthis layer decreases at the rate of 1°C for every 165m of height. This is the most
importantlayer for all biological activity. The zone separating the troposphere from
stratosphere is known as the tropopause. The air temperature at the tropopause is about
minus 800C over the equator and about minus450C over the poles. The temperature here is
nearly constant, and hence, it is called thetropopause.

a) What is the height of troposphere at the equator?


b) At what rate temperature decreases in the troposphere?
c) What is the air temperature at the tropopause over the poles?

Q2) Study the given diagram and answer the questions that follow.

a) Name the third layer of atmosphere from the earth surface.


b) What is the thickness of Stratosphere?

54
c) What is the temperature at Stratopause?

Section-C [Long Answer Questions]

Q1) Which layer of the atmosphere is the most important for human being and why?

Q2) Explain the composition of the atmosphere.

Q3) Describe the various layers (structure) of the atmosphere.

ANSWERS

Section-A [MCQ Type]

Q1) B) Stratosphere
Q2) B) 32 km
Q3) C) Mesosphere
Q4) B) Ozone Gas
Q5) A) Troposphere
Q6) C) Dust particles
Q7) B) Ultra violet

Section-B [Source Based Questions]

Q1- a) 18 km
b) Decreases at the rate of 1°C for every 165m
c) Minus 800C

Q2- a) Mesosphere
b) About 40 km.
c) Near 00C

Section-C [Long Answer Questions]

Q1-a) Troposphere is the most important layer of human beings. It is equally important for
other living beings on the earth.
b) Troposphere is the lower most layer and immediately in contact with the earth surface. Its
average height is about 13 km.
c) Troposphere has maximum concentration of gases, dust particles and water vapour.
However, amount decreases with height.
d) Oxygen need by humans, carbon dioxide needed by plants have maximum concentration
in the troposphere.

55
e) Most of the weather / climate related phenomena take place in the troposphere only like
lightening, thunder, hail, storm, rainfall, snowfall etc.

Q2) The gaseous cover around the earth is called atmosphere. Our atmosphere is mainly
composed of three things.
a) Gases: Important gases found in out atmosphere are Nitrogen [72%], Oxygen [21%],
CO2, Helium, Ozone, Hydrogen etc. Oxygen is very important for human beings. Similarly,
Carbon Dioxide is taken by plants and trees. Ozone gas filters ultraviolet rays of the sun and
protects life on the earth.
b)Water Vapour: Water is also present in the atmosphere, but in gaseous form – water
vapour. Rainfall, snowfall, dew, frost, hail etc are result of condensation of water. Water in
the atmosphere comes mainly by evaporation of sea water. Most of the water is
concentrated only in the lower part of the atmosphere.
c)Dust Particles: Dust particles include solid particles, ash, pollen, smoke etc. They play
very important role in the atmosphere. These dust particles scatter the sunlight and adds
charming colour – red and orange – during sunrise and sunset. Dust particle become nuclei
for the rain drop and help formation of clouds.

Q3) Atmosphere is extended up to thousand of kilometres in the sky. It has five distinct
layers. From the earth surface, these fiver layers are: a) Troposphere, b) Stratosphere, c)
Mesosphere, d) Ionosphere and e) Exosphere.
a) Troposphere is the lower most layers. It is also the most important layer because highest
concentration of gases, water vapour is found in this layer. All the weather phenomena like
rainfall, snowfall, hail, storm, lightening etc. take place in this layer only. This layer is
extended up to 18 km. from the earth surface.
b) Stratosphere is the second layer which is extended up to 50 km from the earth surface.
Ozone gas is found in this layer which filters harmful ultraviolet radiation coming from the
sun and protects life on the earth.
c) Mesosphere: It is the middle layer. It extends from 50 km to 80 km in the sky.
d) Ionosphere: Ionosphere is very important for communication as it transmit radio waves
back to the earth surface; therefore this layer is used for radio communication.
e) Exosphere: Exosphere is the last layer about which very little is known. Only light and
rare gases are found in these layers. This layer starts from a height of 400 km. from the
earth surface.

56
CHAPTER - 8
SOLAR RADIATION, HEAT BALANCE AND TEMPERATURE

IMPORTANT FACT OF THIS CHAPTER:

The energy that the earth receives is known as insolation.


The Concept of Aphelion and Perihelion – The earth's position in relation to the sun
changes throughout the year. On July 4th, the earth is furthest from the sun, at a distance of
152 million km. This position is referred to as aphelion. When, the earth is closest to the sun
on January 3rd, at a distance of 147 million km. This position is known as perihelion.

Factors responsible for variations in Insolation at the Earth's Surface:

i. The earth's rotation on its axis.


ii. The angle of the sun's rays.
iii. The length of the day.
iv. The transparency of the atmosphere.
v. The configuration of the land in relation to its aspect.

Terrestrial Radiation and the Heating and Cooling of the Atmosphere

Terrestrial Radiation – The earth receives solar radiation in short wave forms which heats
up its surface. The earth then acts as a radiating body, emitting energy in the form of long
waves to the atmosphere. This process is known as terrestrial radiation. These long wave
radiations heat up the atmosphere from below. The atmosphere in turn radiates and
transmits heat to space. This process helps maintain a constant temperature at the earth’s
surface, as the amount of heat received from the sun is balanced by the amount transmitted
to space.

Heating and Cooling of the Atmosphere:

Conduction - The terrestrial radiation heats up the lower atmosphere, which is directly in
contact with the surface of the earth. This process is called conduction and involves the
transfer of energy from a warmer body to a cooler one until both reach the same
temperature.

Convection - As the lower layer of the atmosphere heats up, it rises vertically in the form of
currents, transmitting the heat of the atmosphere. This vertical heating of the atmosphere is
called convection and is restricted only to the troposphere.

Advection - the transfer of heat through the horizontal movement of air. In India, the local
winds known as 'loo' during summer are a result of the advection process. Advection is
more significant than convection. In middle latitudes, most diurnal (day and night) changes
are due to advection alone.

57
Factors Influencing Temperature Distribution

Several factors influence the temperature at any given location:

Latitude of the Place – The temperature of a place is largely determined by the amount of
solar radiation it receives, which varies according to latitude. Solar radiations pass vertically
along the equator, but the angle of incidence decreases from the equator towards the poles.
As a result, the area heated by the solar radiation increases towards the poles, leading to a
decrease in temperature from the equator to the poles.

Altitude of the Place – The terrestrial radiation heats up the atmosphere from below.
Therefore, places near sea level have a higher temperature than places at higher altitudes.
Temperature generally decreases with an increase in height, a phenomenon known as the
"normal lapse rate" or "vertical temperature gradient". The temperature decreases at a rate
of 6.5°C per km of ascent.

Distance from the Sea – The proximity of a place to the sea also influences its
temperature. The temperature variation over the sea is less compared to that over land.
This is because land heats up and cools down quickly, while the sea heats up slowly and
loses heat slowly. Places near the sea are influenced by sea and land breezes, which
moderate the temperature.

Air Mass and Ocean Currents – Places that are influenced by warm air masses
experience higher temperatures, while those influenced by cold air masses experience
lower temperatures. Similarly, places located on the coast where warm ocean currents flow
have higher temperatures than places on the coast where cold currents flow.

Inversion of Temperature

i. The inversion of temperature refers to a condition where the temperature increases with
altitude.
ii. Conditions Leading to Temperature Inversion

Long Winter Nights – During long winter nights, the heat of the day is radiated off, causing
the earth to be cooler than the air above by early morning. Temperature inversion is a
common occurrence in polar regions.

Clear Sky – A clouded sky acts as a shield, preventing the escape of terrestrial radiation
from the ground. A clear sky allows more terrestrial radiation to escape, resulting in lower
temperatures at lower levels.

Dry Air Near the Ground – Dry air absorbs less terrestrial radiation, allowing more
radiation to escape.

Calm and Stable Air – Calm and stable air allows warm air to rise smoothly. Turbulence,
on the other hand, prevents the inversion of temperature as it leads to the mixing of warm
and cool air.

58
Impacts of Temperature Inversion

An inversion acts as a barrier to the upward movement of air from the layers below. As a
result, it hampers the diffusion of smoke, dust, and other pollutants. These particles
accumulate beneath the inversion layer and spread horizontally to fill the lower strata of the
atmosphere. The inversion layer inhibits the growth of clouds to a height that would produce
rain. Visibility is greatly reduced due to the accumulation of smoke and dust particles below
the inversion layer. Dense morning fogs are a common occurrence, especially during winter.

MCQ

1. The process of heating up of land through horizontal movement of heat is called:


A. Conduction
B. Convection
C. Advection
D. Air drainage

2. With increase in height the temperature decreases at a normal rate it is called:


A. Air drainage
B. Earth radiation
C. Normal lapse rate
D. Inversion of temperature

3. Being heavy and dense, the cold air acts almost like water and moves down the slope to
pile up deeply in pockets and Valley bottoms with warm air above. This is called what?
A. Earth radiation
B. Normal lapse rate
C. Inversion of temperature
D. Air drainage

4. What is normal lapse rate at 1000 metre?


A. 1 degree Celsius
B. 2 degree Celsius
C. 6 degree Celsius
D. 6.5 degree Celsius

5. Which of the following has longest day and night?


A. Equator
B. Poles
C. Tropic of Cancer
D. Tropic of Capricorn

6. Which of the following causes inversion of temperature in mountainous region?


A. Due to water vapours
B. Due to gravitation
C. Due to dust particle

59
D. Due to air drainage

7. On 4th July, the earth is farthest from sun. What is its distance?
A. 148 million km
B. 150 million km
C. 151 million km
D. 152 million km

8. In which continent highest temperature range occurs?


A. Europe
B. Africa
C. Asia
D. Australia

9. Assertion(A): atmosphere become warm from bottom to top.


Reason (R): it is due to heavier gases lies in the top of the atmosphere.
A. Both are correct and reason explain the assertion
B. Both are correct but reason not explains the assertion.
C. Only assertion is correct
D. Only reason is correct
10. Assertion (A): In desert, terrestrial radiation took place more.
Reason (R): It is due to presence of clear sky.
A. Both are correct and reason explain the assertion
B. Both are correct but reason not explains the assertion.
C. Only assertion is correct
D. Only reason is correct

Source Based Questions

1. Read the following paragraph and answer the questions given below:
Sun is the most important source of atmospheric heat. Sun’s diameter is more than 100
times the earth’s diameter and has surface temperature of more than 10,000° F. The heat
waves radiate from the sun in all directions at a speed of 1,86,000 miles per second or 3
lakh km per second. The solar radiation received by the earth is 1/2000 million part of solar
output. Yet, this small amount is vital for our existence on the earth. It is estimated that, on
an average, the earth receives about 1.9 gm. calories per minute per sq. cm. This is known
as ‘solar constant’. Thus, insolation means ‘the incoming solar radiation received on the
earth surface’. The word insolation is a combination of three words (in + sol + action) or
“Incoming solar radiation.”

(i) Which of the following is an important source of atmospheric heat?


(a) Moon
(b) Sun
(c) Earth

60
(d) Jupiter
Answer: -(b) Sun

(ii) What does insolation mean?


(a) Incoming solar radiation
(b) Outgoing solar radiation
(c) Reflecting solar waves
(d) None of these
Answer: -(a) Incoming solar radiation

(iii) How many words are combined to form the word insolation?
(a) 2
(b) 3
(c) 4
(d) 5
Answer: - (b) 3

(iv) How many calories does the earth receive per minute per sq km?
(a) 1.5 gm
(b) 1.8 gm
(c) 1.9 gm
(d) 1.7 gm
Answer: -(c) 1.9 gm

2. Read the following paragraph and answer the given questions:


The early Greek philosophers divided the globe into three regions on the basis of
temperature and latitudes. These are known as temperature zones. Torrid zone lies
between the Tropic of Cancer 231/2°N and the Tropic of Capricorn 23!4°S. The sun remains
overhead in this zone and the temperatures are high throughout the year. It has tropical
climate.

Temperate zone lies between 231/2° to 6614° latitudes in both the hemispheres. It has
moderate climate. In the northern hemisphere, it lies between Tropic of Cancer and Arctic
Circle. Frigid zone lies between 6614° N and North Pole in the northern hemisphere and in
the southern hemisphere, it lies between 6614° S and South Pole. It is the polar region and
extremely cold throughout the year.

(i) Where is tropical climate found?


(a) Torrid zone
(b) Temperate zone
(c) Tropical zone
(d) Frigid zone
Answer: -(a) Torrid zone

(ii) Which zone has moderate temperature?


(a) Torrid zone
(b) Temperate zone

61
(c) Tropical zone
(d) Frigid zone
Answer: -(b) Temperate zone

(iii) Which of the following zone lies between 6614° N and North Pole?
(a) Torrid zone
(b) Temperate zone
(c) Tropical zone
(d) Frigid zone
Answer: -(d) Frigid zone

(iv) In which zone sun remains overhead?


(a) Temperate
(b) Tropical
(c) Torrid
(d) Frigid
Answer: -(c) Torrid

Short answer questions (3 marks each)


1. What are the factors that control temperature distribution on the surface of the earth?
Ans. The factors controlling the temperature distribution on the surface of the earth are as
follows:
1. The latitude of the place
2. Altitude of the place
3. Distance from the sea
4. Ocean current
5. Others local aspect

2. What are the factors responsible for variability of insolation of the earth's surface?
Ans. Amount of insulation varies on the earth's surface due to the following reasons.
1. Rotation of the earth on its axis
2. Angle of inclination of Sun rays
3. Length of the day
4. Transparency of the atmosphere
5. Configuration of land in terms of its aspects

3. Why does inversion of temperature took place in some part of the world?
Ans. Inversion of temperature took place in the world due to the following factors.
1. Long Nights
2. Absence of cloud
3.No wind
4.Snow cover on earth's surface

4.Why is the annual range of temperature high in the Siberian plains?


Answer:-Siberian plains are quite far off from sea. Uneven climate is found in areas located
away from oceans and seas. The mean monthly temperature for January is between -18° C

62
to -48° C in the Siberian plains. In summers, it is up to 20°celcius. Therefore, the annual
range of temperature is -68 (-48-20) which is extremely high. The pressure of warm ocean
currents, Gulf Stream and north Atlantic drift, make the northern Atlantic Ocean warmer and
the isotherms bend towards the north. Over the land, the temperature decreases sharply
and the isotherms bend towards south in Europe.

LONG ANSWER TYPE QUESTION


Q.1. Discuss the processes through which the earth-atmosphere system maintains heat
balance.
Answer:
1. Conduction:

• The earth after being heated by insolation transmits the heat to the atmospheric layers
near to the earth in long wave form. The air in contact with the land gets heated slowly
and the upper layers in contact with the lower layers also get heated.
• It takes place when two bodies of unequal temperature are in contact with one
another, there is a flow of energy from the warmer to cooler body. The transfer of heat
continues until both the bodies attain the same temperature or the contact is broken.
Conduction is important in heating the lower layers of the atmosphere.

2. Convection:

• The air in contact with the earth rises vertically on heating in the form of currents and
further transmits the heat
of the atmosphere. This vertical heating of atmosphere is known as convection.
• The convection transfer of energy is confined only to the troposphere.

3. Advection:

• The transfer of heat through horizontal movement of air is called advection. Horizontal
movement of the air is relatively more important than the vertical movement.
• In tropical regions particularly in northern India during summer season local winds
called ‘loo’ is the outcome of advection process.

Q2. Explain about heat budget of the earth.


Answer:-The earth receives almost all of its energy from the sun. The earth in turn radiates
back to space the energy received from the sun. As a result, the earth neither warms up nor
does it get cooled over a period of time. Thus, the amount of heat received by different parts
of the earth is not the same. This variation causes pressure differences in the atmosphere.
This leads to transfer of heat from one region to the other by winds. The insolation received
at the top of the atmosphere is 100 per cent. While passing through the atmosphere some
amount of energy is reflected, scattered and absorbed. Only the remaining part reaches the
earth surface.

63
Roughly 35 units are reflected back to space even before reaching the earth’s surface. Of
these, 27 units are reflected back from the top of the clouds and 2 units from the snow and
ice- covered areas of the earth. The reflected amount of radiation is called the albedo of the
earth. The remaining 65 units are absorbed, 14 units within the atmosphere and 51 units by
the earth’s surface. The earth radiates back 51 units in the form of terrestrial radiation. Of
these, 17 units are radiated to space directly and the remaining 34 units are absorbed by
the atmosphere (6 units absorbed directly by the atmosphere, 9 units through convection
and turbulence and 19 units through latent heat of condensation). 48 units absorbed by the
atmosphere (14 units from insolation +34 emits from terrestrial radiation) are radiated back
into space.

Q3. Describe the major factors which control the distribution of insolation on the earth.
Answer: - The distribution of insolation varies from latitude to latitude. Different parallels
receive different amount of insolation. The following are the major factors which control the
distribution of insolation:
1. Intensity of insolation or angle of incidence of sun’s rays. The intensity of insolation
depends upon the angle of incidence of sun’s rays. Due to the spherical shape and
inclination of the axis of the earth, the angle of sun’s rays varies from latitude to latitude.
The sun shines vertically at the equator but the angle of sun’s rays decreases towards
poles.

The vertical rays deliver more energy per unit area than oblique rays. Vertical rays heat up a
smaller surface area and hence the heat received per unit area is large. Vertical rays travel
in shorter distance through the atmosphere. The loss of heat through atmosphere is small.
On the other hand the oblique rays are spread over a large area and the heat per unit area
is much less. The oblique rays have to pass a greater distance through the atmosphere and
it results in a large amount of loss of heat.

Angle of Incidence of sun’s rays Length of rays in atmosphere Intensity of insolation


90° 1.00 78%
60° 1.15 65%
30° 2.00 31%
0° 45.00 0%

2. Duration of solar radiation of length of day. Due to the inclination of axis and the
revolution of the earth, the length of day varies from place to place. The days are longer in
summer and shorter in winter, as we go away from the equator. The longer the sun shines,
the greater the amount of the insolation received. The two factors operate jointly. The length
of the day and the angle of the sun’s rays are equal on all parts of the same parallel.
Therefore, all places on the same parallel receive the same amount of the insolation.

3. Effect of atmosphere. The atmosphere weakens the insolation by the process of


scattering, reflection and absorption. Dust particles, water vapours and gases reduce the
amount of insolation. The oblique rays have to travel a greater distance through the
atmosphere and it results in the larger amount of loss of heat.

64
4. The distance between the earth and the sun. The distance between the sun and the earth
varies at several positions in its orbit. In January (Perihelion), the earth is only 165 million
kms. away from the sun. In July (Aphelion), the earth is 170 million kms. away from the
earth. More insolation is received by the earth when it is closer to the sun.

5. Number of Sun Spots. Number of sun spots increases and decreases with a period, with
the result the solar output changes from time to time.

6. Distribution of Land and Water. Land and water do not receive equal amount of
insolation. Land becomes quickly heated and cooled, while the water is slowly heated or
cooled. Therefore, the amount of the insolation received on continents and oceans varies.

MAP BASED QUESTIONS

Fig.1 – The distribution of surface air temperature in the month of July

Q1. What is Thermal equator?

Ans. The zone of highest mean temperature over the earth is called thermal equator.

Q2. Why thermal equator shifted towards the northern hemisphere?

Ans. The tilt of the earth’s axis and the orbit of the earth around the sun cause to move
north in the month of July.

Q3. Define isotherm.

Ans. The line joining places of equal temperature is called isotherm.

65
Chapter:9
Atmospheric Circulation
Q. 1 Multiple choice questions.
(i) If the surface air pressure is 1,000 mb, the air pressure at 1 km above the surface will be:
(a) 700 mb (b) 1,100 mb (c) 900 mb (d) 1,300 mb

► (c) 900 mb

(ii) The Inter Tropical Convergence Zone normally occurs:


(a) Near the Equator (b) Near the Tropic of Cancer
(c) Near the Tropic of Capricorn (d) Near the Arctic Circle

► (a) Near the Equator

(iii) The direction of wind around a low pressure in northern hemisphere is:
(a) Clockwise (b) Perpendicular to isobars
(c) Anti-clock wise (d) Parallel to isobars

► (c) Anti-clock wise

(iv) Which one of the following is the source region for the formation of air masses?
(a) The Equatorial forest (b) The Himalayas
(c) The Siberian Plain (d) The Deccan Plateau

► (c) The Siberian Plain

Q. 2 Answer the following briefly in not more than 30 words.


(i) What is the unit used in measuring pressure? Why is the pressure measured at station level reduced to
the sea level in preparation of weather maps?

❖Answer: -
Millibar or Pascal is the unit used in measuring pressure. It is reduced to the sea level in the preparation of
weather maps as the gravity of air at the surface is denser and hence has higher pressure.

(ii) While the pressure gradient force is from north to south, i.e. from the subtropical high pressure to the
equator in the northern hemisphere, why are the winds north easterlies in the tropics?

❖Answer: -
The Coriolis force acts perpendicular to the pressure gradient force. The pressure gradient force is
perpendicular to an isobar. The higher the pressure gradient force, the more is the velocity of the wind and
the larger is the deflection in the direction of wind. As a result of these two forces operating perpendicular
to each other, in the low-pressure areas the wind blows around it. The winds blow from high pressure to
the low pressure, so this pressure gradient force is from north to south. Therefore the north easterlies blow
from north east to south west.

66
(iii) What are the geostrophic winds?
❖ Answer: - Already Done.

(iv) Explain the land and sea breezes.


❖ Answer: - Already Done.

➢Page No: 93
Q. 3. Write the answers of the following questions in 120 words.
(i) Discuss the factors affecting the speed and direction of wind.
❖ Answer: - Already Done.

(ii) Draw a simplified diagram to show the general circulation of the atmosphere over the globe. What are
the possible reasons for the formation of subtropical high pressure over 30°N and S latitudes?
❖Answer: -Already Done.

(iii) Why does tropical cyclone originate over the seas? In which part of the tropical cyclone do torrential
rains and high velocity winds blow and why?
❖ Answer: -Already Done.

Multiple choice questions (MCQs):


1. Which of the following is a type of front?
a. Cold front
b. B. warm front
c. Stationary front
d. all of these

Ans. d. all of these

2. Atmospheric pressure determines


a. when earthquake occurs
b. when volcanic eruption takes place
c. when air will rise or sink
d. when climate will change

Ans c. when air will rise or sink

3. At sea level the average atmospheric pressure is


a. 1013.2mb
b. 100.2mb
c. 1021.4mb
d. 1006.7mb
Ans. a. 1013.2mb

67
4. Which of the following are the lines connecting places having equal pressure?
a. Isobars
b. isoclinic
c. Isoneph
d. Isohyet
Ans. a. isobars
5. Which of the following is not a type of cell?
a. Hadley cell
b. Ferrel cell
c. Polar cell
d. Deadly cell
Ans. d. deadly cell
6. Coriolis force is the result of
a. Gravitational pull of the sun
b. Gravitational pull of the moon
c. Rotation of the earth
d. Revolution period of the earth
Ans. c. rotation of the earth
7. --------expands when heated and gets compressed when cooled
a. Air
b. water
c. clouds
d. None of these
Ans. a. air
8. The -------rising of moist air cools it down to from the clouds and bring precipitation.
a. Horizontal
b. Vertical
c. Zigzag
d. Diagonal
Ans b. vertical
9. The high pressure area is known as the ---------
a. Equatorial low
b. Sub- tropical high
c. Polar high
d. None of the above
Ans. b. sub-tropical high
10. The air in horizontal motion is called----------
a. Cyclones
b. Tornadoes
c. Wind
d. Hurricane

Ans. c. wind

68
Assertion and reason:

Directions: (Q.no. 11 and 12)

In the questions given below are two statements labelled as assertion (A) and other is labelled as
reason (R). In the context of two statements which one of the following is correct?

Codes:

a. Both A and R are true and R is the correct explanation of A


b. Both A and R are true, but R is not the correct explanation of A
c. A is true, but R is false
d. A is false, but R is true
11. Assertion (A) The pressure decreases with height.
Reason (R) the air at the surface is denser and hence, has higher pressure.
Ans. (a)Both A and R are true and R is the correct explanation of A
12. Assertion (A) Wind moves from high pressure areas to low pressure areas.
Reason (R) at any elevation pressure varies from place to place and its variation is the
primary cause of air motion.
Ans. (a) Both A and R are true and R is the correct explanation of A

Source/Diagram Based Question:


13. Read the case/ source given and answer the questions that follow by choosing the correct
option.
General Atmospheric Circulation and Its Effects on Oceans
Warming and cooling of the Pacific Ocean is the most important in terms of general
atmospheric circulation. The warm water of the central Pacific Ocean slowly drifts towards
South American coast and replaces the cool Peruvian current. Such appearance of warm
water off the coast of Peru is known as the EI Nino. The EI Nino event is closely associated
with the pressure changes in the Central Pacificand Australia. This change in pressure
condition over pacific is known as the Southern oscillation. The combined phenomenon of
Southern oscillation and EI Nino is known as ENSO. In the year when the ENSO is strong,
large- scale variations in weather occur over the world. The arid west coast of South America
receives heavy rainfall, drought occurs in Australia and sometimes in India and floods in
China. This phenomenon is closely monitored and is used for long range forecasting in major
parts of the world.
i. Warm water of Central Pacific Ocean replaces which oceanic current near
South American Coast?

Ans.The warm water of Central Pacific ocean slowly drifts towards South American Coast
and replaces the cool Peruvian current.

ii. Drifting of warm water from Central Pacific to the coast of Peru results in
phenomenon known as.

69
Ans. Driftingof warm water from Central Pacific to the coast of Peru results in a
Phenomenon known as EI Nino. EI Nino is closely associated with the pressure changes in
the central Pacific and Australia.

iii. What is the result of Strong ENSO?

Ans.The result of Strong ENSO to heavy rainfall on West South American coast, drought in
Australia and flooding in China.

14. Study the given diagram and answer the questions that follow by choosing the correct
answer.

i. What type of wind is shown by the Symbol V?


a. Easterly wind
b. Sea Breeze
c. Geostrophic wind
d. Jet stream

Ans. c. Geostrophic wind

ii. Which of the following forces determines the velocity of the wind?
a. Coriolis force
b. pressure gradient force
c. gravitational force
d. magnetic force

Ans. b. Pressure gradient force

iii. The Coriolis force is zero in which of the following regions?


a. Polar region
b. Mountains
c. Equatorial region
d. Temperate region.
Ans. c. Equatorial region.

70
Short answer type questions:
1. How pressure is horizontally distributed? Explain in brief.

Ans. Small difference in pressure is highly significant in term of the wind direction and
velocity. Horizontal distribution of pressure is studies by drawing isobars at constant levels.
Isobars are lines connecting places having equal pressure. In order to eliminate the effect of
altitude on pressure, it is measured at any station after being reduced to sea level for the
purpose of comparison.

2. Explain the pressure belt on the globe and world distribution of sea level pressure.
Ans. On the basis of mode of genesis, pressure belts are divided into two broad
categories i.e. low pressure belt and high pressure belt. A brief description is given below:
i. Equatorial low pressure belt-
ii. Sub-tropical high pressure belt –
iii. Sub-polar low pressure belt-
iv. Polar high pressure belt- (explain it)
3. What are factors which determine the pattern of planetary wind?
Ans. The pattern of planetary wind largely depends upon following aspects:
i. Latitudinal variation of atmospheric heating
ii. Emergence of pressure belt
iii. The migration of belts following apparent path of the sun
iv. The distribution of continents and oceans.
v. The rotation of the earth
4. Front is boundary zone between two air masses. Discuss its various types.
Ans. When two different air masses meet, the boundary zone between them is called a
front. Fronts may be classified into four types:
i. Stationary front
ii. Cold front
iii. Warm front
iv. Occluded front (explain)
5. Extra-tropical cyclones have many important characteristics. Identify them.
Ans. the important characteristics of extra-tropical cyclones are as follows:
i. These cyclones have a clearfrontal system
ii. They cover a large area and can originate over the land and sea.
iii. They affect a much larger area as compared to tropical cyclone.
iv. The extra-tropical cyclones moves from west to east but tropical cyclones move
from east to west.
v. They are less destructive and causes light showers.

Long answer type questions:

1. What are primary or planetary winds? State their major features.

71
Ans: Primary wind or planetary winds:
i. Planetary winds constitute aa large scale motion of atmosphere under the influence
of pressure gradients, Coriolis effect and frictional force.
ii. They are related to general circulatory pattern of winds on a rotating earth’s surface.
iii. There is no effect of seasonal heating and land water contrast on the earth ‘s surface.
iv. The primary winds are –
a. Trade winds
b. Westerlies winds
c. Polar easterlies.

Major features:

The pattern of planetary winds depends largely on the following:


i. Latitudinal variation of the atmospheric heating.
ii. Emergence of pressure belts.
iii. The migration of belts following the path of sun rays.
iv. Distribution of continents and oceans.
v. Rotation of the earth.

The pattern of movement of planetary winds is called general circulation of the


atmosphere.

2. Explain the air massesand their major characteristics.


Ans. The air masses are a large body of air which fairly uniform properties of temperature
and humidity over thousands of square kilometres. The properties of air masses are as-
i. Temperature
ii. Humidity
iii. Hydrostatic moisture content
iv. Stability
v. Abundance of minor components

Characteristics:

i. Characteristics of air masses are acquired manly from its source region.
ii. Which it movement from the source region, it is modified by weather conditions
prevailing along the route.
iii. There are two basic air masses i. polar ii. Tropical
iv. They are based upon the differentiation of temperature.
v. They are further sub-division into continental and maritime on the basis of humidity.

MAP BASED QUESTIONS:

1. Identify the distribution of pressure (in millibars) in January month shown by the lines
marked as (i), (ii), (iii), (iv) and (v), on the map of the world given below:

72
Ans. i. low pressure-1000mb

Ii. High pressure –1030mb

iii. High pressure –1020mb

iv. High pressure –1020mb


v.High pressure –1015mb

2. Identify the distribution of pressure (in millibars) in January month shown by the lines
marked as (i), (ii), (iii), (iv) and (v), on the map of the world given below:

73
Ans. i. High pressure-1015mb

Ii. High pressure –1020mb

iii. Low pressure –1010mb

iv. Low pressure –1005mb


v.Low pressure –1000mb

====================0====================

74
Chapter: 10
Water in the Atmosphere
MCQs:

1. What is known the amount of water vapours present in the atmosphere?


a) Dew point
b) Saturation
c) Humidity
d) Fog
2. What is called the conversion of water vapourinto water?
a) Fog
b) Condensation
c) Humidity
d) Dew
3. What is called the mixture of fog with smoke?
a) 7Dew
b) Humidity
c) Smog
d) Fog
4. What is the actual amount of water vapour presentin the atmosphere called?
a) Absolute humidity
b) Relative humidity
c) Humidity
d) None
5. Which one of the following process is responsible for transforming liquid into vapour?
a) Condensation
b) Transpiration
c) Evaporation
d) Precipitation
6. Which one of the following is the highest cloud in the sky?
a) Stratus
b) Nimbus
c) Cumulus
d) Cirrus
7. Which are termed as hygroscopic condensation nuclei?
a) Dust particles
b) Smoke
c) Ocean-salt
d) All
8. Name the clouds having feathery appearance and always white in colour.
a) Cirrus
b) Cumulus
c) Stratus
d) Nimbus
9. Identify the clouds look like cotton wool.

75
a) Cirrus
b) Cumulus
c) Stratus
d) Nimbus
10. What are layered clouds called?
a) Cirrus
b) Cumulus
c) Stratus
d) Nimbus
11. Where is rain shadow area found?
a) Windward side
b) Leeward side
c) Coastal area
d) All
12. SOURCE BASED:
After the condensation of water vapour, the release of moisture is known as
precipitation. This may take place in liquid or solid form. The precipitation in the form
of water is called rainfall, when the temperature in lower than 0°c, precipitation takes
place in form of fine flakes of snow and is called snowfall. Besides rain and snow
other forms of precipitation are sleet and hail.
Answer the following questions:
I) Name the different forms of precipitation.
II) What is rainfall?
III) What is snowfall?
IV) What is precipitation?
13. SOURCE BASED:
The amount of water vapour in the atmosphere is added or withdrawn due to
evaporation and condensation respectively. Evaporation is a process by which water
is transformed from liquid to gaseous state. Heat is a main cause for evaporation.
The temperature at which the water starts evaporating is referred to as the latent
heat of vapourisation.
Answer the following questions:
I) By which one of the followings water vapour in the atmosphere is added?
a) Evaporation
b) Condensation
c) Humidity
d) None
II) By which one of the followings water vapour in the atmosphere is withdrawn?
a) Evaporation
b) Condensation
c) Humidity
d) None
III) Name the main cause for evaporation.
a) Condensation
b) Humidity
c) Heat
d) None
IV) What is referred to the temperature at which the water starts in evaporating?

76
a) Condensation
b) Humidity
c) Heat
d) Latent heat of vapourisation

LONG ANSWER QUESTIONS:


14. Name three different types of rainfall and explain them with diagram.
Ans:i)
Conventional Rain- The air on being heated becomes light and rises up inform of
convection currents. As it rises, it expands and loses its heat and consequently,
condensation takes place and Cumulus clouds are formed. With thunder and
lightening, heavy rainfall takes place but this does not last long. Such rainfall takes
place in equatorial regions and interior part of the continents.

ii) Orographic Rain:It is known as relief rain also.When the saturated air mass comes
across the mountain, it rises up along the slope of mountain. After reaching a considered
height, it becomes cool and condensation takes place. The chief characteristic of this sort of
rain is that the windward side, receive greater rainfall. After giving rain on the windward
side, when these winds reach the other slope, they descend, and their temperature rises.
Then their capacity to take in moisture increases and hence, these leeward side remain
rainless and dry that is called rain-shadow area.

77
III) Cyclonic Rain: In a cyclone, the winds from all directions converge at the centre. Air with
different temperature meets at the center. The cold air pushes the warm air upward, which gets cool.
As it moves up and precipitation occurs. So rainfall due to cyclones and depression is termed as
cyclonic rainfall.

15. Name the various forms of precipitation and explain them.


Followings are the various forms:
I) Dew: When the moisture deposited in form of water droplet son cooler surfaces
of solid objects. it is called dew.
II) Fog: When the temperature of an air mass containing large quantity of water
vapour falls all of a sudden, condensation takes place within itself on fine dust
particles that he known as fog.
III) Smog: Smoke with Fog is known as smog. It is found in urban and industrial
areas.
IV) Rainfall : the precipitation in the form of water is called rainfall
V) Snowfall: When the precipitation takes place in the form of fine flakes of snow.
It is known as snowfall.
VI) Hailstones : Sometimes, drops of rain after being released by the clouds become
solidified into small rounded solid pieces of Ice and which reach the surface of
the earth are called hailstones.
16. What are clouds? Name the different types of clouds and explain them.
Ans. Clouds are the mass of minute water droplets or tiny crystals of ice formed by the condensation
of water vapour in free air at considerable elevations.
Types-
I) Cirrus: They are thin and detached having a feathery appearance. Highest clouds.
II) Cumulus: Cumulus clouds look like cotton wool. They exist in patches and can be seen
scattered here and there.
III) Stratus: These are layered clouds covering large portions of the sky.
IV) Nimbus: Nimbus clouds black or dark gray. They are found very near to the surface.
Sometimes, the clouds are so low that they seem to touch the ground. Nimbus clouds are
shapeless masses of thick vapour.
17. Identify the major precipitation regimes of the world on the basis of total amount of annual
precipitation. Name them and explain them.

78
ANS.: Four precipitation regimes have been identified-

1) Areas receiving heavy rainfall- Amount of rainfall is over 200cm per annum.
2) Areas receiving moderate rainfall- Amount of rainfall is 100- 200cm per
annum.
3) Areas receiving low rainfall- Amount of rainfall is 50-100cm per annum.
4) Areas receiving very low rainfall- Amount of rainfall is less than 50cm per
annum.

79
CHAPTER: 11

WORLD CLIMATE AND CLIMATE CHANGE


MCQs:

1. Which type of climate he expressed by the letter code ET?


a) Tropical monsoon
b) Mediterranean
c) Subtropical
d) Tundra
2. Which protocol was held for reducing the emission of greenhouse gases into the
atmosphere in 1997?
a) Rio de Janeiro Earth Summit
b) Kyoto Protocol
c) Johnsonburg meet
d) Copenhagen Meet
3. Which one of the followings is not a greenhouse gas?
a) Carbon dioxide
b) Chlorofluorocarbons
c) Methane
d) Oxygen
4. Which gas does absorb ultraviolet rays in the stratosphere ?
a) Ozone
b) Nitrogen
c) carbon dioxide
d) Methane
5. What arecalled the materials thrown in atmosphere by volcanic eruptions?
a) Aerosoles
b) solid waste
c) Alcohol
d) Carbonyloxide
6. At which place the highest said temperature of 58°C was recorded in Libya on 13
September 1922 ?
a) Tripoli
b) Al Aziziyah
c) Taiwan
d) Tikari
7. Koppen’s system of classification of climates can be termed as:
a) Applied
b) Systematic
c) Genetic
d) Empirical
8. Most of the Indian peninsula will be grouped according to Koppen’ssystem under :
a) Af
b) BSh
c) Cfb
80
d) Am
9. Koppen’sclassification of climate is based on:
a) Temperature
b) Precipitation
c) Both
d) None
10. Identify the broad approaches adopted for classifying climate.
a) Empirical
b) Genetic
c) Applied
d) All
11. The largest concentration of greenhouse gases in the atmosphere is :
a) Carbon dioxide
b) Nitrous oxide
c) Methane
d) CFCs
12. SOURCE BASED QUESTIONS:
The term greenhouse is derived from the analogy to a greenhouse used in
cold areas for preserving heat. A greenhouse is made up of glass . The glass
which is transparent to incoming short wave solar radiation is opaque to
outgoing longwave radiation. The glass, therefore, allows in more radiation
and prevents the long wave radiation going outside the glass house, causing
the temperature inside the glasshouse structure warmer than outside. When
you enter a car or a bus, during summers, where windows are closed, you feel
more heat than outside. Likewise during winter the vehicles with closed doors
and windows remain warmer than the temperature outside. This is another
example of greenhouse effect.
I) For which area is greenhouse effective?
a) Cold
b) Hot
c) Both
d) None
II) Incoming solar radiation come in form of:
a) Short waves
b) Long waves
c) Both
d) None
III) Why do we feel more heat than outside, Whenwe enter into a car or a
bus, during summers, where windows are closed?
a) Due to greenhouse effect
b) Due to cold house effect
c) Due to bamboo house effect
d) All
IV) The glass in greenhouse is opaque for:
a) Long wave radiation
b) Short wave radiation
c) Both
d) None

81
LONG ANSWER QUESTIONS:
13. What do you understand by the term “Greenhouse Gases”? Make a list of
greenhouse gases.
ANS.- Ggreenhouse gases are those gases which cause global warming and result in rising
in atmospheric temperature. These gases absorb long wave radiation. The processes that
warm the atmosphere are often collectively referred to as the greenhouse effect.

Greenhouse Gases are-Carbon dioxide, Chlorofluorocarbons, Methane,Nitrous oxoxide,


Ozone,Nitric oxide, Carbon monoxide etc.

14. What is global warming? What are its effects.


ANS. –The increasingtrend in concentration of greenhouse gases in atmosphere may, in
the long run, warm upearth. It is called global warming.

Followings are its effects:

i) Rise in the sea level


ii) Loss of coastal land

iii) A change in precipitation pattern

iv) Increased risks of droughts and floods

v) Threats to biodiversity.

82
Chapter: 12
Water (Oceans)
Multiple choice questions (1 marks)

1. Which of the following is the shallowest part of oceans?


a) Continental shelf
b) Continental slope
c) Abyssal plain
d) Sea trenches
Ans. a) Continental shelf
2. Which of the following is not correct regarding continental shelf?
a) It is the extended margin of each continent occupied by relatively shallow seas
and gulfs.
b) The width of the continental shelves vary from one ocean to another.
c) This regions are the source of fossil fuel.
d) Average gradient of continental shelf varies between 2-5°.
Ans. d) Average gradient of continental shelf varies between 2-5°.
3. Which of the following indicates the end of continental boundary?
a) Continental shelf
b) Continental slope
c) Abyssal plain
d) Sea trenches
Ans. b) Continental slope
4. Which of the following part of oceans consider the source of strong earthquake?
a) Continental shelf
b) Continental slope
c) Abyssal plain
d) Sea trenches
Ans. d)Sea trench
5. Assertion (A): The temperature of surface water in oceans decreases from the
equator towards the poles
Reason (R): The amount of insolation decreases poleward.
a) Both the statements are correct and R justify A.
b) Both the statements are correct but R does not justify A
c) Only A is correct
d) Only R is correct.
Ans. a)Both the statements are correct and R justify A.
6. Which of the following is not a characteristic feature of thermocline?
a) It is the second layer from the surface of sea water to the bottom
b) It is characterised by rapid increase in temperature with increasing depth.
c) The thermocline is 500 -1,000 m thick.
d) About 90 per cent of the total volume of water is found below the thermocline
in the deep ocean.
Ans.c) It is characterised by rapid increase in temperature with increasing
depth.
7. What is the average temperature of surface water of the oceans?
a) 25°C
b) 27°C

83
c) 29°C
d) 31°C
Ans.b) 27°C
8. Which of the following water bodies recorded highest salinity?
a) Dead Sea
b) Great Salt Lake
c) Lake Van
d) Red Sea
Ans. c) Lake Van
9. Which of the following sentences define ‘halocline’?
a) It is the layer of ocean water, characterised by rapid increase in temperature
with increasing depth.
b) It is a distinct zone where salinity of ocean water increases sharply.
c) It is the layer of low salinity water which rests above the high salinity water.
d) It is the layer of ocean water where density of water increases rapidly.
Ans. b) It is a distinct zone where salinity of ocean water increases sharply.
10. Identify the minor really feature of oceans:
a) Deep sea plain
b) Deep sea trenches
c) Mid oceanic ridges
d) Continental slope
Ans. c) Mid oceanic ridges

Short answer type questions (3 marks)


11. What are the differences between continental shelf and slope?
Ans.
Sl Continental shelf Continental slope
no
1 It is the shallowest part of the ocean It connects the continental shelf and
which connects coast and continental the ocean basins.
slope.
2 The average gradient of the shelf is 1° or The gradient of the slope region varies
even less. between 2-5°.
3 The shelf typically ends at a very steep The slope end at continental rise from
slope, called the shelf break. where abyssal plain starts.
4 Massive sediment deposits here and Least or no depositions are found here
hence fossil fuels are found here. due to its degree of slope.

12. What are the layers of temperature found in ocean water from surface to bottom?
Write their features.
Ans. Following three layers are found from the surface to the bottom of ocean water:
i. The first layer represents the top layer of warm oceanic water and it is about
500m thick with temperatures ranging between 20° and 25° C.
ii. The second layer called the thermocline layer lies below the first layer and is
characterised by rapid decrease in temperature with increasing depth. The
thermocline is 500 -1,000 m thick.
iii. The third layer is very cold and extends upto the deep ocean floor.

Source based questions (3 marks)

84
13. Study the following diagram and answer the questions given below:

i. Name the relief features of ocean which is most sensitive for strong
earthquakes.
ii. Mention any two minor relief features of Ocean.
iii. Write a difference between guyot and seamounts.
Ans.
I. The relief features of ocean which is most sensitive part of strong
earthquakes are deep sea trenches.
II. Two minor relief features of ocean our guyot and seamounts.
III. Seamounts are volcanic mountains with pointed summits, rising from the
seafloor that does not reach the surface of the ocean. But Guyots are the
flat topped seamounts.
Long answer type questions (5 marks)
14. What do you understand by the term ‘salinity of ocean water’? What are the factors
affecting salinity of ocean water? Explain.
Ans. Salinity is the term used to define the total content of dissolved salts in sea
water. It is calculated as the amount of salt (in gm) dissolved in 1,000 gm (1 kg) of
seawater. It is usually expressed as parts per thousand (%o) or ppt.
Factors: Factors affecting ocean salinity are mentioned below:
(i)The salinity of water in the surface layer of oceans depend mainly on evaporation
and precipitation.
(ii)Surface salinity is greatly influenced in coastal regions by the fresh water flow from
rivers, and in polar regions by the processes of freezing and thawing of ice.
(iii)Wind, also influences salinity of an area by transferring water to other areas.
(iv)The ocean currents contribute to the salinity variations. Salinity, temperature and
density of water are interrelated. Hence, any change in the temperature or density
influences the salinity of water in an area.
15. What is ‘thermocline’? Explain any three factors which are responsible for distribution
of temperatureof ocean water.
Ans.Thermocline: Thermocline is the second layer from the surface of sea water to
the bottom where temperatureincreases rapidly with increasing depth. It is 500 -
1,000 m thick.

Factors: The factors which affect the distribution of temperature of ocean water are :
i. Latitude : the temperature of surface water decreases from the equator
towards the poles because the amount of insolation decreases poleward.

85
ii. Distribution of land and water: the oceans in the northern hemisphere receive
more heat due to their contact with larger extent of land than the oceans in the
southern hemisphere.
iii. Prevailing wind : Onshore wind influences the temperature of water near
coastal area. It results into the longitudinal variation in the temperature.
iv. Ocean currents : warm ocean currents raise the temperature in cold areas
while the cold currents decrease the temperature in warm ocean areas.
Map based questions
16. Label the following on the map of world:
A. Black Sea
B. Baltic Sea
C. Caspian Sea
D. Mediterranean Sea
E. North Sea
F. Red sea
G. Bay of Fund

86
Chapter : 13
MOVEMENTS OF OCEAN WATER
Key Points:
➢ The horizontal and vertical motions are common in ocean water bodies. The horizontal
motion refers to the ocean currents and waves. The vertical motion refers to tides.
➢ Ocean currents are the continuous flow of huge amount of water in a definite direction
while the waves are the horizontal motion of water.
➢ Water moves ahead from one place to another through ocean currents while the water in
the waves does not move, but the wave propagates ahead. The vertical motion refers to the
rise and fall of water of oceans and seas that are called tides.
➢ Once in a month, when the moon’s orbit is closest to the earth (perigee), unusually high
and low tides occur. During this time the tidal range is greater than normal. Two weeks later,
when the moon is farthest from earth (apogee), the moon’s gravitational force is limited and
the tidal ranges are less than their average heights.
➢ When the earth is closest to the sun (perihelion), around 3rd January each year, tidal
ranges is also much greater, with unusually high and unusually low tides. When the earth is
farthest from the sun (aphelion), around 4th July each year, tidal ranges are much less than
average.
➢ The time between the high tide and low tide, when the water level is falling, is called the
ebb. The time between the low tide and high tide, when the tide is rising, is called the flow or
flood.
➢ On the basis of the frequency, tides may be of three types: a) Semi-diurnal tide, b)
Diurnal tide and c) Mixed tide.
➢ On the basis of the Sun, Moon and the Earth Positions, tides may be of two types: a)
Spring tide and b) Neap tide.
➢ Ocean currents depend on certain factors like temperature differences by solar energy,
wind, gravity, coriolis force, density and salinity differences in ocean water and so on.
Ocean currents may be of two types: Warm Ocean current and Cold ocean current.

Multiple Choice Questions (MCQs)

Q1. Ocean waves primarily get energy from which of the following source?

a) Water b) Wind c) Tide d) None of these

Answer: b) Wind

Q2. Upward and downward movement of Ocean water is known as the:

a) Tide b) Wave c) Currents d) None of these

Answer: a) Tide

Q3. The highest and lowest point of waves is known as:

87
a) Crest and trough c) Trough and wave

b) Wave period and wave length d) Wave length and wave amplitude

Answer: a) Crest and trough

Q4. The horizontal distance between two successive wave crests is known as:

a) Wave amplitude b) Wave period c) Wave height d) Wave length

Answer: d) Wave length

Q5. In which date of the year the earth is farthest from the Sun (Aphelion)?

a) 3rd January b) 3rd February c) 4th July d) 4th September

Answer: c) 4th July

Q6. In which date of the year the earth is nearest to the Sun (perihelion)?

a) 3rd January b) 3rd February c) 4th July d) 4th September

Answer: a) 3rd January

Q7. The distance between the earth and the moon is the minimum when the moon is in:

a) Aphelion b) Perihelion c) Perigee d) Apogee

Answer: c) Perigee

Q8. The distance between the earth and the moon is the maximum when the moon is in:

a) Aphelion b) Perihelion c) Perigee d) Apogee

Answer: d) Apogee

Q9. Which of the following place in the world records the highest tidal range?

a) Bay of Fundy, Canada c) Gulf of Mexico

b) Bay of Bengal d) Hudson Bay

Answer: a) Bay of Fundy, Canada

Q10. Spring tides are caused:

a) As result of the moon and the sun pulling the earth gravitationally in the same direction

b) As result of the moon and the sun pulling the earth gravitationally in the opposite direction

c) Indention in the coast line

d) None of the above

Answer: a) As result of the moon and the sun pulling the earth gravitationally in the same
direction

88
Source Based Questions (SBQs)

Q11. Read the given passage carefully and answer the questions that follow:

The ocean currents may be classified based on their depth as surface currents and deep
water currents: (i) surface currents constitute about 10 per cent of all the water in the ocean,
these waters are the upper 400 m of the ocean; (ii) deep water currents make up the other
90 per cent of the ocean water. These waters move around the ocean basins due to
variations in the density and gravity. Deep waters sink into the deep ocean basins at high
latitudes, where the temperatures are cold enough to cause the density to increase.

Ocean currents can also be classified based on temperature: as cold currents and warm
currents: (i) cold currents bring cold water into warm water areas. These currents are
usually found on the west coast of the continents in the low and middle latitudes (true in
both hemispheres) and on the east coast in the higher latitudes in the Northern Hemisphere;
(ii) warm currents bring warm water into cold water areas and are usually observed on the
east coast of continents in the low and middle latitudes (true in both hemispheres). In the
northern hemisphere they are found on the west coasts of continents in high latitudes.

a) Into how many categories can ocean currents be classified on the basis of their depth?
Write their names also.

Answer: On the basis of deepness, ocean currents may be grouped into two categories: a)
Surface currents and b) Deep water currents

b) Where is Warm Ocean currents observed?

Answer: Warm ocean currents are usually observed on the east coast of continents in the
low and middle latitudes (true in both hemispheres). In the northern hemisphere they are
found on the west coasts of continents in high latitudes.

c) Write one characteristic of surface currents.

Answer: surface currents constitute about 10 per cent of all the water in the ocean, these
waters are the upper 400 m of the ocean.

Q12. Observe the given diagram and answer the following questions:

89
a) What is wave crest and wave trough?

Answer: The highest and lowest points of a wave are called the crest and trough
respectively.

b) What results waves in the ocean?

Answer: Waves are actually the energy that is resulted due to blow of wind over the surface
of the seas and oceans.

c) How do waves travel?

Answer: Waves travel because wind pushes the water body in its course while gravity pulls
the crests of the waves downward. The falling water pushes the former troughs upward, and
the wave moves to a new position

Short Answer Type Question (SAQs)

Q13. Where do waves in the ocean get their energy from?

Waves are actually the energy that is resulted due to blow of wind over the surface of the
seas and oceans. Wind causes waves to travel in the ocean and the energy is released on
shorelines. The largest waves are found in the open oceans. Waves continue to grow larger
as they move and absorb energy from the wind. The energy depends upon three factors:
the wind velocity, the period of time during which the wind has blown and the length of the
fetch. The fetch is the maximum distance of open water over which the wind can blow, and
so places with the greatest fetch potentially receive the highest-energy waves.

Q14. Differentiate between spring tides and neap tides.

Spring tides occur when the sun, the moon and the earth are in a straight line that increases
in the gravitational attraction and results the highest high tide and the lowest low tide and
the maximum tidal range. Spring tides occur twice a month, one on full moon period and
another during new moon period. On the other hand, Midway between the spring tides
(approximately seven day interval) are the neap tides, which occur when the sun earth and
moon form a right angle, with the earth at the apex. As the sun’s attraction partly counter
balances that of the moon, the tidal range is at a minimum with the lowest of high tides and
the highest of low tides.

Q15. Why are tides important?

I) Tides are predictable that helps the navigators and fishermen plan their activities. Tidal
flows are of great importance in navigation. Tidal heights are very important, especially
harbours near rivers and within estuaries having shallow depth near the coast which
prevents big ships to enter into the harbours

II) Tides are also helpful in desalting the sediments and in removing polluted water from
river estuaries.

90
III) Tides are used to generate electrical power that is known as Tidal Energy.

Q16. How do ocean currents affect the climate? Explain.

The oceanic circulation through currents transports the heat from one latitude belt to
another in a manner similar to the heat transported by the general circulation of the
atmosphere. The cold waters of the Arctic and Antarctic circles move towards warmer water
in tropical and equatorial regions, while the warm waters of the lower latitudes move
polewards. Consequently, it helps to maintain the latitudinal heat balance of the planet
earth. Moreover, when a warm current passes along a coast, it makes the place warmer.
Similarly, when a cold current passes along a coast, it makes the place colder.

Long Answer Type Questions (LAQs)

Q17. What cause the occurrence of tides? Explain with appropriate diagram.

The moon’s gravitational pull to a great extent and to a lesser extent the sun’s gravitational
pull, are the major causes for the occurrence of tides The periodical rise and fall of the sea
level, once or twice a day, mainly due to the attraction of the sun and the moon, is called a
tide. Another factor is centrifugal force, which is the force that acts to counter balance the
gravity. Together, the gravitational pull and the centrifugal force are responsible for creating
the two major tidal bulges on the earth. On the side of the earth facing the moon, a tidal
bulge (high tide) occurs while on the opposite side though the gravitational attraction of the
moon is less as it is farther away, the centrifugal force causes tidal bulge (high tide) on the
other side. The low tides occupy the sides of Earth midway between the two tidal bulges.
The difference in sea level between high tide and low tide is called the tidal range. The
highest tidal range occurs in the world in the Bay of Fundy, Canada. When the tide is
channelled between islands or into bays and estuaries they are called tidal currents.

Figure 1: Relationship between gravitational forces and tides

91
Q18. What are the primary forces that influence the ocean currents? Explain

The primary forces that influence the currents are:

I) Heating by solar energy: It causes the water to expand. That is why, near the equator
the ocean water is about 8 cm higher in level than in middle latitudes. This causes a very
slight gradient and water tends to flow down the slope.

II) Wind: Wind blowing on the surface of the ocean pushes the water to move. Friction
between the wind and the water surface affects the movement of the water body in its
course.

III) Gravity: Gravity tends to pull the water down the pile and create gradient variation.

IV) The coriolis force: The Coriolis force intervenes and causes the water to move to the
right in the northern hemisphere and to the left in the southern hemisphere.

These large accumulations of water and the flow around them are called Gyres. These
produce large circular currents in all the ocean basins.

Map Based Questions

Q19. Identify the following currents from the given map.

A) Humboldt current

B) California current

C) Oyashio current

D) Canaries current

E) Falkland current

F) Kuroshio current

G) Gulf Stream and

H) Aughlas current

92
Figure -2: Major currents in the Pacific, Atlantic and Indian ocean

93
CHAPTER 14
BIODIVERSITY AND CONSERVATION

MIND MAPPING: SUMMARY

⚫ Biodiversity is a combination of two words, Bio (life) and diversity (variety).


⚫ Simply, biodiversity is the number and variety of organisms found within a specified
geographic region.
⚫ It is the result of 2.5-3.5 billion years of evolution of lives.
⚫ The number of species globally vary from 2 million to 100 million.

Biodiversity can be discussed at three levels:

(i) Genetic diversity


(ii) Species diversity
(iii) Ecosystem diversity.

(i) Genetic Diversity

94
⚫ Genes are the basic building blocks of various life forms.
⚫ It refers to the variation of genes within species.

Note: Human beings genetically belong to the Homo Sapiens group.

(ii) Species Diversity:

⚫ It refers to the variety of species and the number of species in a defined area.
⚫ It can be measured through its richness, abundance and types.
⚫ Some areas are richer in species than others.
⚫ Areas rich in species diversity are called hotspots of diversity.

(iii) Ecosystem Diversity

95
The broad differences between ecosystem types and the diversity occurring within each
ecosystem type constitute the ecosystem diversity.

Importance of Biodiversity

It has contributed in many ways to the development of:

⚫ Human culture and human communities.


⚫ It have played a major role in shaping the diversity of nature at the genetic, species and
ecological levels.

THE ROLES/IMPORTANCE OF BIODIVERSITY:

Ecological Role of Biodiversity

⚫ Species capture and store energy.


⚫ It produces and decompose organic materials.
⚫ It helps to cycle water and nutrients throughout the ecosystem.
⚫ That means, every organism needs to contribute something of useful to other organisms.

Economic Role of Biodiversity

96
• Biodiversity is an important resource in their day-to-day life.
• One important part of biodiversity is ‘crop diversity’, which is also called agro-
biodiversity.
• The important economic commodities that biodiversity supplies to humankind are: food
crops, livestock, forests, fish, medicinal resources, etc.
Scientific Role of Biodiversity

It also helps in understanding how life functions and the role of each species in sustaining.

LOSS OF BIODIVERSITY

⚫ Tropical rain forests contain 50 per cent of the species on the earth, destruction of natural
habitats have proved disastrous for the entire biosphere.

⚫ Natural calamities such as earthquakes, floods, volcanic eruptions, forest fires,


droughts, etc. cause damage to the flora and fauna of the earth.

⚫ Pesticides and other pollutants such as hydrocarbons and toxic heavy metals destroy
the weak and sensitive species.
97
⚫ Species, which are not the natural inhabitants of the local habitat but are introduced into the
system, are called exotic species.
The International Union of Conservation of Nature and Natural Resources (IUCN) has
classified the threatened species of plants and animals into three categories for the
purpose of their conservation.

Endangered Species

⚫ It includes those species, which are in danger of extinction.


⚫ The IUCN publishes information about endangered species worldwide as the Red List of
threatened species.
Vulnerable Species

This includes the species, which are likely to be in danger of extinction in near future.

Rare Species

⚫ Population of these species is very small in the world.


⚫ They are confined to limited areas or thinly scattered in the world.

98
CONSERVATION OF BIODIVERSITY: IMPORTANCE

⚫ Biodiversity is important for human existence.


⚫ They cause degradation in the environment.
⚫ The Government of India along with 155 other nations have signed the Convention of
Biodiversity at the Earth Summit held at Rio de Janeiro, Brazil in June 1992.

STEPS FOR BIODIVERSITY CONSERVATION:

⚫ Efforts should be made to preserve the species that are endangered.


⚫ Prevention of extinction requires proper planning and management.
⚫ Varieties of food crops, forage plants, timber trees, livestock, animals and their wild
relatives should be preserved.
⚫ Each country should identify habitats of wild relatives and ensure their protection.
⚫ Habitats where species feed, breed, rest and nurse their young should be safeguarded
and protected.
⚫ Regulate international trade for wild plants and animals.

Note: To protect, preserve and propagate the variety of species within natural
boundaries, the Government of India passed the Wild Life (Protection) Act, 1972

MEGA DIVERSITY

⚫ A large numbers of the world’s species diversity is called Mega Diversity.


⚫ There are 12 such countries, namely Mexico, Columbia, Ecuador, Peru, Brazil etc.

Note: The International Union for the Conservation of Nature and Natural Resources
(IUCN) has identified certain areas as biodiversity hotspots.

4. Map based questions from the following map.(identification based)

99
1. A hotspot of Brazil

2. A hotspot of Australia

3. A hotspot of India

4. A hotspot of Africa

1. Multiple choice questions.

(1) Conservation of biodiversity is important for:

(a) Animals (b) Plants

(c) Animals and plants (d) All organisms

(2.) Threatened species are those which:

(a) Threaten others (b) Lion and tiger

(c) Area abundant in number (d) are suffering from the danger of extinction

(3.) National parks and sanctuaries are established for the purpose of:

(a) Recreation (b) Pets


100
(c) Hunting (d) Conservation

(4.) Biodiversity is richer in:

(a) Tropical Regions (b) Temperate Regions


(c) Polar Regions (d) Oceans

(5) In which one of the following countries, the ‘Earth Summit’ was held?

(a) UK (b) Brazil

(c) Mexico (d) China

(6). Which of the following is an area rich in biodiversity ______?

(a) Tropical region(b) Polar region

(c) Temperate tropical region(d) Oceanic area

(7). In which of the following countries was the Earth Summit held?

(a) U. K(b) Mexico

(c) Brazil(d) China

(8). According to the following estimates, what percentage of freshwater fish in South
America are still unclassified?

(a)40 percent (b) 2 percent

(c) 4 percent ((d) 100 percent

(9). What percentage of the following species are estimated to be extinct?

(a) 99 percent(b)50 percent

(c) 75 percent(d) 100 percent

(10) What is a group of organisms having the following similar physical characteristics
called?

(a) Species (b) Organism

(c) Gene (d) None of the above

2. Passage based question-answer:

A. passage no. 1

101
Biodiversity as we have today is the result of 2.5-3.5 billion years of evolution. Before the
advent of humans, our earth supported more biodiversity than in any other period.
Since, the emergence of humans, however, biodiversity has begun a rapid decline,
with one species after another bearing the brunt of extinction due to overuse. The
number of species globally vary from 2 million to 100 million, with 10 million being the
best estimate. New species are regularly discovered most of which are yet to be
classified (an estimate states that about 40 per cent of fresh water fishes from South
America are not classified yet). Tropical forests are very rich in bio-diversity.

Answer the following question..1*3=3

1. When did biodiversity start on the earth?

2. Number of species varies from………… to…… ……..

3. Why does tropical forest has 40 % population of trees and plants?

B. Passage no.2

There are some countries which are situated in the tropical region; they possess a
large number of the world’s species diversity. They are called mega diversity centres.
There are 12 such countries, namely Mexico, Columbia, Ecuador, Peru, Brazil,
Democratic Republic of Congo, Madagascar, China, India, Malaysia, Indonesia and
Australia in which these centres are located. In order to concentrate resources on
those areas that are most vulnerable, the International Union for the Conservation of
Nature and Natural Resources (IUCN) has identified certain areas as biodiversity
hotspots

Answer the following question..1*3=3

1. What is mega diversity?

2. How many countries ha great biodiversity

3. In which countries does Brazil located at.?

3. Answer the following questions in about 120-150 words. Marks 5

(i) What are the roles played by biodiversity in the shaping of nature?
(ii) What are the major factors that are responsible for the loss of biodiversity?
(iii) What steps are needed to prevent them?
(iv) What is biodiversity? What are the different levels of biodiversity?
(v) Explain the types of Biodiversity.

102
India: Physical Environment

Chapter – 1 India - Location


Revision Capsule

The mainland of India, extends from Kashmir in the north to Kanyakumari in the south and
Arunachal Pradesh in the east to Gujarat in the west. India‘s territorial limit further extends
towards the sea up to 12 nautical miles (about 21.9 km) from the coast.

Our southern boundary extends up to 6°45' N latitude in the Bay of Bengal. Let us work out
implications of having such a vast longitudinal and latitudinal

Extent-
The latitudinal and longitudinal extent of India are roughly about 30 degrees, whereas the
actual distance measured from north to south extremity is 3,214km, and that from east to
west is only 2,933 km.

What is the reason for this difference? This difference is based on the fact that the distance
between two longitudes decreases towards the poles whereas the distance between two
latitudes remains the same everywhere. Find out the distance between two latitudes?

From the values of latitude, it is understood that the southern part of the country lies within
the tropics and the northern part lies in the sub-tropical zone or the warm temperate zone.

This location is responsible for large variations in land forms, climate, soil types and natural
vegetation in the country. Now, let us observe the longitudinal extent and its implications on
the Indian people.

From the values of longitude, it is quite discernible that there is a variation of nearly 30
degrees, which causes a time difference of nearly two hours between the easternmost and
the westernmost parts of our country. You are familiar with the concept of Indian Standard
Time (IST).

What is the use of the standard meridian? While the sun rises in the northeastern states
about two hours earlier as compared to Jaisalmer, the watches in Dibrugarh, Imphal in the
east and Jaisalmer,Bhopal or Chennai in the other parts of India show the same time.Why
does this happen? There is a general understanding among the countries of the world to
select the standard meridian

103
in multiples of 7°30' of longitude. That is why 82°30' E has been selected as the s̳ tandard
meridian of India. Indian Standard Time is ahead of Greenwich Mean Time by 5 hours and
30 minutes.

There are some countries where there are more than one standard meridian due to their
vast east-to-west extent. For example, the USA has seven time zones.

India with its area of 3.28 million sq. km accounts for 2.4 per cent of the world‘s land surface
area and stands as the seventh largest country in the world. Find out the names of the
countries which are larger than India.

Size
Total area of India is 32, lakh sq. km 7th largest country with 2.4 % of total land area of the
world Indian subcontinent.

It includes the countries Pakistan, Nepal, Bhutan, Bangladesh and India.


Coastline length is 7516.5 km and frontier area is 15200 km.

India and its Neighbors

1. Pakistan 2. China 3. Nepal 4. Bhutan 5. Myanmar 6. Bangladesh 7. Afghanistan


8. Sri Lanka 9. Maldives..

Sri Lanka is separated from India by the Gulf of Mannar and Palk Strait

Multiple Choice Type Questions

1. Find out the local time of Jaisalmer located at 71 degree longitude when the time at
Greenwich is 12.00am midnight.
A. 04.44 pm
B. 04.45 pm
C. 04.44 am
D. 07.16 pm

Answer- C

2. Which of the following is the oldest route of contact between India and other countries of the
world?

A. Ocean routes
104
B. Air routes
C. Land routes
D. None of the above

Answer- C

3. Which of the following influences the duration of the day and night as one moves from south
to north ?

A. Longitudinal extent
B. Latitudinal extent
C. Indian Standard Time
D. All of the above

Answer- A

4. Assertion ( A ) India has a 2 hours time difference between the easternmost and
westernmost parts of the country.
Reason ( R ) India has variations of nearly 30 degrees in its latitudinal extent.

A. A and R are true and R is the right explanation of A.


B. A and R are true but R is not the right explanation of A
C. A is true but R is false
D. A is false but R is true

Answer- B

5. India lies

A. Latitudinally - North, Longitudinally - West


B. Latitudinally - South, Longitudinally - East
C. Latitudinally - North , Longitudinally - East
D. None of the above

Answer- C

6. Which of the following statements is not correct with the great physical diversity of India?

A. India is bounded by Hindu Kush and Sulaiman in the north-east.


B. The Indian subcontinent includes Pakistan,Nepal,Bhutan, Bangladesh and India.
C. The vast sandy expanse of Marusthali lies in the western part of India
D. None of the above.

Answer- A

7. Indian Standard Time is ____________ of ___________________by 5 hours 30


minutes.
105
A. ahead, Greenwich Mean Time
B. behind, Pacific standard Time
C. behind, Greenwich Mean Time
D. None of the above

Answer- A

8. If you work out of the latitudinal and longitudinal extent of India, they are roughly about
30 degrees, whereas the actual distance measured from north to south extremity is 3214
km and that from east to west is only 2933 km.What is the reason for this difference?

A. Distance between two longitudes decreases toward the pole


B. Distance between two latitudes remains the same everywhere.
C. Both
D. None of the above.

Answer- C

9. Assertion ( A ) 82 degree 30 minutes E has been selected as the standard meridian of


India.
Reason ( R ) There is a general understanding among the countries of the world to
select the standard meridian in multiples of 7 degree 30 minutes of longitude.

A. A and R are true and R is the right explanation of A.


B. A and R are true but R is not the right explanation of A.
C. A is true but R is false
D. A is false but R is true.

Answer- A

10. The neighboring countries that share their boundaries with India are

A. Afghanistan and Pakistan


B. Myanmar and Bhutan
C. China and Nepal
D. All of the above

Answer- D

Read the source given below and answer the following questions :

106
The mainland of India extends from Kashmir in the north to Kanyakumari in the south and
Arunachal Pradesh in the east to Gujarat in the west. India’s territorial limit further extends
towards the sea up to 12 nautical miles from the coast. Our southern boundary extends up to 6
degree 45 minutes N latitude in the Bay of Bengal. Let us try to analyse the implications of
having such a vast longitudinal and latitudinal extent. If you work out the latitudinal and
longitudinal extent of India, they are roughly about 30 degrees. From the values of latitudes, it is
understood that the southern part of the country lies within the tropics and the northern part
lies in the subtropical zone or the warm temperate zone. This location is responsible for large
variations in landforms, climate , soil types and natural vegetation in the country. Now, let
us observe the longitudinalextent and its implications on the Indian people. From the values of
longitude it is quite discernible that there is a variation of nearly 30 degrees, which causes a
time difference of nearly 2 hours between the easternmost and westernmost parts of our
country. You are familiar with the concept of Indian Standard Time. What is the use of standard
meridian? While the sun rises in the north-eastern states about 2 hours earlier as compared to
Jaisalmer, the watches in Dibrugarh, Imphal in the east and Jaisalmer, Bhopal and Chennai in
the other parts of India show the same time.

11. Discuss the implications of vast latitudinal extension of India.

Answer
Variations in landforms
Variations in climate- Tropical, Sub -Tropical
Variations in types of soil
Variations in natural vegetation

12. What is the use of standard meridian?

Answer-
Uniformity in time
To determine standard time
To avoid confusion of time.

13. What do you mean by tropical and subtropical regions?


Answer-
Area ranging from 23.5 degree north to 23.5 degree south.
Area to the north and south of the tropics.

Short Answer Type Questions

14. Why does the Indian subcontinent show a distinct Geography? Give reason

Answer- The Indian subcontinent has considered a distinct geographical unit. Because it is
separated from the rest of the Asian continent by HinduKush in the north-west and in the north
by Himalayan ranges. On the southern side it is isolated by the Indian Ocean from the rest of
the world.

107
15.Why is there a time variation of 2 hours between the easternmost and westernmost part
of the country?

Answer: Longitudinal extent of India is 68°7′E to 97°25′ E (29°). There is a time


variation of 2 hours between the easternmost and the westernmost parts of our
country. The sun rises two hours earlier in Arunachal Pradesh as compared to
Gujarat. This is because the earth is tilted and also it rotates in an east to west
direction. So while rotation, the eastern parts of the world experience the sun rays
earlier as compared to the western parts of the world.

1 degree = 4 minutes

30 degree = 4 x 30 = 120 minutes

120 minutes = 2 hours

16.Differentiate between a Gulf and a Strait.

A gulf is a large body of water almost encircled by land except for a small mouth that opens
out to the sea. Gulfs can be easily associated with inland bodies of water as well as the
ocean.

A strait is a strip of water that separates two lands or two large bodies of water.Straits are
used for navigational purposes and have played an integral part when it comes to shipping
routes.

17.How is the latitudinal spread of India advantageous to her?:

1. Tropic of cancer (23 1/2°.) – It divides India into 2 parts. Southern part is in the tropical
zone while the northern part is in the temperate zone. Therefore biodiversity in India is
exceptional.
2. It also brings climatic diversity in India which has many advantages.
3. As we move from Kanyakumari to Jammu and Kashmir, the duration of day and night
decreases by more than 4 1/2 hours. We have places of all climates. It makes India a
subcontinent.

Long Answer Type Questions

18.Size of India has endowed India with unique physical diversity. Justify.

108
The size of India has endowed her with great physical diversity. The presence of lofty
mountains in the north; large rivers such as Ganga, Brahmaputra, Mahanadi, Krishna,
Godavari and Kaveri; green forested hills in the north-east and south India; and the
vast sandy expanse of Marusthali is very impressive.

It is bounded by the Himalayas in the north, Hindukush and Sulaiman ranges in the
north-west, Purvachal hills in the northeast and by the large expanse of the Indian
ocean in the south.

The Himalayas, together with other ranges, have acted as a formidable physical
barrier in the past. Except for a few mountain passes such as the Khyber, the Bolan,
the Shipkila, the Nathula, the Bomdila, etc. it was difficult to cross it.

It has contributed towards the evolving of a unique regional identity of the Indian
subcontinent. Peninsular part of India extends towards the Indian Ocean.

This has provided the country with a coastline of 6,100 km in the mainland and 7,517
km in the entire geographical coast of the mainland plus the island groups Andaman
and Nicobar located in the Bay of Bengal and the Lakshadweep in the Arabian Sea.
Thus India, as a country, is a physically diverse land providing an abundance of
varied resources.

19.Do you think physical barriers are a hindrance in interaction with our neighbouring
countries in modern times? Give some examples how we have overcome these difficulties
in the present day.?

Answer- Most Upvoted Answer Do you think physical barrier is a hindrance in interaction
with our n... Physical barriers have been a hindrance in interaction with neighbouring
countries since ages. Even Vindhya Mountain separated North and South India and it
seems funny in the present day. But it is not as much as before. India could have a very
good relation with Pakistan looking at our physical boundary. That boundary could have
been a harbinger of scenic beauty. But Physical barrier juxtaposes with the political barrier
to make the situation worse. Despite having friendly ties with Afghanistan and for not having
amicable relations with Pakistan,Indo-Afghan relation is not realising its true potential. So
the conclusion is that Physical barriers can be a hindrance in maintaining good relations
with neighbouring countries for a developing nation. We can overcome them by

1) Maintaining peace with other countries.

2) Trying to help out each other....

3) Making Good Relationships.

By these measures we can establish peace and harmony among countries.

20.Explain in detail about size and extension of India.

109
Total area of India is 32,87,263 sq. Km. From the point of view of the area, India is the
seventh largest country of the world. Its population is 1210 million and it has second position
in the world after China from a population point of view.

The mainland of India, extends from Kashmir in the north to Kanyakumari in the south and
Arunachal Pradesh in the east to Gujarat in the west. India extends from the territorial limit
further towards the sea up to 12 nautical miles (about 21.9 km) from the coast. India lies
entirely in the Northern hemisphere.

The mainland extends between latitudes 8 4′ N (Kanyakumari, Cape Caverian) and 37 6′ N (


Indira col, Jammu and Kashmir) and longitudes 68 7′ E (Dwarka, Gujarat) and 97 25′ E
(Sadiya, Arunachal Pradesh).

Longitudinal extent of India is 68 7′ to 97 25′ E -(29 ). India’s land length is 15,200 km and it
has a coastline of 6,100 km in the mainland and 7,517 km in the entire geographical coast
of the mainland plus the island groups Andaman and Nicobar located in the Bay of Bengal
and the Lakshadweep in the Arabian Sea Thus India, as a country, is a physically diverse
land providing occurrence of varied resources. India is located in the south central part of
the continent of Asia, bordering the Indian Ocean and its two arms extending in the form of
Bay of Bengal and the Arabian Sea. This maritime location of Peninsular India has provided
links to its neighboring regions through the sea and air routes.

21.On the physical outline map of India locate and label the following with suitable symbols.

a. Latitudinal extent of India


b. Longitudinal extent of India
c. Standard Meridian of India
d. Important latitude passing through India [ Tropic of Cancer ]
e. Southernmost point of mainland of India [ Kanyakumari ]

110
Chapter-2
STRUCTURE AND PHYSIOGRAPHY
Key Points

➢ The earth and its landforms that we see today have evolved over a very long time.
➢ Current estimation shows that the earth is approximately 460 million years old. Over these
long years, it has undergone many changes brought about primarily by the endogenic and
exogenic forces.
➢ Indian plate was to the south of the equator millions of years ago.
➢ Over millions of years, this plate broke into many parts and the Australian plate moved
towards the southeastern direction and the Indian plate to the north.
➢ Based on the variations in its geological structure and formations, India can be divided into
three geological divisions. These geological regions broadly follow the physical features: (a) The
Penisular Block (b) The Himalayas and other Peninuslar Mountains and (c) Indo-Ganga-
Brahmaputra Plain.
➢ The Peninsula is formed essentially by a great complex of very ancient gneisses and
granites, which constitutes a major part of it. Since the Cambrian period, the Peninsula has
been standing like a rigid block with the exception of some of its western coast which is

111
submerged beneath the sea and some other parts changed due to tectonic activity without
affecting the original basement.
➢ The Himalayas along with other Peninsular mountains are young, weak and flexible in their
geological structure unlike the rigid and stable Peninsular Block.
➢ The third geological division of India comprises the plains formed by the river Indus, the
Ganga and the Brahmaputra. Originally, it was a geo-synclinal depression which attained its
maximum development during the third phase of the Himalayan mountain formation
approximately about 64 million years ago. Since then, it has been gradually filled by the
sediments brought by the Himalayan and Peninsular rivers. Average depth of alluvial deposits in
these plains ranges from 1,000-2,000 m.
➢ ‘Physiography’ of an area is the outcome of structure, process and the stage of development.
Based on these macro variations, India can be divided into the following physiographic divisions:

I) The Northern and North-eastern Mountains

II) The Northern Plain

III) The Peninsular Plateau

IV) The Indian Desert

V) The Coastal Plains

VI) The Islands.

112
Figure 1: Major physiographic divisions of India

Multiple Choice Questions (MCQs)

Q1. The collision of the Indian plate into the Eurasian plate about 50 million years ago resulted
in the erection of the mountain chain which contains the highest peaks on the earth today.
Name the mountain chain.

a) Andese b) Himalayas c) Rockies d) Alps

Answer: b) Himalayas

113
Q2. Which of the following is the approximate length of the Great Himalayan range, also known
as the central axial range from east to west?

a) 2300 km b) 2400 km c) 2500 km d) 2600 km

Answer: c) 2500 km

Q3. Which of the following is the oldest and the most stable landmass of India?

a) Himalayas b) Northern Plain c) Peninsular Plateau d) Coastal Plain

Answer: c) Peninsular Plateau

Q4. It is an extension of the main peninsular plateau in the northeastern part of India. Which of
the following part is this?

a) Rajmahal Hills c) Karbi-Anglong Plateau

b) Chota Nagpur Plateau d) Nilgiri Hills

Answer: c) Karbi-Anglong Plateau

Q5. Which of the following is the highest peak of peninsular plateau?

a) Anaimudi b) Dodabetta c) Nilgiri hills d) Mahendragiri

Answer: a) Anaimudi

Q6. Garo, Khasi and Jaintia hills named after the tribal groups inhabiting this region from west to
east are located in the:

a) Choto Nagpur plateau c) Karbi-Anglong plateau

b) Meghalaya plateau d) Rajmahal hills

Answer: b) Meghalaya plateau

Q7. On which of the following hill range is the ‘Dodabeta’ peak situated?

a) Nilgiri hills b) Anaimalai hills c) Cardamom hills d) Nallamala hills

Answer: a) Nilgiri hills

Q8. Assertion (A): The Meghalaya plateau has a highly eroded surface.

Reason (R): This area receives maximum rainfall from the south west monsoon.

a) Only assertion is correct

b) Only reason is correct

c) Both assertion and reason are correct and reason is the correct explanation for assertion

d) Both assertion and reason are correct but reason is not the correct explanation for assertion

114
Answer: c) Both assertion and reason are correct and reason is the correct explanation for
assertion

Q9. The largest river of the Thar Desert falls into the Rann of Kutch, making it a typical example
of inland drainage. Which of the following river is this?

a) Luni River b) Ghaghar River c) Mahi River c) Chambal river

Answer: a) Luni River

Q10. Western coastal plain in Maharashtra is known as:

a) Konkan coast b) Kathiawar coast c) Goan coast d) Malabar coast

Answer: a) Konkan coast

Q11. This coast has got certain distinguish features in the form of ‘Kayals’ (backwaters), which
are used for fishing, inland navigation and also due to its special attraction for tourists.

a) Konkan coast b) Kathiawar coast c) Goan coast d) Malabar coast

Answer: d) Malabar coast

Q12. Assertion (A): The Western coastal plain is a narrow belt and provides natural conditions
for the development of ports and harbours.

Reason (R): The western coastal plain is an example of submerged coastal plain.

a) Only assertion is correct

b) Only reason is correct

c) Both assertion and reason are correct and reason is the correct explanation for assertion

d) Both assertion and reason are correct but reason is not the correct explanation for assertion

Answer: c) Both assertion and reason are correct and reason is the correct explanation for
assertion.

Q13. The only active volcano in India is:

a) Andaman island b) Barren island c) Minicoy d) Lakshdweep island

Answer: b) Barren island

Q14. The Lakshdweep Island is made of which of the following deposits?

a) Coral b) Volcanic c) Alluvial d) Moraine

Answer: a) Coral

115
Source Based Questions (SBQs)

Q15. Read the given passage carefully and answer the questions that follow:

To the northwest of the Aravali hills lies the Great Indian desert. It is a land of undulating
topography dotted with longitudinal dunes and barchans. This region receives low rainfall below
150 mm per year; hence, it has arid climate with low vegetation cover. It is because of these
characteristic features that this is also known as Marusthali. It is believed that during the
Mesozoic era, this region was under the sea. This can be corroborated by the evidence
available at wood fossils park at Aakal and marine deposits around Brahmsar, near Jaisalmer
(The approximate age of the wood-fossils is estimated to be 180 million years). Though the
underlying rock structure of the desert is an extension of the peninsular plateau, yet, due to
extreme arid conditions, its surface features have been carved by physical weathering and wind
actions. Some of the well pronounced desert land features present here are mushroom rocks,
shifting dunes and oasis (mostly in its southern part). Most of the rivers in this region are
ephemeral. The Luni river flowing in the southern part of the desert is of some significance. Low
precipitation and high evaporation makes it a water deficit region.

a) Why does the desert region of India known as Marusthali?

Answer: The desert region of India receives low rainfabelow 150 mm per year. Hence, the
region has arid cliate with low vegetation cover. It is because of these characteristic features
that this is also known as Marusthali.

b) What are the evidences that support the fact that during Mesozoic era the Indian desert was
under the sea?

Answer: The evidence available at wood fossils park at Aakal and marine deposits around
Brahmsar, near Jaisalmer support the fact that during Mesozoic era the Indian desert was under
the sea.

c) What are the important land features of Indian desert?

Answer: Some of the well pronounced desert land features present in Indian desert are
mushroom rocks, shifting dunes and oasis.

Q16. Observe the given map and answer the following questions:

116
a) What is the location of Indian desert in respect to Aravalli range?

Answer: The Great Indian desert is located to north west of Aravalli range.

b) Which physiographic division is located between the Himalayas and the Peninsular plateau?

Answer: The Northern plain is located between the Himalayas and the Peninsular Plateau.

c) Which river is located between the Vindhya and the Satpura ranges?

Answer: The Narmada river is located between the Vindhya and the Satpura ranges.

d) “The general elevation of the peninsular plateau is from the west to the east, which is also
proved by the pattern of the flow of rivers.” Justify the statement.

Answer: The general slope direction of peninsular plateau is from west to east. Consequently,
most of the peninsular rivers like Godavari, Krishna, Kaveri are originated from the Western
Ghats and flows towards the east to the Bay of Bengal.

Short Answer Type Questions (SAQs)

117
Q17. Into how many geological divisions can India be divided? Write down their name and
describe each geological division in brief.

India can be divided into three broad geological divisions. They are:

i) The Peninsular Block: It is the oldest geological division of India that is located to the south
of the northern plains. Most part of it consists of a great complex of very ancient gneisses and
granites. Since the Cambrian period, the Peninsula has been standing like a rigid block with the
exception of some of its western coast which is submerged beneath the sea and some other
parts changed due to tectonic activity without affecting the original basement. The Peninsula
mostly consists of relict and residual mountains like the Aravali hills, the Nallamala hills, the
Javadi hills, the Veliconda hills, the Palkonda range and the Mahendragiri hills, etc. Some rivers
like Narmada, Tapi are flowing through rift valleys. .

ii) The Himalays and other Peninsular Mountains: The Himalayas along with other
Peninsular mountains are young, weak and flexible in their geological structure unlike the rigid
and stable Peninsular Block. These mountains are tectonic in origin, dissected by fast-flowing
rivers which are in their youthful stage. Various landforms like gorges, V-shaped valleys, rapids,
waterfalls, etc. are indicative of this stage.

iii) Indo-Ganga-Brahmaputra Plain: This is the newest geological division of India comprise
the plains formed by the river Indus, the Ganga and the Brahmaputra. Originally, it was a geo-
synclinal depression that has been gradually filled by the sediments brought by the Himalayan
and Peninsular rivers. Average depth of alluvial deposits in these plains ranges from 1,000-
2,000 m.

Q18. Define the term physiography. India can be sub-divided into how many physiographic
divisions? Write down their name.

‘Physiography’ of an area is the outcome of structure, process and the stage of development.

Based on these macro variations, India can be divided into the following physiographic
divisions:

I) The Northern and North-eastern Mountains

II) The Northern Plain

III) The Peninsular Plateau

IV) The Indian Desert

V) The Coastal Plains

VI) The Islands.

Q19. How is Bhabar different from Tarai?

Bhabar is a narrow belt ranging between 8 and 10 km at the foothill of the Shiwalik ranges in
which there is a sudden break of slope. Consequently, the streams and rivers coming from the
mountains deposit heavy materials of rocks and boulders, and at the same time, become
118
disappear in this zone. On the other hand, south of bhabar is the Tarai belt, with an approximate
width of 10 to 20 km. In this belt, the streams that become underground and disappear in
bhabar region, re-emerge without having any properly demarcated channels and thus creating
marshy and swampy conditions known as the Tarai. The region is associated with lush green
forest and a wide variety of wildlife.

Long Answer Type Question (LAQs)

Q20. On the basis of the prominent relief features, the peninsular plateau can be divided into
how many groups? Write down their name and describe each group in details.

Penisnsular plateau is the oldest and the most stable landmass of India. On the basis of the
prominent relief features, the peninsular plateau can be grouped into three major sub-divisions:

I) Deccan Plateau:

a) It is bounded by the Western Ghats in the west, Eastern Ghats in the east and the Satpura,
Maikal range and Mahadeo hill in the north.

b) Western Ghats are locally known by different names such as Sahyadri in Maharashtra, Nilgiri
hills in Karnataka and Tamil Nadu and Anaimalai hills and Cardamom hills in Kerala.

c) Western Ghats are comparatively higher in elevation and more continuous than the Eastern
Ghats. Their average elevation is about 1,500 m with the height increasing from north to south.
Anaimudi (2,695 m), the highest peak of Peninsular plateau is located on the Anaimalai hills of
the Western Ghats.

d) Eastern Ghats comprising the discontinuous and low hills are highly eroded by the rivers
such as the Mahanadi, the Godavari, the Krishna, the Kaveri and so on.

e) Highest peak of Eastern Ghats is Mahendragiri. Some of the important ranges are Javadi
hills, Palconda range, Nallamala hills and so on.

f) Western Ghats and Eastern Ghats meet to each other at Nilgiri hill. Highest peak of Nilgiri hill
is Dodabetta.

II) The Central Highlands:

a) The general elevation of the Central Highlands ranges between 700-1,000 m above the
mean sea level and it slopes towards the north and northeastern directions. They are bounded
to the west and south by the Aravali and satpura ranges respectively.

b) Vindhyan and Kaimur ranges are the other two important mountain ranges in this region.
Most of the tributaries of the river Yamuna have their origin in these two ranges.

c) An eastern extension of the Central Highland is formed by the Rajmahal hills, to the south of
which lies a large reserve of mineral resources in the Chotanagpur plateau.

III) The Northeastern Plateau:

119
a) The Meghalaya and Karbi Anglong plateau stand detached from the main Peninsular block.
The Meghalaya plateau is further sub-divided into three: (i) The Garo Hills; (ii) The Khasi Hills;
and (iii) The Jaintia Hills, named after the tribal groups inhabiting this region. An extension of
this is also seen in the Karbi Anglong hills of Assam.

b) This area receives maximum rainfall from the south west monsoon. As a result, the
Meghalaya plateau has a highly eroded surface.

Q21. What are the major differences between Western Ghats and Eastern Ghats?

Sl. Basis of Western Ghats Eastern Ghats


No. difference
1 Location Western Ghats are located along Eastern Ghats are located along
the west coast of India. the east coast of India.
2 Continuity Western Ghats are more Eastern Ghats comprising the
continuous than the Eastern discontinuous and low hills are
Ghats. highly eroded by the rivers such as
the Mahanadi, the Godavari, the
Krishna, the Kaveri and so on.
3 Elevation The average height of the The average height of the Eastern
Western Ghats is 1500 m and it Ghats is comparatively less than
is more than the Eastern Ghats. the Western Ghats.
4 Local hills Western Ghats are locally known Some of the important ranges are
by different names such as Javadi hills, Palconda range,
Sahyadri in Maharashtra, Nilgiri Nallamala hills and so on.
hills in Karnataka and Tamil
Nadu and Anaimalai hills and
Cardamom hills in Kerala.
5 Highest Anaimudi is the higest peak of Mahendragiri is the highest peak of
peak the Western Ghats. the Eastern Ghats.

Q22. Differentiate between western coastal plain and eastern coastal plain.

Sl. Basis of Western coastal planis Eastern coastal plains


No. difference
1 Location They lie between the Western They lie between the Eastern Ghats
Ghats and the Arabian sea. and the Bay of Bengal.
2 Type The western coastal plains are The western coastal plains are an
an example of submerged example of emerged coastal plain.
coastal plain.
3 Width The western coastal plain is The eastern coastal plain is
narrower than the eastern comparatively wider than the
coastal plain. western coastal plain.
4 Delta The rivers like Narmada, Tapi, The rivers like Mahanadi, Godavari,
Formation Sabarmati, Zuari, etc. flowing Krishna, Kaveri, etc. flowing
through this coastal plain do not through this coastal plain have
form any delta. formed well developed deltas.
5 Sub From north to south the western The eastern coastal plains can be
division coastal plains can be sub divided sub-divided into following sub-
into following sub-divisions: divisions:
i) the Kachchh and Kathiawar i) the North Circars coast and
coast in Gujarat, ii) the Coromandel coast
120
ii) the Konkan coast in
Maharashtra,
iii) the Goan coast in Karnataka
and
iv) the Malabar coast in Kerala.

Q23. Make a difference of the island groups of the Bay of Bengal and the Arabian Sea.

Sl. Basis of Islands of Bay of Bengal Islands of Arabian Sea


No. difference
1 Name of The islands of the Bay of Bengal The islands of the Arabian sea
islands include Anadaman and Nicobar include Lakshadweep and Minicoy.
Islands.
2 Location These are situated roughly These are scattered between 8°°N-
between 6°°N-14°°N and 92°°E - 12°°N and 71°°E -74°°E longitude.
94°°E.
3 Number The Bay of Bengal island groups The Arabian sea islands consist of
of islands consist of about 572 islands. approximately 36 islands.
4 Category The entire group of island is The entire group of islands is
divided into two broad categories broadly divided by the Ten degree
– the Andaman in the north and channel, north of which is the Amini
the Nicobar in the south. They Island and to the south of the
are separated by a water body Canannore Island.
which is called the Ten degree
channel.
5 Origin It is believed that these islands The entire island group is built of
are an elevated portion of coral deposits.
submarine mountains. However,
some smaller islands are
volcanic in origin. Barren island,
the only active volcano in India is
also situated in the Nicobar
islands.

MAP BASED QUESTIONS


Q24. On the given political map of India, locate and label the following features with appropriate
symbols:
A) Karakoram Range,
B) Aravalli Range
C) Vindhyan Range
D) Western Ghats,
E) Eastern Ghats,
F) K2,
G) Kanchenjunga,
H) Anaimudi,
I) Namcha Barwa,
J) Malwa Plateau,

121
K) Chhotanagpur Plateau,
L) Malabar Coast,
M) Caromandel Coast,
N) Andaman & Nicobar Islands and
O) Lakshadweep Islands

122
Chapter:3
Drainage System
Multiple Choice Questions

1. Identify the incorrect sentence from below:


a) A river drains the water collected from a specific area, which is called its
‘catchment area’.
b) An area drained by a river and its tributaries is called a drainage basin.
c) The boundary line separating one drainage basin from the other is known as the
watershed.
d) The catchments of large rivers are called watersheds while those of small rivulets
and rills are often referred to as river basins.

Ans. d) The catchments of large rivers are called watersheds while those of small
rivulets and rills are often referred to as river basins.
2. Assertion (A): River basins and watersheds are accepted as the most appropriate micro,
meso or macro planning regions.
Reason (R): In one part of the basin or watershed directly affects the other parts.
a) Both the A and R are correct and R justifies A.
b) Both the A and R are correct but R doesn’t justify A.
c) Only A is correct
d) Only B is correct.

Ans. a) Both the A and R are correct and R justifies A.


3. Statement I: The Brahmaputra is well-known for floods, channel shifting and bank erosion.
Statement II: Most of the tributaries of Brahmaputra are large, and bring large quantity of
sediments owing to heavy rainfall in its catchment area.
a) only statement I is correct
b) Only statement II is correct
c) Both the statements are correct and statement II justifies statement I.
d) Both the statements are true but statement II does not justify statement I

Ans. c) Both the statements are correct and statement II justifies statement I.
4. Match the following and choose the correct option:
Rivers Local Name
(i)Indus A.Sorrow of
Bihar
(ii) B.Dakshin
Brahmaputra Ganga
(iii) Kosi C.SingiKhamban
(iv) Godavari D.Tsangapo
Options:

a) (i)-A, (ii)-B, (iii)-C, (iv)-D


b) (i)-C, (ii)-B, (iii)-A, (iv)-D
123
c) (i)-C,(ii)-D, (iii)-A, (iv)-B
d) (i)-B, (ii)-A, (iii)-D, (iv)-C

Ans. C) (i)-C, (ii)-D, (iii)-A, (iv)-B

5. Which of the following is the largest tributary of river Indus?


a) Jhelum
b) Chenab
c) Sutlej
d) Beas
Ans. b) Chenab
6. Which of the following is the longest tributary of Ganga?
a) Son
b) Gomti
c) Damodar
d) Yamuna

Ans. d) Yamuna
7. Which of the following never once known as the ‘sorrow of Bengal’?
a) Damodar
b) Ajay
c) Mayurakshi
d) Tista

Ans. a) Damodar
8. Which of the following is the largest river system in South India?
a) Mahanadi
b) Godavari
c) Krishna
d) Kaveri

Ans. b) Godavari
9. Which of the following pair of rivers are flowing through rift Valley?
a) Ganga -Yamuna
b) Krishna - Kaveri
c) Narmada -Tapi
d) Mahanadi- Godavari

Ans. c) Narmada -Tapi


10. Which of the following waterfall is located on the way of river Narmada?
a) Shiva Samudram
b) Jog
c) Dhuandhar
d) Dudh sagar

Ans. c) Dhuandhar

Short answer type questions (3 marks)


11. Give three examples of drainage pattern.

124
Ans. The drainage pattern of an area is the outcome of the geological time period, nature
and structure of rocks, topography, slope, amount of water flowing and the periodicity of
the flow. The examples of drainage pattern are as follows:
i. The drainage pattern resembling the branches of a tree is known as “dendritic” the
examples of which are the rivers of northern plain.
ii. When the rivers originate from a hill and flow in all directions, the drainage pattern
is known as ‘radial’. The rivers originating from the Amarkantak range present a
good example of it.
iii. When the primary tributaries of rivers flow parallel to each other and secondary
tributaries join them at right angles, the pattern is known as ‘trellis’. Many rivers
originating in Himalaya have formed this type of pattern.
12. Why are the rivers polluted? Give three reasons.
Ans. Rivers are polluted due to many reasons. Such as:
i. Most of the cremation grounds are on the banks of rivers and the dead bodies are
sometimes thrown in the rivers.
ii. On the occasion of some festivals, the flowers and statues are immersed in the
rivers.
iii. Large scale bathing and washing of clothes also pollute river waters.
13. Classify the drainage basins in India on the basis of the size of watershed.
Ans. On the basis of the size of the watershed, the drainage basins of India are grouped
into three categories:
i. Major river basins with more than 20,000 sq. km of catchment area. It includes 14
drainage basins such as the Ganga, the Brahmaputra, the Krishna, the Tapi, the
Narmada, the Mahi, the Pennar, the Sabarmati, the Barak, etc.
ii. Medium river basins with catchment area between 2,000-20,000 sq. km
incorporating 44 river basins such as the Kalindi, the Periyar, the Meghna, etc.
iii. Minor river basins with catchment area of less than 2,000 sq. km include fairly
good number of rivers flowing in the area of low rainfall.
14. Write any three features of the ‘NamamiGange’ programme.
Ans.
i. NamamiGange Programme’, is an Integrated Conservation Mission, approved as
“Flagship Programme” by the Union Government in June 2014 with the twin
objectives of (a) effective abatement of pollution, (b) conservation and
rejuvenation of the National River Ganga.
ii. Main pillars of the NamamiGange Programme are:
•Sewerage Treatment Infrastructure
•River-Front Development
•River-Surface Cleaning
•Bio-Diversity
•Afforestation
•Public Awareness
•Industrial Effluent Monitoring
•Ganga Gram etc.

Source based questions (3 marks)


15. Read the following passage carefully and answer the questions given below:
The rivers of India carry huge volumes of water per year but it is unevenly distributed
both in time and space. There are perennial rivers carrying water throughout the year
while the non-perennial rivers have very little water during the dry season. During the

125
rainy season, much of the water is wasted in floods and flows down to the sea. Similarly,
when there is a flood in one part of the country, the other area suffers from drought.
a) Mention one reason behind uneven flow of water in rivers in different season in
India.
b) How can the problem regarding flood in one part of the country but drought in
another area, be solved? Give a solution.
c) Is there any way to save the wasted water during flood? Suggest a method.
Ans.

a) There is an uneven flow of water in rivers in different seasons in India because of


the variability of monsoon rainfall in different parts of the country and in different
time.
b) The problem regarding flood in one part of the country but drought in another area
be solved by planning river linking projects. It will supply the excess water from
flood areas to the drought prone areas.
c) The wasted water during flood can be managed through watershed management
programme and water harvesting methods.

Long answer type questions (5 marks)

16. How are the Himalayan rivers different from the Peninsular Rivers?
Ans.
Sl Himalayan Rivers Peninsular Rivers
no
1 Glaciers are the place of origin of most of These rivers are originated from
these rivers. monsoon rainfall.
2 Example of such rivers are Indus, Ganga, Example of such rivers are
Brahmaputra. Mahanadi, Godavari, Kaveri,
Narmada, Tapi et
3 These rivers are perineal in nature. These rivers are seasonal in
nature.
4 These rivers have large river basins. These rivers have comparatively
small river basins.
5 These rivers are younger than the These rivers are older than the
Peninsular rivers Himalayan rivers.
17. Examine the features of Himalayan rivers.
Ans. The features of Himalayan rivers are as follows:
i. The Himalayan drainage system has evolved through a long geological history.
ii. It mainly includes the Ganga, the Indus and the Brahmaputra river basins.
iii. Since these are fed both by melting of snow and precipitation, rivers of this system
are perennial.
iv. These rivers pass through the giant gorges carved out by the erosional activity
carried on simultaneously with the uplift of the Himalayas.
v. Besides deep gorges, these rivers also form V-shaped valleys, rapids and
waterfalls in their mountainous course.
vi. While entering the plains, they form depositional features like flat valleys, ox-bow
lakes, flood plains, braided channels, and deltas near the river mouth.
Map based questions
18. Locate the following river on the map of India:
i. Brahmaputra
ii. Indus
iii. Satluj
iv. Ganga
126
v. Yamuna
vi. Chambal
vii. Damodar
viii. Mahanadi
ix. Krishna
x. Kaveri
xi. Godavari
xii. Narmada
xiii. Tapti
xiv. Luni

19. Locate the following on the map of India:


i. Wular lake
ii. Sambhar lake
iii. Chilika lake
iv. Kolleru lake
v. Pulicat lake
vi. Vembanad lake
vii. Palk Strait
viii. Rann of Kachch
ix. Gulf of Kachch
x. Gulf of Mannar

127
xi. Gulf of Khambat

128
Chapter -4

CLIMATE
1. Which one of the following state receives floods in the winter?
(A).Assam. (B). West Bengal
(C) Kerala (D) Tamilnadu

2. Which one of the following gases is not a greenhouse gas?

(A ) Methane. (B). CFC


(C) Carbon dioxide. (D). Hydrogen

3. Where do the Western Cyclonic Disturbances originate ?

(A) Bay of Bengal. (B). Pacific Ocean


(C) Mediterranean sea. (D) Arabian Sea

4. -----------are dreaded evening thunderstorms in Bengal and Assam.


(A) Mango Shower (B) Blossom Shower
(C) Norwesters or Kalbaisakhi. (D) Loo

5. ……………. receives the highest amount of rainfall .

(A) Cherapunji. (B) Dras


(C) Guwahati (D) Mawsynram

6. Which one of the following phenomenon happens when the sun shines vertically over the
Tropic of Capricorn in the southern hemisphere?

(A) High pressure develops over North-western India due to low temperatures.
(B) Low pressure develops over North-western India due to high temperatures.
(C) No changes in temperature and pressure occur in north-western India.
(D) ‘Loo’ blows in the North-western Region

7.. October month is hot due to

(A) The absence of rain. (B) combination of high temperature and high humidity.
(C) Dry hot weather. (D) Low pressure

There are two statements given below, marked as Assertion (A) and Reason (R). Read the
statements and choose the correct option. ………..

(A) A is true but R is false (B) A is false but R is true.


(C) Both A and R are true and R explains A. (D) Both A and R are true but R does not explain A
Assertion…. Tamilnadu coast remains dry during SW monsoon season.

129
Reason ….It is parallel to Bay of Bengal branch of SW monsoon and it lies in the rain shadow
area of Arabian sea branch .

9 . Assertion.. “.Pune receives low rainfall 70 cms’


Reason. …Pune is located on the Windward side of monsoon winds.

10 .Assertion…. “The easterly jet stream steers the tropical depression into India”
Reason….This depression plays a significant role in the distribution of rainfall.

SOURCE-BASED QUESTIONS (3 MARKS)

11. Read the following passage carefully and answer the following questions……l

El-Nino is a complex weather system that appears once every three to seven years, bringing
drought, floods and other weather extremes to different parts of the world.

The system involves oceanic and atmospheric phenomena with the appearance of warm
currents off the coast of Peru in the Eastern Pacific and affects weather in many places
including India. El-Nino is merely an extension of the warm equatorial current which gets
replaced temporarily by cold Peruvian current or Humbolt current (locate these currents in your
atlas). This current increases the temperature of water on the Peruvian coast by 10°C. This
results in:

(i) the distortion of equatorial atmospheric circulation; (ii) irregularities in the evaporation of sea
water;

(iii) reduction in the amount of planktons which further reduces the number of fish in the sea.

The word El-Nino means 'Child Christ because this current appears around Christmas in
December. December is a summer month in Peru (Southern Hemisphere).

El-Nino is used in India for forecasting long range monsoon rainfall. In 1990-91, there was a wild
El-Nino event and the onset of southwest monsoon was delayed over most parts of the country
ranging from five to twelve days.

Answer the following questions

i) What is meant by El-Nino?


ii) What are the effects of El-Nino on Indian weather conditions?
iii) What are the results of increased temperature of water on the Peruvian Coast ?

Q 12.. The Inter Tropical Convergence Zone (ITCZ) is a low pressure zone located at the
equator where trade winds converge, and so, it is a zone where air tends to ascend. In July, the
ITCZ is located around 20°N-25°N latitudes (over the Gangetic plain), sometimes called the
monsoon trough. This monsoon trough encourages the development of thermal low over north
and northwest India. Due to the shift of ITCZ, the trade winds of the southern hemisphere cross
the equator between 40° and 60°E longitudes and start blowing from southwest to northeast
due to the Coriolis force. It becomes southwest monsoon. In winter the ITCZ moves South word
and so the reversal of winds from North East to south and Southwest takes place. They are
called northeast monsoons.
130
I) What is Inter Tropical Convergence Zone?
ii) What is Coriolis Force ?
iii) How does the reversal of winds from Northeast to south and Southwest take place ?

LONG ANSWER QUESTIONS. ( 5 MARKS )

Q13. Discuss the factors that determine the climate of India.


ANS : The following factors determine the climate of India

i) Latitude: - Due to the curved surface of the Earth, the amount of solar energy received
varies according to latitude. The Tropic of Cancer divides India into subtropical (North)
and tropical (South) areas, so the tropical area receives more heat than the subtropical
area.
ii) Distribution of Land and water….

-India is flanked by the Indian Ocean on three sides in the south and bordered by a high
and continuous mountain-wall in the north. As compared to the landmass, water heats
up or cools down slowly. This differential heating of land and sea creates different air
pressure zones in different seasons in and around the Indian subcontinent. Difference in
air pressure causes reversal in the direction of monsoon winds.

iii). The Distance From The Sea: The Sea exerts a moderating influence on a climate as
the distance from the sea increases its moderating influence decreases and the
people experience extreme weather conditions. This condition is known as
continentality.

iv) Altitude: - Temperature decreases with height Due to thin air, places in the mountains
are cooler than places on the plains. For example, Agra and Darjiling are located on the
same latitude, but temperature of January in Agra is 16°C whereas it is only 4°C in
Darjiling.
v) Relief Features: Nature of Relief plays a major role in determining the climate of a place
e.g. high mountains act as barriers for cold or hot winds; they may also cause
precipitation. Desert soil has the capacity to hold more heat which leads to rise in
temperature

vi) The Pressure And Winds: -The pressure and the wind system of an area depend on the
latitude and altitude of the place.

a) During summer low pressure is created over interior Asia as well as India, this leads to
incoming of South-West Monsoon winds causing rainfall.

b) During winter the high pressure area north of the Himalaya

Q 14. What is Global Warming? What are the effects of Global Warming? Or

What is global warming? Give its causes and effects.

ANS. Global warming is a gradual increase in the earth’s average surface temperature over the
past one to two centuries, generally due to the greenhouse gases caused by increased levels of
carbon dioxide, CFCs, and other pollutants.
131
Causes of Global Warming: -

1) Carbon dioxide is the major source of global warming. This gas is released to the
atmosphere by burning of fossil fuel.

2) Other gases like methane, chlorofluorocarbons, ozone and nitrous oxide which are present in
much smaller concentrations in the atmosphere, together with carbon dioxide are known as
greenhouse gases. These gases are contributing to global warming.

3) Rapid industrialization and technological changes, the revolution in agriculture and transport
sectors has resulted in large supplies of carbon dioxide and methane which cause global
warming.

Effects of Global Warming: -

1) Due to global warming the polar ice caps and mountain glaciers would melt and the amount
of water in the ocean would increase.

2) Melting of glaciers and sea-ice due to global warming leads to rise in the sea level up to 48
cm by the end of twenty first century.

3) Increase the incidence of annual flooding.

4) Insect-borne diseases like malaria, and leads to shift in climatic boundaries, making some
regions wetter and other driers.

5) Agricultural pattern would shift and human population as well as

the ecosystem would experience change.

6) The peninsular India would be submerged

Q.15. Give the main characteristics of the monsoonal rainfall.

ANS.

1) Rainfall received from the southwest monsoons is seasonal in character, which occurs
between June and September.

2) Monsoonal rainfall is largely governed by relief or topography. For instance the windward side
of the Western Ghats registers a rainfall of over 250 cm.

3) During the southwest monsoon period, the monsoon rainfall has a declining trend with
increasing distance from the sea. Kolkata receives 119 cm, Patna 105 cm,

Allahabad 76 cm and Delhi 56 cm.

4) The monsoon rains occur in wet spells of few days duration at a time. The wet spells are
interspersed with rainless interval known as ‘breaks’.

5) The summer rainfall comes in a heavy downpour leading to considerable run off and soil
erosion.

132
6) Monsoons play a pivotal role in the agrarian economy of India because over three-fourths of
the total rain in the country is received during the southwest monsoon season.

7) Its spatial distribution is also uneven which ranges from 12 cm to more than 250 cm.

8) The beginning of the rains sometimes is considerably delayed over the whole or a part of the
country.

9) The rains sometimes end considerably earlier than usual, causing great damage to standing
crops and making the sowing of winter crops difficult.

Q 16. How economic life in India is affected by monsoon?


Or
“Monsoon is a gambling for Indian farmers.” Explain.

ANS.

1) The entire agricultural cycle of India revolve around Monsoon. In India about 64% people
depend on agriculture for their livelihood and agriculture itself is based on southwest monsoon.

2) Except Himalayas all the parts of the country have temperature above the threshold level to
grow the crops or plants throughout the year.

3) Variability of rainfall brings droughts or floods every year in some parts of the country.

4) Agricultural development of India depends on timely and adequately distributed rainfall. If it


fails, agriculture is adversely affected particularly in those regions where means of irrigation are
not less.

5) Sudden monsoon burst creates problem of soil erosion over large areas in India.

6) Winter rainfall by temperate cyclones in north India is highly beneficial for Rabi crops.

7) Regional climatic variation in India is reflected in the vast variety of food, clothes and house
types.

8) Sudden monsoon burst creates problems of soil erosion over large areas in India.

9) Winter rainfall by temperate cyclones in north India is highly beneficial for Rabi crops.

10) Regional climatic variation in India is reflected in the vast variety of food, clothes and house
types.

Q 17.. What is the Inter-Tropical Convergence Zone (ITCZ)? How does it attract south
monsoon? What are the important factors which influence the mechanism of Indian weather?

ANS .

ITCZ is the zone near the Equator from where the north-east trade winds and the south-east
trade winds meet each other. It changes its position with the effect of the vertical rays of the
sun. It is near the Tropic of Cancer in the north in summer season and near the Tropic of

133
Capricorn in the south in the winter season at about 25°N, as a result of which the south-east
trade winds cross over the equator and enter in India as the south-west monsoon.

Factors influencing the mechanism of Indian weather are: -

I) Surface distribution of pressure and winds include monsoon, location of low and high
pressure.

II) Upper air circulation which includes global weather conditions like air masses and the jet
stream.

III) Atmospheric disturbances like western cyclones and tropical cyclones cause rainfall.

These are the factors which affect the Indian weather.

MAP-BASED QUESTIONS ( 1MARK)

Q .18. On the map of India show the following :-

I) Areas having more than 200 cms annual rainfall


II) Areas having 100 –200 cms annual rainfall
III) Areas having 50—100 cms annual rainfall
IV) Areas having less than 50 cms rainfall
Ans…..

134
ANSWERS

ANSWER MCQ( 1MARK)

1. D

2. D

3. C

4. C

135
5. D

6. A

7.B

8. C

9.A

10. D

SOURCE-BASED QUESTIONS

ANS 11.

I) El-Nino is a complex whether system that appears once every 3 to 7 years bringing
drought floods and other weather extremes to different parts of the world.

ii) Appearance of El Nino cause extreme weather conditions like delay in monsoon,
droughts , floods famines in India .

iii) The increased temperature of water in Peruvian Coast results in the distortion of
equatorial atmospheric circulation Irregularities in the evaporation of sea water
Reduction in the amount of planktons.

ANS 12.

I) Inter Tropical Convergence Zone is a low pressure zone located at the equator where
trade winds converge and so it is a zone where tends to ascend.

ii) Coriolis force is an apparent force caused by the earth's rotation. The Coriolis force is
responsible for deflecting winds towards the right in the northern hemisphere and
towards the left in the southern hemisphere.

iv) In winter the ITCZ moves southward and so the reversal of winds from North East to
south and Southwest takes place.

136
Chapter: 5
Natural Vegetation
Key Points:
➢ Natural vegetation refers to a plant community that has been left undisturbed over a long
time, so as to allow its individual species to adjust themselves to climate and soil
conditions as fully as possible.
➢ India is endowed with natural vegetation and wild life.
➢ Depending upon the variations in the climate and the soil, the vegetation of India
changes from one region to another.
➢ India is one of the 12 mega bio-diversity countries of the world. With about 47,000 plant
species India occupies tenth place in the world and fourth in Asia in plant diversity.
➢ Tropical deciduous forest covers larger part of India.
➢ During the British Period, they exploited vegetation mercilessly for commercial purpose.
➢ For the conservation and management of the forest resources, the Government of India
adopted the national Forest Policy in 1950.
➢ For the conservation of wild life different schemes such as Project Tiger (1973), Project
Elephant (1992), Crocodile Breeding Project, Project Hangul have been introduced.
➢ There are 18 Biosphere Reserves in India. Eleven Biosphere Reserves have been
recognised by the UNESCO on World Network of Biosphere Reserves.

MCQs

Choose the right answer from the four alternatives given below.
(i) Sandalwood is an example of:
(a) Evergreen forest (c) Deltaic forest
(b) Deciduous forest (d) Thorny forest
(ii) Which one of the following was the purpose of Project Tiger?
(a) to kill tigers (c) to protect tigers from illegal hunting
(b) to put tigers in the Zoo (d) to make films on tigers
(iii) In which one of the following states is the Nandadevi Biosphere reserve situated?
(a) Bihar (c) Uttarakhand
(b) Uttar Pradesh (d) Odisha
(iv) In India there are 18 Biosphere Reserves in India. How many have been recognised by the
UNESCO?
(a) 18 (c) 11
(b) 15 (d) 6

137
(v) Those forests which shed their leaves during dry season:
(a) Deciduous forests
(b) Evergreen forests
(c) Tidal forests
(d) Thorny forests.
(vi) Sunderbans or mangrove trees are found in:
(а) Deciduous forests
(b) Semi-Evergreen forests
(c) Tidal forests
(d) Thorny forests.
(vii) Where is Sunderbans biosphere reserve located?
(a) In Ganga river delta
(b) Orissa
(c) Chhattisgarh
(d) Madhya Pradesh.
(viii) Which one of the following was first biosphere of India?
(a) Nilgiri Biosphere
(b) Nanda Devi Biosphere
(c) Sunderbans Biosphere
(d) Gulf of Mannar Biosphere.

Short Answer Type Questions

Q. ‘India is a land of great variety of natural vegetation.’ How ?


Ans: India is a land of great variety of natural vegetation.
Himalayan heights are marked with temperate vegetation; the Western Ghats and the Andaman
Nicobar Islands have tropical rain forests, the deltaic regions have tropical forests and
mangroves; the desert and semi desert areas of Rajasthan are known for cactii, a wide variety
of bushes and thorny vegetation.
India is one of the 12 mega bio-diversity countries of the world. With about 47,000 plant
species India occupies tenth place in the world and fourth in Asia in plant diversity. There are
about 15,000 flowering plants in India, which account for 6 per cent in the world’s total number
of flowering plants. The country has many non-flowering plants, such as ferns, algae and fungi.
India

138
also has approximately 90,000 species of animals, as well as, a rich variety of fish in its fresh
and marine waters.
Q. What are the types of forests found in India?
Ans: Types of Forests found in India:
(i) Tropical Evergreen and Semi-Evergreen forests
(ii) Tropical Deciduous forests
(iii) Tropical Thorn forests
(iv) Montane forests
(v) Littoral and Swamp forests.
Q. Mention the main features of tropical thorn forests.
Characteristics of Tropical Thorn Forests
➢ Tropical thorn forests occur in the areas which receive rainfall less than 50 cm.
➢ These consist of a variety of grasses and shrubs. It includes semi-arid areas of
south west Punjab, Haryana, Rajasthan, Gujarat, Madhya Pradesh and Uttar
Pradesh.
➢ In these forests, plants remain leafless for most part of the year and give an
expression of scrub vegetation.
➢ Important species found are babool, ber, and wild date palm, khair, neem, khejri,
palas, etc

Q. What is meant by ‘Sholas’?


Ans: The temperate forests are called Sholas in the Nilgiris, Anaimalai and Palani hills.

Q. Where are the mangrove forests found?


Ans: In India, the mangrove forests spread over 6,740 sq. km which is 7 per cent of the world’s
mangrove forests. They are highly developed in the Andaman and Nicobar Islands and the
Sunderbans of West Bengal. Other areas of significance are the Mahanadi, the Godavari and
the Krishna deltas.

Long Answer Questions

Q. Name the type of forests found in the region receives rainfall above 200cm? Mention its
characteristics.
Ans: Tropical Evergreen Forest is found in the region receives rainfall above 200cm.
Characteristics of the tropical evergreen forests:

139
➢ These forests are found in the western slope of the Western Ghats, hills of the north-
eastern region and the Andaman and Nicobar Islands.
➢ They are found in warm and humid areas with an annual precipitation of over 200 cm and
mean annual temperature above 22oC.
➢ Trees are very tall and straight, trees reach great heights up to 60 m or above.
➢ The ground and are covered with shrubs and creepers, with short structured trees.
➢ In these forests, there is no definite time for trees to shed their leaves, flowering and
fruition. As such these forests appear green all the year round.
➢ Important species found in these forests include rosewood, mahogony, aini, ebony, etc.

Q. Why did the British exploit the forests in India?


Ans: The British were aware of the economic value of the forests in India, hence, large scale
exploitation of these forests was started.
The structure of forests was also changed.
The oak forests in Garhwal and Kumaon were replaced by pine (chirs) which was needed
to lay railway lines.
Forests were also cleared for introducing plantations of tea, rubber and coffee.
The British also used timber for construction activities as it acts as an insulator of heat.
The protectional use of forests was, thus, replaced by commercial use.

Q. Name the type of forest found in the region receiving rainfall between 70 and 200 cms.
What are its two types? Mention two characteristics of each type.
Ans: Monsoon type of forest / Deciduous type of forests are found in the region receiving rainfall
between 70 and 200 cms. Its two types are : (a) Moist Deciduous Forests and (b) Dry
Deciduous Forests
Characteristics of Tropical Deciduous Forests:
➢ These are the most widespread forests in India.
➢ They are also called the monsoon forests.
➢ They spread over regions which receive rainfall between 70-200 cm. On the basis of the
availability of water, these forests are further divided into moist and dry deciduous.
(a) The Moist deciduous forests :
➢ These forests are found in the regions which record rainfall between 100-200 cm.
➢ These forests are found in the north-eastern states along the foothills of Himalayas,
eastern slopes of the Western Ghats and Odisha.
➢ Teak, sal, shisham, hurra,mahua, amla, semul, kusum, and sandalwood etc. are the
main species of these forests.

140
(b) Dry deciduous forest:
➢ It covers vast areas of the country, where rainfall ranges between 70 -100 cm.
➢ On the wetter margins, it has a transition to the moist deciduous, while on the drier
margins to thorn forests.
➢ These forests are found in rainier areas of the Peninsula and the plains of Uttar Pradesh
and Bihar.
➢ Tendu, palas, amaltas, bel, khair, axlewood, etc. Are the common trees of these forests.

Q. “The Himalayan ranges show a succession of vegetation from the tropical to the tundra,
which change in with the altitude.” How?
Ans:
• Deciduous forests are found in the foothills of the Himalayas.
• Deciduous forest is succeeded by the wet temperate type of forests between an altitude
of 1,000-2,000 m.
• In the higher hill ranges of north-eastern India, hilly areas of West Bengal and
Uttaranchal, evergreen broad leaf trees such as oak and chestnut are predominant.
• Between 1,500-1,750 m, pine forests are also well-developed in this zone, with Chir Pine
as a very useful commercial tree.
• Deodar, a highly valued endemic species grows mainly in the western part of the
Himalayan range.
• Deodar is a durable wood mainly used in construction activity.
• Blue pine and spruce appear at altitudes of 2,225-3,048 m. At many places in this zone,
temperate grasslands are also found.
• Silver firs, junipers, pines, birch and rhododendrons, etc. occur between 3,000-4,000 m.
• At higher altitudes, mosses and lichens form part of the tundra vegetation.

Q. When was the National Forest Policy passed? State its four objectives.
Ans: The national Forest Policy was in 1952. It was further modified in 1988. According to the
new forest policy, the Government will emphasise sustainable forest management in order to
conserve and expand forest reserve.
The forest policy aimed at :
(i) Bringing 33 per cent of the geographical areas under forest cover;
(ii) Maintaining environmental stability and to restore forests where ecological balance
was disturbed;
(iii) Conserving the natural heritage of the country, its biological diversity and genetic
pool;
141
(iv) Checks soil erosion, extension of the desert lands and reduction of floods and
droughts;
(v) Creating of a massive peoples movement involving women to encourage planting of
trees, stop felling of trees and thus, reduce pressure on the existing forest.
Q. What are the reasons for declining wildlife in India?
Ans: Some reasons of the declining of wildlife are as follows:
(i) Industrial and technological advancement brought about a rapid increase in the
exploitation of forest resources.
(ii) More and more lands were cleared for agriculture, human settlement, roads, mining,
reservoirs, etc.
(iii) Pressure on forests mounted due to lopping for fodder and fuelwood and removal of
small timber by the local people.
(iv) Grazing by domestic cattle caused an adverse effect on wildlife and its habitat.
(v) Hunting was taken up as a sport by the elite and hundreds of wild animals were killed
in a single hunt. Now commercial poaching is rampant.
(vi) Incidence of forest fire.
Q. What is social forestry? What are its types?
Ans: Social forestry means the management and protection of forests and afforestation on
barren lands with the purpose of helping in the environmental, social and rural development.
Social forestry into three categories.
a. These are Urban forestry,
b. Rural forestry and
c. Farm forestry.

142
MAP WORK
Q. Locate the following on the outline map of India.
a. Region of Tropical Evergreen Forests
b. Region of Thorn Forests
c. the First biosphere reserve in India
d. Nanda Devi Biosphere reserve

143
SOURCE BASED QUESTIONS
Q. Read the following paragraph and answer the questions that follow:
The Himalayan ranges show a succession of vegetation from the tropical to the tundra, which
change in with the altitude. Deciduous forests are found in the foothills of the Himalayas. It is
succeeded by the wet temperate type of forests between an altitude of 1,000-2,000 m. In the
higher hill ranges of northeastern India, hilly areas of West Bengal and Uttaranchal, evergreen
broad leaf trees such as oak and chestnut are predominant.
144
Between 1,500-1,750 m, pine forests are also well-developed in this zone, with Chir Pine as a
very useful commercial tree. Deodar, a highly valued endemic species grows mainly in the
western part of the Himalayan range. Deodar is a durable wood mainly used in construction
activity. Similarly, the chinar and the walnut, which sustain the famous Kashmir handicrafts,
belong to this zone. Blue pine and spruce appear at altitudes of 2,225-3,048 m. At many places
in this zone, temperate grasslands are also found. But in the higher reaches there is a transition
to Alpine forests and pastures. Silver firs, junipers, pines, birch and rhododendrons, etc. occur
between 3,000-4,000 m. However, these pastures are used extensively for transhumance by
tribes like the Gujjars, the Bakarwals, the Bhotiyas and the Gaddis. The southern slopes of the
Himalayas carry a thicker vegetation cover because of relatively higher precipitation than the
drier north-facing slopes. At higher altitudes, mosses and lichens form part of the tundra
vegetation.

Q. Why does the vegetation type vary with increasing altitude?


a. because of air pressure change
b. because of change in temperature
c. due to change in amount of rainfall
d. all of the above
Q. Transhumance means....
a. shifting of pasture or grasslands
b. seasonal migration of animals with the herders
c. rearing of animals in the higher altitude
d. All of the above
Q. Rhododendrons is found at an altitude between:
a. 2,225-3,048 metres
b. 3,000-4,000 metres
c. 1,000-2,000 metres
d. Below 1000 metres
Q. Read the following paragraph and answer the questions that follow:
To a vast number of tribal people, the forest is a home, a livelihood, their very existence. It
provides them food, fruits of all kinds, edible leaves, honey, nourishing roots and wild game. It
provides them with material to build their houses and items for practising their arts. The
importance of forests in tribal economy is well-known as they are the source of sustenance and
livelihood for tribal communities. It is commonly believed that the tribal communities live in
harmony with nature and protect forests. Forest and tribals are very closely related. The age-old
knowledge of tribals regarding forestry can be used in the development of forests. Rather than
treating tribals as minor forest produce collectors they should be made growers of minor forest
produce and encouraged to participate in conservation.

145
Q. Who live in harmony with nature?
a. wild animals
b. tribals
c. hunters and gatherers
d. all the above
Q. Forestry is an / a
a. burning of forests for shifting cultivation
b. way to develop forest
c. way to conserve forest
d. both b and c
Q. Which one of the following is not provided by forests?
a. edible leaves
b. wheat and barley
c. honey and fruits
d. edible roots

146
Chapter -6

NATURAL HAZARDS AND DISASTERS

MULTIPLE CHOICE QUESTIONS


1. Question: What causes earthquakes?
a) Tidal forces
b) Volcanic eruptions
c) Movement of tectonic plates
d) Meteor impacts
Answer: c) Movement of tectonic plates
2. Question: Which scale is commonly used to measure the magnitude of earthquakes?
a) Richter scale
b) Celsius scale
c) Beaufort scale
d) Fahrenheit scale
Answer: a) Richter scale
3. Question: What is the primary mitigation measure for earthquake hazards?
a) Early warning systems
b) Planting trees
c) Flood barriers
d) Crop rotation
Answer: a) Early warning systems
4. Question: What is a tsunami primarily triggered by?
a) Earthquakes under the ocean
b) Volcanic eruptions on land
c) High winds
d) Rapid snowmelt
Answer: a) Earthquakes under the ocean

5. Question: Which precaution can significantly reduce tsunami risks in coastal areas?
a) Building seawalls
b) Constructing tall buildings
c) Draining wetlands
d) Reducing fishing activities
Answer: a) Building seawalls
6. Question: What is the most common consequence of a tsunami?
a) Soil erosion
b) Oil spills
c) Flooding
d) Drought
Answer: c) Flooding

147
7. Question: Which of the following is a precautionary measure for cyclone preparedness?
a) Water conservation
b) Reinforcing roofs
c) Forest preservation
d) Road construction
Answer: b) Reinforcing roofs
8. Question: What is the eye of a cyclone?
a) The centre with calm and clear weather
b) A strong outer wall of the storm
c) The point of landfall
d) The tail end of the cyclone
Answer: a) The centre with calm and clear weather
9. Question: What is the primary cause of riverine floods?
a) Heavy snowfall
b) Monsoon rains
c) Volcanic eruptions
d) Oil spills
Answer: b) Monsoon rains
10. Question: What is a levee?
a) A type of boat
b) A barrier to prevent river flooding
c) A volcanic mountain
d) A type of forest
Answer: b) A barrier to prevent river flooding
11. Question: Which precaution can reduce the risk of flash floods?
a) Deforestation
b) Building on riverbanks
c) Proper drainage systems
d) Excessive irrigation
Answer: c) Proper drainage systems
12. Question: What is a common consequence of a prolonged drought?

a) Excessive water supply


b) Crop failure
c) Increased agricultural productivity
d) Reduced water conservation efforts
Answer: b) Crop failure
13. Question: What is a common mitigation measure for drought conditions?
a) Planting water-intensive crops
b) Implementing water rationing
c) Building water reservoirs
d) Encouraging excessive water usage
Answer: c) Building water reservoirs
14. Question: What can trigger a landslide?
a) Deforestation and heavy rainfall
b) Recycling and composting
c) Low humidity and mild temperatures
148
d) Urban development and road construction
Answer: a) Deforestation and heavy rainfall
ASSERTION AND REASON BASED QUESTIONS

1. Assertion: Tsunamis can be caused by heavy rainfall.


Reason: Heavy rainfall can lead to soil erosion and landslides, which, when occurring near
coastlines, can trigger tsunamis.
Options:
a) Both the assertion and reason are true.
b) The assertion is true, but the reason is false.
c) The assertion is false, but the reason is true.
d) Both the assertion and reason are false.
Answer: d) Both the assertion and reason are false.

2. Assertion: Earthquakes and tsunamis are unrelated natural disasters.


Reason: Earthquakes occur due to tectonic plate movements, while tsunamis result from
volcanic eruptions.
Options:
a) Both the assertion and reason are true.
b) The assertion is true, but the reason is false.
c) The assertion is false, but the reason is true.
d) Both the assertion and reason are false.
Answer: d) Both the assertion and reason are false.

3. Assertion: Droughts can lead to landslides in hilly regions.


Reason: Prolonged droughts can dry out soil and vegetation, making slopes more
vulnerable to landslides during heavy rainfall.
Options:
a) Both the assertion and reason are true.
b) The assertion is true, but the reason is false.
c) The assertion is false, but the reason is true.
d) Both the assertion and reason are false.
Answer: a) Both the assertion and reason are true.

4. Assertion: Cyclones rotate in a clockwise direction in the Northern Hemisphere.


Reason: The Coriolis effect causes cyclones to rotate clockwise in the Northern Hemisphere
and counterclockwise in the Southern Hemisphere.
Options:
a) Both the assertion and reason are true.
b) The assertion is true, but the reason is false.
c) The assertion is false, but the reason is true.
d) Both the assertion and reason are false.
Answer: a) Both the assertion and reason are true.
SOURCE BASED QUESTION
1. Read the given paragraph and answer the following questions;
The idea of an earthquake is often associated with fear and horror due to the scale,
magnitude and suddenness at which it spreads disasters on the surface of the earth without
149
discrimination. It becomes a calamity when it strikes the areas of high density of population.
It not only damages and destroys the settlements, infrastructure, transport and
communication network, industries and other developmental activities but also robs the
population of their material and socio-cultural gains that they have preserved over
generations. It renders them homeless, which puts an extra-pressure and stress, particularly
on the weak economy of the developing countries.

Question 1: Why is the idea of an earthquake often associated with fear and horror?

Answer 1: The idea of an earthquake is often associated with fear and horror due to its
scale, magnitude, and suddenness, which can spread disasters on Earth's surface without
discrimination.

Question 2: In what circumstances does an earthquake become a calamity?

Answer 2: An earthquake becomes a calamity when it strikes areas with a high density of
population.

Question 3: Apart from damaging settlements and infrastructure, what are the other
significant impacts of earthquakes, as mentioned in the paragraph?

Answer 3: In addition to damaging settlements and infrastructure, earthquakes also rob the
population of their material and socio-cultural gains that they have preserved over
generations.
2. Read the given paragraph and answer the following questions;
The speed of wave in the ocean depends upon the depth of water. It is more in the shallow
water than in the ocean deep. As a result of this, the impact of tsunami is less over the
ocean and more near the coast where they cause large-scale devastations. Therefore, a
ship at sea is not much affected by tsunami and it is difficult to detect a tsunami in the
deeper parts of sea. It is so because over deep water the tsunami has very long wave-
length and limited wave-height. Thus, a tsunami wave raises the ship only a metre or two
and each rise and fall takes several minutes. As opposed to this, when a tsunami enters
shallow water, its wave-length gets reduced and the period remains unchanged, which
increases the waveheight. Sometimes, this height can be up to 15m or more, which causes
large-scale destructions along the shores. Thus, these are also called Shallow Water
Waves. Tsunamis are frequently observed along the Pacific ring of fire, particularly along the
coast of Alaska, Japan, Philippines, and other islands of Southeast Asia, Indonesia,
Malaysia, Myanmar, Sri Lanka, and India etc
Question 1: Why is the impact of a tsunami less over the open ocean compared to near the
coast?

Answer 1: The impact of a tsunami is less over the open ocean because the speed of a
tsunami wave is higher in shallow water than in deep ocean water. Tsunamis cause large-
scale devastation near the coast because the wave height increases as they enter shallower
waters.

Question 2: Why is it difficult to detect a tsunami in the deeper parts of the sea?
150
Ans 2: It is difficult to detect a tsunami in the deeper parts of the sea because over deep
water, tsunamis have very long wavelengths and limited wave height. A tsunami wave
raises a ship only by a meter or two, and each rise and fall takes several minutes, making it
less noticeable.

Question 3: Why are tsunamis referred to as "Shallow Water Waves"?

Ans 3: Tsunamis are referred to as "Shallow Water Waves" because when a tsunami enters
shallow water, its wave length decreases while the period remains unchanged, leading to an
increase in wave height. This increase in wave height can be substantial, sometimes
reaching up to 15 meters or more, causing large-scale destruction along the shores.

LONG ANSWER QUESTIONS


Q1: Describe the primary causes of earthquakes and their distribution patterns. Discuss the
potential consequences of a major earthquake and the key mitigation measures that can be
implemented to reduce its impact.

Ans: Earthquakes are mainly caused by the movement of tectonic plates beneath the
Earth's surface. They often occur along plate boundaries, such as the Pacific Ring of Fire.
The consequences of a major earthquake can be catastrophic, including loss of life,
structural damage, and economic turmoil.
To mitigate these impacts, several measures can be taken.
*Early warning systems can provide valuable seconds or minutes for people to take cover.
*Building codes and construction standards can ensure that structures are earthquake-
resistant.
*Public education and preparedness campaigns can help communities be ready for seismic
events.

Q2 : Explain the processes that lead to the formation of tsunamis and their distribution
patterns. Discuss the potential consequences of a tsunami and the strategies for tsunami
mitigation and preparedness. (5 marks)

Ans: Tsunamis are often triggered by underwater earthquakes, volcanic eruptions, or


landslides. They are most common in regions along tectonic plate boundaries and the
Pacific Ring of Fire.
Tsunamis can result in coastal flooding, damage to infrastructure, and loss of life.
Mitigation strategies include:
*early warning systems that detect undersea seismic activity and provide alerts to coastal
communities.
*Land-use planning can discourage construction in vulnerable areas.
* Public education efforts can teach residents how to respond in the event of a tsunami,
including evacuation routes and shelter locations.

Q3 : Discuss the factors that contribute to the formation of cyclones and their distribution
patterns. Explain the potential consequences of a cyclone and the measures for cyclone
151
preparedness and management. (5 marks)

Ans: Cyclones, also known as hurricanes or typhoons, develop over warm ocean waters,
typically in tropical regions. They are characterized by low-pressure systems and high
winds.
Cyclones often occur in areas like the Atlantic and Pacific Oceans. Consequences of
cyclones include powerful winds that can destroy buildings and infrastructure, heavy rainfall
leading to flooding, and storm surges that inundate coastal areas.
To prepare for cyclones,
*communities can establish early warning systems,
*build storm shelters, and create evacuation plans.
* Emergency response teams should be trained and equipped to handle cyclone-related
emergencies.

Q4: Explain the causes of riverine and flash floods and their distribution patterns. Discuss
the potential impacts of floods on communities, infrastructure, and the environment. Outline
strategies for flood mitigation and effective response. (5 marks)

Ans: Riverine floods result from prolonged heavy rainfall or snowmelt, often affecting regions
near rivers and their tributaries.
Flash floods are sudden, intense events triggered by heavy rainfall or dam failures.
The distribution of floods depends on local geography and weather patterns. Floods can
have significant consequences, including property damage, displacement of people, and
disruption of transportation and utilities. Mitigation measures include
*floodplain zoning, construction of levees and dams, and
*improved stormwater management.
Effective response strategies involve
the early warning system,
swift evacuations, and
disaster relief efforts to provide aid to affected population.

Q5 : Describe the factors that contribute to drought conditions and their distribution patterns.
Discuss the long-term consequences of drought on agriculture, water supply, ecosystems,
and society. Outline strategies for drought management and building resilience. (5 marks)

Ans: Droughts result from prolonged periods of below-average rainfall, often exacerbated by
climate variability and human activities such as deforestation and excessive water use.
Distribution patterns vary regionally.
Droughts can have far-reaching consequences, including crop failure, reduced water supply,
damage to ecosystems, and social and economic impacts.
To manage drought,
*water conservation measures such as efficient irrigation techniques and
*water reuse can be implemented.
*Diversifying agriculture with drought-resistant crops can build resilience. *Education
campaigns can promote water-saving behaviors among the population.

152
MAP BASED QUESTION
1. Study the map carefully and answer the following questions

153
1. Name the region in India which falls under very high damage risk zone?
Ans: North Eastern India
2. Under which earthquake zone does Himalaya come ?
Ans: High damage risk zone
3. Name any one region in India which falls under Very low damage risk zone?
Ans: Deccan Plateau.

2. Study the map carefully and answer the following questions

1. Name the river in North East India which causes flood.


Ans. R. Brahmaputra
2. Name the river which brings flood in Odisha.
Ans. R. Mahanadi
3. River Ganga brings flood in the states of……..
Ans. U.P, Bihar, West Bengal

***

154

You might also like